You are on page 1of 94

MERCANTILE LAW REVIEW || Insurance Law 1

Compilation of Digests (Except 37)

INTERPRETATION respondent their claims under their respective fire insurance


policies with book debt endorsements; that as of February 25,
1. GAISANO CAGAYAN, INC. vs. INSURANCE COMPANY OF 1991, the unpaid accounts of petitioner on the sale and delivery
NORTH AMERICA,  of ready-made clothing materials with IMC was P2,119,205.00
G.R. No. 147839, June 8, 2006, AUSTRIA-MARTINEZ, J.: while with LSPI it was P535,613.00; that respondent paid the
claims of IMC and LSPI and, by virtue thereof, respondent was
When the words of a contract are plain and readily understood, subrogated to their rights against petitioner; that respondent
there is no room for construction. Indeed, when the terms of the made several demands for payment upon petitioner but these
agreement are clear and explicit that they do not justify an went unheeded
attempt to read into it any alleged intention of the parties, the On August 31, 1998, the RTC rendered its decision dismissing
terms are to be understood literally just as they appear on the face respondent's complaint.
of the contract. Dissatisfied, an appeal was taken to the CA, and the CA reversed
the decision rendered by the RTC.
Facts:
Issue:
Intercapitol Marketing Corporation (IMC) is the maker of WON the Interpretation of the insurance policy by the CA is
Wrangler Blue Jeans. Levi Strauss (Phils.) Inc. (LSPI) is the local correct?
distributor of products bearing trademarks owned by Levi
Strauss & Co.. IMC and LSPI separately obtained from respondent Ruling:
fire insurance policies with book debt endorsements. The
insurance policies provide for coverage on "book debts in The Court disagrees with petitioner's stand.
connection with ready-made clothing materials which have been
sold or delivered to various customers and dealers of the Insured It is well-settled that when the words of a contract are plain and
anywhere in the Philippines." The policies defined book debts as readily understood, there is no room for construction. In this
the "unpaid account still appearing in the Book of Account of the case, the questioned insurance policies provide coverage for
Insured 45 days after the time of the loss covered under this "book debts in connection with ready-made clothing materials
Policy. Petitioner is a customer and dealer of the products of IMC which have been sold or delivered to various customers and
and LSPI. On February 25, 1991, the Gaisano Superstore Complex dealers of the Insured anywhere in the Philippines.; and defined
in Cagayan de Oro City, owned by petitioner, was consumed by book debts as the "unpaid account still appearing in the Book of
fire. Included in the items lost or destroyed in the fire were Account of the Insured 45 days after the time of the loss covered
stocks of ready-made clothing materials sold and delivered by under this Policy." Nowhere is it provided in the questioned
IMC and LSPI. insurance policies that the subject of the insurance is the goods
sold and delivered to the customers and dealers of the insured.
On February 4, 1992, respondent filed a complaint for damages
against petitioner. It alleges that IMC and LSPI filed with Indeed, when the terms of the agreement are clear and explicit
that they do not justify an attempt to read into it any alleged
MERCANTILE LAW REVIEW || Insurance Law 2
Compilation of Digests (Except 37)

intention of the parties, the terms are to be understood literally IMC and LSPI did not lose complete interest over the goods. They
just as they appear on the face of the contract. Thus, what were have an insurable interest until full payment of the value of the
insured against were the accounts of IMC and LSPI with delivered goods. Unlike the civil law concept of res perit domino,
petitioner which remained unpaid 45 days after the loss through where ownership is the basis for consideration of who bears the
fire, and not the loss or destruction of the goods delivered. risk of loss, in property insurance, one's interest is not
determined by concept of title, but whether insured has
Petitioner argues that IMC bears the risk of loss because it substantial economic interest in the property.
expressly reserved ownership of the goods by stipulating in the
sales invoices that "[i]t is further agreed that merely for purpose
of securing the payment of the purchase price the above
described merchandise remains the property of the vendor until
the purchase price thereof is fully paid."

The Court is not persuaded.

The present case clearly falls under paragraph (1), Article 1504
of the Civil Code:

ART. 1504. Unless otherwise agreed, the goods remain at the


seller's risk until the ownership therein is transferred to the
buyer, but when the ownership therein is transferred to the
buyer the goods are at the buyer's risk whether actual delivery
has been made or not, except that:

(1) Where delivery of the goods has been made to the buyer or to
a bailee for the buyer, in pursuance of the contract and the
ownership in the goods has been retained by the seller merely to
secure performance by the buyer of his obligations under the
contract, the goods are at the buyer's risk from the time of such
delivery.

Thus, when the seller retains ownership only to insure that the
buyer will pay its debt, the risk of loss is borne by the buyer.
Accordingly, petitioner bears the risk of loss of the goods
delivered.
MERCANTILE LAW REVIEW || Insurance Law 3
Compilation of Digests (Except 37)

2. MALAYAN INSURANCE CORPORATION vs. THE HON. TKC Marketing Corp. accordingly, advised petitioner that it might
COURT OF APPEALS and TKC MARKETING CORPORATION tranship the cargo and requested an extension of the insurance
coverage until actual transhipment, which extension was
G.R. No. 119599 March 20, 1997 ROMERO, J. approved upon payment of additional premium. The insurance
coverage was extended under the same terms and conditions
Indemnity and liability insurance policies are construed in embodied in the original policies while in the process of making
accordance with the general rule of resolving any ambiguity arrangements for the transhipment of the cargo from Durban to
therein in favor of the insured, where the contract or policy is Manila, covering the period October 4 - December 19, 1989.
prepared by the insurer.  A contract of insurance, being a contract
of adhesion, par excellence, any ambiguity therein should be However, on December 11, 1989, the cargo was sold in Durban,
resolved against the insurer; in other words, it should be construed South Africa, for US$154.40 per metric ton or a total of
liberally in favor of the insured and strictly against the insurer. P10,304,231.75 due to its perishable nature which could no
Limitations of liability should be regarded with extreme jealousy longer stand a voyage of twenty days to Manila and another
and must be construed in such a way as to preclude the insurer twenty days for the discharge thereof. On January 5, 1990,
from noncompliance with its obligations.  private respondent forthwith reduced its claim to US$448,806.09
(or its peso equivalent of P9,879,928.89 at the exchange rate of
FACTS: P22.0138 per $1.00) representing private respondent's loss after
the proceeds of the sale were deducted from the original claim of
TKC Marketing Corp. was the owner/consignee of some $916,886.66 or P20,184,159.55.
3,189.171 metric tons of soya bean meal which was loaded on
board the ship MV Al Kaziemah on or about September 8, 1989 Petitioner maintained its position that the arrest of the vessel by
for carriage from the port of Rio del Grande, Brazil, to the port of civil authorities on a question of ownership was an excepted risk
Manila. Said cargo was insured against the risk of loss by under the marine insurance policies.
petitioner Malayan Insurance Corporation for which it issued
two (2) Marine Cargo policy Nos. M/LP 97800305 amounting to Lower court and CA decided in favor of TKC Marketing
P18,986,902.45 and M/LP 97800306 amounting to Corporation.
P1,195,005.45, both dated September 1989. ISSUES:

While the vessel was docked in Durban, South Africa on a. Whether or not the arrest of the vessel was a risk covered
September 11, 1989 enroute to Manila, the civil authorities under the subject insurance policies.
arrested and detained it because of a lawsuit on a question of b. Whether or not insurance policies must be strictly
ownership and possession. According to Malayan Insurance construed against the insurer.
Corporation, the arrest of the vessel by civil authority was not a
peril covered by the policies. RULING:
a. YES.
MERCANTILE LAW REVIEW || Insurance Law 4
Compilation of Digests (Except 37)

By way of a historical background, marine insurance developed Should Clause 12 be deleted, the relevant current
as an all-risk coverage, using the phrase "perils of the sea" to institute war clauses shall be deemed to form part of this
encompass the wide and varied range of risks that were covered. insurance.
The subject policies contain the "Perils" clause which is a
standard form in any marine insurance policy. Said clause reads: However, the F. C. & S. Clause was deleted from the policies.
Consequently, the Institute War Clauses (Cargo) was deemed
“they are of the Seas; Men-of-War, Fire, Enemies, incorporated.
Pirates, Rovers, Thieves, Jettisons, Letters of Mart and
Counter Mart, Suprisals, Takings of the Sea, Arrests, Petitioner cannot adopt the argument that the "arrest" caused by
Restraints and Detainments of all Kings, Princess and ordinary judicial process is not included in the covered risk
Peoples, of what Nation, Condition, or quality soever, simply because the F.C. & S. Clause under the Institute War
Barratry of the Master and Mariners, and of all other Perils, Clauses can only be operative in case of hostilities or warlike
Losses, and Misfortunes, that have come to hurt, detriment, operations on account of its heading "Institute War Clauses."
or damage of the said goods and merchandise or any part This Court agrees with the Court of Appeals when it held that ". . .
thereof . AND in case of any loss or misfortune it shall be . Although the F.C. & S. Clause may have originally been inserted
lawful to the ASSURED, their factors, servants and assigns, in marine policies to protect against risks of war, (see generally
G. Gilmore & C. Black, The Law of Admiralty Section 2-9, at 71-73
to sue, labour, and travel for, in and about the defence,
[2d Ed. 1975]), its interpretation in recent years to include
safeguards, and recovery of the said goods and
seizure or detention by civil authorities seems consistent with
merchandises, and ship, & c., or any part thereof, without the general purposes of the clause, . . . ."  In fact, petitioner itself
prejudice to this INSURANCE; to the charges whereof the averred that subsection 1.1 of Section 1 of the Institute War
said COMPANY, will contribute according to the rate and Clauses included "arrest" even if it were not a result of hostilities
quantity of the sum herein INSURED. AND it is expressly or warlike operations. 6 In this regard, since what was also
declared and agreed that no acts of the Insurer or Insured excluded in the deleted F.C. & S. Clause was "arrest" occasioned
in recovering, saving, or preserving the Property insured by ordinary judicial process, logically, such "arrest" would now
shall be considered as a Waiver, or Acceptance of become a covered risk under subsection 1.1 of Section 1 of the
Abandonment.” Institute War Clauses, regardless of whether or not said "arrest"
by civil authorities occurred in a state of war.
The exception or limitation to the "Perils" clause and the "All
other perils" clause in the subject policies is specifically referred
to as Clause 12 called the "Free from Capture & Seizure Clause"
b. YES, insurance policies must be strictly construed
or the F.C. & S. Clause.
against the insurer.
MERCANTILE LAW REVIEW || Insurance Law 5
Compilation of Digests (Except 37)

It has been held that a strained interpretation which is


unnatural and forced, as to lead to an absurd conclusion or to
render the policy nonsensical, should, by all means, be
avoided.  Likewise, it must be borne in mind that such
contracts are invariably prepared by the companies and
must be accepted by the insured in the form in which they
are written.  Any construction of a marine policy rendering it
void should be avoided.  Such policies will, therefore, be
construed strictly against the company in order to avoid a
forfeiture, unless no other result is possible from the
language used. 

If a marine insurance company desires to limit or restrict the


operation of the general provisions of its contract by special
proviso, exception, or exemption, it should express such
limitation in clear and unmistakable language.  Obviously, the
deletion of the F.C. & S. Clause and the consequent
incorporation of subsection 1.1 of Section 1 of the Institute
War Clauses (Cargo) gave rise to ambiguity. If the risk of
arrest occasioned by ordinary judicial process was expressly
indicated as an exception in the subject policies, there would
have been no controversy with respect to the interpretation
of the subject clauses.

Indemnity and liability insurance policies are construed in


accordance with the general rule of resolving any ambiguity
therein in favor of the insured, where the contract or policy is
prepared by the insurer.  A contract of insurance, being a
contract of adhesion, par excellence, any ambiguity therein
should be resolved against the insurer; in other words, it
should be construed liberally in favor of the insured and
strictly against the insurer. Limitations of liability should be
regarded with extreme jealousy and must be construed in
such a way as to preclude the insurer from noncompliance
with its obligations. 
MERCANTILE LAW REVIEW || Insurance Law 6
Compilation of Digests (Except 37)

3. Verendia vs. CA Court of Appeals 168 SCRA 1 [1988]). Its terms and conditions
G.R. No. 75605 January 22, 1993 constitute the measure of the insurer's liability and compliance
therewith is a condition precedent to the insured's right to
An insurance contract should be liberally construed in favor of the recovery from the insurer (Oriental Assurance Corporation vs.
insured and strictly against the insurer company, which usually Court of Appeals, 200 SCRA 459 [1991], citing Perla Compania de
prepares it. Seguros, Inc. vs. Court of Appeals, 185 SCRA 741 [1991]). As it is
also a contract of adhesion, an insurance contract should be
Facts: liberally construed in favor of the insured and strictly against the
Rafael Verendia’s residential building was insured with Fidelity insurer company, which usually prepares it (Western Guaranty
and Surety Incsurance Company and two others, Country Corporation vs. Court of Appeals, 187 SCRA 652 [1980]).
Bankers Insurance and Development Insurance, with Monte de Considering, however, the foregoing discussion pointing
Piedad and Savings Bank as beneficiary. The insured building to the fact that Verendia used a false lease contract to support his
was completely destroyed by fire. With this, the petitioner claim claim under Fire Insurance Policy No. F-18876, the terms of the
for insurance on which Fidelity refused to give depending on its policy should be strictly construed against the insured. Verendia
issued Fire Insurance Policy F-1887. Fidelity, among other failed to live by the terms of the policy, specifically Section 13
things, averred that the policy was avoided by reason of over- thereof which is expressed in terms that are clear and
insurance., that Verendia maliciously represented that the unambiguous, that all benefits under the policy shall be forfeited
building at the time of the fire were leases under a contract "If the claim be in any respect fraudulent, or if any false
executed to a certain Roberto Garcia, when actually it was a declaration be made or used in support thereof, or if any
Marcelo Garcia who was in the Lease. fraudulent means or devises are used by the Insured or anyone
acting in his behalf to obtain any benefit under the policy."
Issue:
Whether or not Verendia forfeited all Benefits due to his
presentation of false declaration to support his claim.

Held:
Yes. Verendia, having presented a false declaration to
support his claim for benefits in the form of a fraudulent lease
contract, he forfeited all benefits therein by virtue of Section 13
of the policy in the absence of proof that Fidelity waived such
provision. Worse yet, by presenting a false lease contract,
Verendia, reprehensibly disregarded that principle that
insurance contracts are uberrimae fidae and demand the most
abundant good faith.
Basically a contract of indemnity, an insurance contract is
the law between the parties (Pacific Banking Corporation vs.
MERCANTILE LAW REVIEW || Insurance Law 7
Compilation of Digests (Except 37)

4. New Life Enterprises v. Court of Appeals RULING


207 SCRA 669
YES. The Supreme Court ruled that the denial of the claim
A policy or contract of insurance is to be construed liberally in is proper.
favor of the insured and strictly against the insurer company, yet
contracts of insurance, like other contracts, are to be construed The SC explained that the terms of the contract are clear and
according to the sense and meaning of the terms which the parties unambiguous. The insured is specifically required to disclose to
themselves have used. If such terms are clear and unambiguous, the insurer any other insurance and its particulars which he may
they must be taken and understood in their plain, ordinary and have effected on the same subject matter. Furthermore, when the
popular sense. words and language of documents are clear and plain or readily
understandable by an ordinary reader thereof, there is
FACTS absolutely no room for interpretation or construction anymore.

Ms. Julian Sy and Jose Sy Bong have a business partnership in the Petitioners should be aware of the fact that a party is not relieved
City of Lucena under the business name of New Life Enterprises. of the duty to exercise the ordinary care and prudence that
New Life Enterprises engaged in the sale of construction would be exacted in relation to other contracts. The conformity
materials at its two story building. Julian Sy insured their stocks of the insured to the terms of the policy is implied from failure to
in trade with Western Guaranty Corporation, Reliance Surety express any disagreement with what is provided for. It may be
and Insurance Co., Inc., and Equitable Insurance Corporation.The true that the majority rule is that injured persons may accept
building occupied by the New Life Enterprises was gutted by fire, policies without reading them, and that his is not negligence per
the stocks inside the building were burned. After the fire, Julian se. But, this is not without any exception. It is and was incumbent
Sy went to the three insurance companies who were sister upon petitioner Sy to read the insurance contracts and this can
companies to file her claim but the insurance companies denied be reasonably expected on him considering that he has been a
the claim on the ground that Julilan Sy violated the “Other businessman since 1965.
Insurance Clause” that was included in the 3 separate policy. The
other insurance clause stated that the insured shall give notice to
the Company of any insurance or insurances already effected to
the same property.

ISSUE

Whether or not the denial of the claim is proper on the ground of


the “Other Insurance Clause.”
MERCANTILE LAW REVIEW || Insurance Law 8
Compilation of Digests (Except 37)

5. DIOSDADO TY v. FILIPINAS COMPAÑIA DE SEGUROS, et al. is argued that what is compensable is the disability and not the
G.R. No. L-21821-22 and L-21824-27, 31 May 1966, amputation of the hand. The definition of what constitutes loss of
BARRERA, J. hand, placed in the contract, according to appellant,
consequently, makes the provision ambiguous and calls for the
“[T]he agreement contained in the insurance policies is the law interpretation thereof by this Court.
between the parties. As the terms of the policies are clear, express
and specific that only amputation of the left hand should be ISSUE: How should the courts interpret an insurance policy
considered as a loss thereof, an interpretation that would include if ambiguity is raised by a party?
the mere fracture or other temporary disability not covered by the
policies would certainly be unwarranted.” RULING:
The agreement contained in the insurance policies is the
FACTS: Diosdado Ty was a mechanic operator of Broadway law between the parties. As the terms of the policies are clear,
Cotton Factory at Grace Park, Caloocan City. In the latter part of express and specific that only amputation of the left hand should
1953, he took Personal Accident Policies from several insurance be considered as a loss thereof, an interpretation that would
companies, among which are herein defendants-appellees, on include the mere fracture or other temporary disability not
different dates, effective for 12 months. During the effectivity of covered by the policies would certainly be unwarranted.
these policies, or on December 24, 1953, a fire broke out in the While we sympathize with the plaintiff or his employer,
factory where Ty was working. As he was trying to put out said for whose benefit the policies were issued, we can not go beyond
fire with the help of a fire extinguisher, a heavy object fell upon the clear and express conditions of the insurance policies, all of
his left hand. which definite partial disability as loss of either hand
Ty received treatment at the National Orthopedic by amputation through the bones of the wrist. There was no such
Hospital from, most of which were merely fractures of his left amputation in the case at bar. All that was found by the trial
fingers, but no more than the amputation of any part thereof. court, which is not disputed on appeal, was that the physical
The attending surgeon certified that these injuries would cause injuries "caused temporary total disability of plaintiff's left
temporary total disability of appellant's left hand. The insurance hand." Note that the disability of plaintiff's hand was merely
companies refused to pay his claim for compensation on the temporary, having been caused by fractures of the index, the
ground that under the uniform terms of the insurance policies, middle and the fourth fingers of the left hand.
partial disability of the insured caused by loss of either hand to We find no reason to depart from the foregoing ruling on
be compensable, the loss must result in the amputation of that the matter.  Plaintiff-appellant cannot come to the courts and
hand. Also, under the insurance contract signed by Ty, it was claim that he was misled by the terms of the contract. The
stipulated that: “The loss of a hand shall mean the loss, by provision is clear enough to inform the party entering into that
amputation through the bones of the wrist.” contract that the loss to be considered a disability entitled to
Ty contends that to be entitled to indemnification under indemnity, must be severance or amputation of that affected
the foregoing provision, it is enough that the insured is disabled member from the body of the insured.
to such an extent that he cannot substantially perform all acts or
duties of the kind necessary in the prosecution of his business. It
MERCANTILE LAW REVIEW || Insurance Law 9
Compilation of Digests (Except 37)

6. GULF RESORTS, INC. vs. PHILIPPINE CHARTER INSURANCE intention of the parties to extend earthquake shock coverage to
CORPORATION the two swimming pools only.A careful examination of the
G.R. No. 156167 May 16, 2005 J. Puno premium recapitulation will show that it is the clear intent of the
A contract of adhesion is one wherein a party, usually a parties to extend earthquake shock coverage only to the two
corporation, prepares the stipulations in the contract, while the swimming pools.
other party merely affixes his signature or his "adhesion" thereto. Petitioner cannot rely on the general rule that insurance
The parties do not bargain on equal footing, the weaker party's contracts are contracts of adhesion which should be liberally
participation being reduced to the alternative to take it or leave it. construed in favor of the insured and strictly against the insurer
Thus, these contracts are viewed as traps for the weaker party company which usually prepares it. A contract of adhesion is one
whom the courts of justice must protect. wherein a party, usually a corporation, prepares the stipulations
in the contract, while the other party merely affixes his signature
FACTS: or his "adhesion" thereto. Through the years, the courts have
Gulf was insured by American Home Assurance Company which held that in these type of contracts, the parties do not bargain on
includes loss or damage to shock to any of the property insured equal footing, the weaker party's participation being reduced to
by the Policy occasioned by or through in consequence of the alternative to take it or leave it. Thus, these contracts are
earthquake. Later on, an earthquake struck Central and Northern viewed as traps for the weaker party whom the courts of justice
Luzon causing damage to the property of Gulf. Gulf claimed from must protect. Consequently, any ambiguity therein is resolved
the insurance company but was denied as the earthquake shock against the insurer, or construed liberally in favor of the insured.
coverage only covered the swimming pools of the resort. Gulf The case law will show that this Court will only rule out
contends that pursuant to this rider, no qualifications were blind adherence to terms where facts and circumstances will
placed on the scope of the earthquake’s shock coverage. Thus, show that they are basically one-sided. Thus, we have called on
the policy extended to all of the insured properties. The RTC lower courts to remain careful in scrutinizing the factual
ruled in favor of American Home to which the CA affirmed. circumstances behind each case to determine the efficacy of the
claims of contending parties. In Development Bank of the
ISSUE: Whether or not Gulf was entitled to claim. Philippines v. National Merchandising Corporation, et al., the
parties, who were acute businessmen of experience, were
HELD: NO, Gulf was entitled to claim. presumed to have assented to the assailed documents with full
It is basic that all the provisions of the insurance policy knowledge.
should be examined and interpreted in consonance with each We cannot apply the general rule on contracts of
other. All its parts are reflective of the true intent of the parties. adhesion to the case at bar. Petitioner cannot claim it did not
The policy cannot be construed piecemeal. Certain stipulations know the provisions of the policy. From the inception of the
cannot be segregated and then made to control; neither do policy, petitioner had required the respondent to copy verbatim
particular words or phrases necessarily determine its character. the provisions and terms of its latest insurance policy from
Petitioner cannot focus on the earthquake shock endorsement to AHAC-AIU. The testimony of Mr. Leopoldo Mantohac, a direct
the exclusion of the other provisions. All the provisions and participant in securing the insurance policy of petitioner, is
riders, taken and interpreted together, indubitably show the reflective of petitioner’s knowledge.
MERCANTILE LAW REVIEW || Insurance Law 10
Compilation of Digests (Except 37)

7. SIMON DE LA CRUZ vs. THE CAPITAL INSURANCE AND


SURETY CO. INC. HELD: NO. The terms "accident" and "accidental", as used in
G.R. No. L-21574 June 30, 1966 J. BARRERA insurance contracts, have not acquired any technical meaning,
The terms "accident" and "accidental", as used in insurance and are construed by the courts in their ordinary and common
contracts, have not acquired any technical meaning, and are acceptation. Thus, the terms have been taken to mean that which
construed by the courts in their ordinary and common happen by chance or fortuitously, without intention and design,
acceptation. Thus, the terms have been taken to mean that which and which is unexpected, unusual, and unforeseen. An accident is
happen by chance or fortuitously, without intention and design, an event that takes place without one's foresight or expectation
and which is unexpected, unusual, and unforeseen. An accident is — an event that proceeds from an unknown cause, or is an
an event that takes place without one's foresight or expectation — unusual effect of a known cause and, therefore, not expected.
an event that proceeds from an unknown cause, or is an unusual It may be mentioned in this connection, that the tendency
effect of a known cause and, therefore, not expected. of court decisions in the United States in the recent years is to
eliminate the fine distinction between the terms "accidental" and
FACTS: Eduardo de la Cruz, employed as a mucker in the Itogon- "accidental means" and to consider them as legally synonymous.
Suyoc Mines, Inc. in Baguio, was the holder of an accident The generally accepted rule is that, death or injury does
insurance policy underwritten by the Capital Insurance & Surety not result from accident or accidental means within the terms of
Co., Inc., In connection with the celebration of the New Year, the an accident-policy if it is the natural result of the insured's
Itogon-Suyoc Mines, Inc. sponsored a boxing contest for general voluntary act, unaccompanied by anything unforeseen except the
entertainment wherein the insured Eduardo de la Cruz, a non- death or injury. There is no accident when a deliberate act is
professional boxer participated. In the course of his bout with performed unless some additional, unexpected, independent, and
another person, likewise a non-professional, of the same height, unforeseen happening occurs which produces or brings about
weight, and size, Eduardo slipped and was hit by his opponent on the result of injury or death. 4 In other words, where the death
the left part of the back of the head, causing Eduardo to fall, with or injury is not the natural or probable result of the insured's
his head hitting the rope of the ring. He was brought to the voluntary act which produces the injury, the resulting death is
Baguio General Hospital where he expired the following day. The within the protection of policies insuring against the death or
cause of death was reported as hemorrhage, intracranial, left. injury from accident.
Simon de la Cruz, the father of the insured and who was In the present case, while the participation of the insured
named beneficiary under the policy, thereupon filed a claim with in the boxing contest is voluntary, the injury was sustained when
the insurance company for payment of the indemnity under the he slid, giving occasion to the infliction by his opponent of the
insurance policy blow that threw him to the ropes of the ring. Without this
Defendant insurer set up the defense that the death of the unfortunate incident, that is, the unintentional slipping of the
insured, caused by his participation in a boxing contest, was not deceased, perhaps he could not have received that blow in the
accidental and, therefore, not covered by insurance. head and would not have died. The fact that boxing is attended
with some risks of external injuries does not make any injuries
ISSUE: Whether the death of Eduardo was accidental since he received in the course of the game not accidental.
entered in the boxing contest voluntarily.
MERCANTILE LAW REVIEW || Insurance Law 11
Compilation of Digests (Except 37)

BENEFICIARIES The trial court held that the petitioners cannot invoke the law on
donations or the rules on testamentary succession in order to
8. HEIRS OF LORETO C. MARAMAG, represented by surviving defeat the right of herein defendants to collect the insurance
spouse VICENTA PANGILINAN MARAMAG vs. EVA VERNA DE indemnity. The beneficiary in a contract of insurance is not the
GUZMAN MARAMAG, et al, donee spoken in the law of donation. The rules on testamentary
G.R. No. 181132, June 5, 2009 J.NACHURA succession cannot apply here, for the insurance indemnity does
not partake of a donation.
Insurance proceeds shall be applied exclusively to the proper
It also stated that the proceeds to the Life Insurance Policy
interest of the person in whose name or for whose benefit it is
belongs exclusively to the defendant as his individual and
made unless otherwise specified in the policy. (Sec. 53, Insurance
separate property, and not to the estate of the person whose life
Code)
was insured.
FACTS: The RTC disqualifed Loreto’s concubine, Eva, from being a
benficiary pursuant to Art. 2012: Any person who is forbidden
Petitioners were the legitimate wife and children of Loreto
from receiving any donation under Article 739 cannot be named
Maramag (Loreto), while respondents were Loretos illegitimate
beneficiary of a life insurance policy of the person who cannot
family. Loreto designated respondents as beneficiaries in his life
make any donation to him, but stated that the insurance contract
insurance policies from Insular Life and Great Pacific Life
will still remain valid, but the indemnity must go to the legal
(Grepalife).
heirs and not to the concubine, for evidently, what is prohibited
Petitioners insituted in the RTC a petition for revocation and/or under Art. 2012 is the naming of the improper beneficiary.
reduction of insurance proceeds for being void and/or
inofficious, with prayer for a temporary restraining order (TRO)
ISSUE:
and a writ of preliminary injunction. They claim that Eva de
1. Are the members of the legitimate family entitled to the
Guzman Maramag (Eva) was a concubine of Loreto and a suspect
proceeds of the insurance for the concubine?
in the killing of the latter, thus, she is disqualified to receive any
proceeds from his insurance policies; the illegitimate children of
2. Whether or not illegitimate children can be beneficiaries in an
Loreto were entitled only to one-half of the legitime of the
insurance contract.
legitimate children, thus, the proceeds released to them were
inofficious and should be reduced; and petitioners could not be
deprived of their legitimes, which should be satisfied first. HELD:

Petitioners allege that the designation of a beneficiary is an act of 1. NO. Section 53 of the Insurance Code states that the insurance
liberality or a donation and, therefore, subject to the provisions proceeds shall be applied exclusively to the proper interest of the
of Articles 752[8] and 772[9] of the Civil Code. person in whose name or for whose benefit it is made unless
otherwise specified in the policy.
MERCANTILE LAW REVIEW || Insurance Law 12
Compilation of Digests (Except 37)

Pursuant thereto, it is obvious that the only persons entitled to


claim the insurance proceeds are either the insured, if still alive;
or the beneficiary, if the insured is already deceased, upon the
maturation of the policy.The exception to this rule is a situation
where the insurance contract was intended to benefit third
persons who are not parties to the same in the form of favorable
stipulations or indemnity. In such a case, third parties may
directly sue and claim from the insurer.
Petitioners are third parties to the insurance contracts with
Insular and Grepalife and, thus, are not entitled to the proceeds
thereof.
2. YES. The revocation of Eva as a beneficiary in one policy and
her disqualification as such in another are of no moment
considering that the designation of the illegitimate children as
beneficiaries in Loretos insurance policies remains valid.
Because no legal proscription exists in naming as beneficiaries
the children of illicit relationships by the insured, the shares of
Eva in the insurance proceeds must be awarded to the said
illegitimate children, the designated beneficiaries, to the
exclusion of petitioners. It is only in cases where the insured has
not designated any beneficiary, or when the designated
beneficiary is disqualified by law to receive the proceeds, that
the insurance policy proceeds shall redound to the benefit of the
estate of the insured.
MERCANTILE LAW REVIEW || Insurance Law 13
Compilation of Digests (Except 37)

9. SOUTHERN LUZON EMPLOYEES' ASSOCIATION vs. JUANITA


GOLPEO, ET AL. HELD:
G.R. No. L-6114 October 30, 1954 C.J. Paras YES. The children are entitled to claim.

The contract of life insurance is a special contract and the Inasmuch as, according to article 739 of the new Civil Code, a
destination of the proceeds thereof is determined by special laws donation is valid when made "between persons who are guilty or
which deal exclusively with that subject. The Civil Code has no adultery or concubinage at the time of the donation," it is alleged
provisions which relate directly and specifically to life-insurance that the defendant-appellee Aquilina Maloles, cannot be named a
contract or to the destination of life-insurance proceeds. That beneficiary, every assuming that the insurance law is applicable.
subject is regulate exclusively by the Code of Commerce which Without considering the intimation in the brief for the defendant
provides for the terms of the contract, the relations of the parties appellees that appellant Juanita Golpeo, by her silence and
and the destination of the proceeds of the policy. actions, had acquiesced in the illicit relations between her
husband and appellee Aquilina Maloles, appellant argument
FACTS: would certainly not apply to the children of Aquilina likewise
named beneficiaries by the deceased Roman A. Concepcion. As a
One of SLEA’s purpose is mutual aid of its members in case of matter of a fact the new Civil Code recognized certain
death. Roman A. Concepcion was a member until his death. SLEA successional rights of illegitimate children.
adopted a resolution with respect to claims in case of death
stating that a member may name his common-law wife as The contract of life insurance is a special contract and the
beneficiary and children he we had with her and that SLEA will destination of the proceeds thereof is determined by special laws
recognize only the same as the beneficiary entitled to condolence which deal exclusively with that subject. The Civil Code has no
contributions. Roman listed as his eneficiaries Aquilina Maloles, provisions which relate directly and specifically to life-insurance
Roman Jr., Esela, Rolando and Robin. After the death of Roman, contract or to the destination of life-insurance proceeds. That
the association was able to collect contributions from its subject is regulate exclusively by the Code of Commerce which
members amounting to 2505. Three sets of claimants presented provides for the terms of the contract, the relations of the parties
themselves: Juanita Golpeo, Roman’s legal wife and her children and the destination of the proceeds of the policy.
and Elsie Hicban, another common law wife of Roman and her
children. SLEA filed an interpleader with the CFI of Laguna to
determine who is entitled to the claim. The court rendered a
decision declaring Aquilina and her children the sole beneficiary
of P2,505 and ordering SLEA to deliver the same.

ISSUE:
Whether or not the children of Aquilina are entitled to claim.
MERCANTILE LAW REVIEW || Insurance Law 14
Compilation of Digests (Except 37)

10. Basilia Berdin Vda. De Consuegra; Juliana, Pacita, Maria who were the beneficiaries named in the policy. Having been in
Lourdes, Jose, JR., Rodrigo, Lineda and Luis, all surnamed the service of the government for 22.5028 years, Consuegra was
CONSUEGRA vs. Government Service Insurance System, entitled to retirement insurance benefits in the sum of P6,
Commisioner of Public Highways, Highway District Engineer 304.47, however, he did not designate any beneficiary who
of Surigao Del Norte, Commissioner of Civil Service, and would receive the retirement insurance benefits due to him.
Rosario Diaz
G.R. No. L-28093 January 30, 1971 J. Zaldivar Respondent Diaz, the widow (1st), filed a claim with the GSIS
asking that the retirement insurance benefits be paid to her as
The insured in a life insurance may designate any person as the only legal heir. Petitioner Berdin and her children, likewise,
beneficiary unless disqualified to be so under the provisions of the filed a similar claim with the GSIS, asserting that being the
Civil Code. In the absence of any beneficiary named in the life beneficiaries named in the life insurance policy, they are the only
insurance policy or where the designated beneficiary is ones entitled to receive the retirement insurance benefits due
disqualified, the proceeds of the insurance will go to the estate of the deceased.
the deceased insured and will go to his legal heirs in accordance
with law. Where two women, innocently and in good faith, GSIS: Diaz is entitled to one-half, or 8/16, of the retirement
contracted marriage with the same man, the insured, and the insurance benefits while Berdin and their 7 children are entitled
latter did not designate any beneficiary who would receive the to the remaining one-half, or 8/16, each of them to receive an
proceeds of his life insurance, each family shall be entitled to one equal share of 1/16.
half of the insurance benefits.
Dissatisfied, Berdin and her children filed a petition for
FACTS: mandamus with preliminary injunction in the CFI of Surigao,
The late Jose Consuegra, at the time of his death, was employed against the respondents praying that they be declared the legal
as a shop foreman of DPWH in the province of Surigao del Norte. heirs and exclusive beneficiaries of the retirement insurance.
In his lifetime, he contracted 2 marriages, the first with
respondent Rosario Diaz, solemnized in the parish church of San CFI confirmed the action of GSIS.
Nicolas de Tolentino, Surigao, on July 15, 1937 bearing 2
children, Jose Consuegra, Jr. and Pedro Consuegra, but both Hence the present appeal by herein petitioners-appellants,
predeceased their father; and the second with herein petitioner Basilia Berdin and her children contending that because the
Basilia Berdin, which was contracted in good faith while the first deceased failed to designate the beneficiaries in his retirement
marriage was subsisting, on May 1, 1957 in the same parish and insurance, they being beneficiaries named in the life insurance
municipality, out of which marriage were born seven children, should automatically be considered the beneficiaries to receive
namely, Juliana, Pacita, Maria Lourdes, Jose, Rodrigo, Lenida and the retirement insurance benefits, to the exclusion of Diaz.
Luz *(Luis) all surnamed Consuegra.

When Consuegra died the proceeds of his life insurance were ISSUES:
paid by the GSIS to petitioner Basilia Berdin and her children (1) Is the contention tenable?
MERCANTILE LAW REVIEW || Insurance Law 15
Compilation of Digests (Except 37)

(2) To whom should this retirement insurance benefits be paid? beneficiary named in the life insurance policy, the
proceeds of the insurance will go to the estate of the
HELD: (1) NO. The contention of appellants is untenable. insured.

When Consuegra designated his beneficiaries in his life (b) Retirement insurance- primarily intended for the
insurance he could not have intended those beneficiaries benefit of the employee to provide for his old age, or
of his life insurance as also the beneficiaries of his incapacity, after rendering service in the government for
retirement insurance because the provisions on a required number of years. If the employee reaches the
retirement insurance under the GSIS came about only age of retirement, he gets the retirement benefits even to
when Com. Act 186 was amended by Rep. Act 660 on the exclusion of the beneficiary or beneficiaries named in
June 16, 1951. Hence, it cannot be said that because his application for retirement insurance. The beneficiary
appellants were designated beneficiaries in Consuegra’s of the retirement insurance can only claim the proceeds
life insurance they automatically became the of the retirement insurance if the employee dies before
beneficiaries also of his retirement insurance. The retirement.
provisions clearly indicate that there is need for the
employee to file an application for retirement insurance If the employee failed or overlooked to state the
benefits when he becomes a member of the GSIS, and he beneficiary of his retirement insurance, the retirement
should state in his application the beneficiary of his benefits will accrue to his estate and will be given to his
retirement insurance. Hence, the beneficiary named in legal heirs in accordance with law, as in the case of a life
the life insurance does not automatically become the insurance if no beneficiary is named in the insurance
beneficiary in the retirement insurance unless the same policy.
beneficiary in the life insurance is so designated in the
application for retirement insurance. (2) BOTH. The proceeds of the retirement insurance of the
late Jose Consuegra should be divided equally between
The GSIS offers two separate and distinct systems of Rosario Diaz and his second wife Basilia Berdin and his
benefits to its members paid out from two distinct and children by her. The fact that the second marriage was
separate funds: contracted in good faith, the only just and equitable solution
in this case would be to recognize the right of the second wife
(a) Life insurance- proceeds are paid to whoever is to her share of one-half in the property acquired by her and
named the beneficiary in the life insurance policy as her husband, and consider the other half as pertaining to the
provided for in the Insurance Act (Act 2427, as conjugal partnership of the first marriage.
amended), the beneficiary in a life insurance under the
GSIS may not necessarily be an heir of the insured. The The decision appealed from is affirmed.
insured in a life insurance may designate any person as
beneficiary unless disqualified to be so under the
provisions of the Civil Code. And in the absence of any
MERCANTILE LAW REVIEW || Insurance Law 16
Compilation of Digests (Except 37)

11. THE INSULAR LIFE ASSURANCE COMPANY, LTD. vs. Buenaventura Cristor Ebrado and directing the payment of the
CARPONIA T. EBRADO and PASCUALA VDA. DE EBRADO insurance proceeds to the estate of the deceased insured.
G.R NO. L-44059, OCT. 28, 1977, MARTIN, J.
ISSUE/S:
Under Article 2012 of the Civil Code, "any person who is forbidden
Can a common-law wife named as beneficiary in the life
from receiving any donation under Article 739 cannot be named
insurance policy of a legally married man claim the proceeds
beneficiary of a life insurance policy by the person who cannot
thereof in case of death of the latter?
make a donation to him. Common-law spouses are, definitely,
barred from receiving donations from each other.
HELD:
FACTS:

On 1968, Buenaventura Cristor Ebrado was issued by No.


The Life Assurance Co., Ltd., a policy on a whole-life for
P5,882.00 with a rider for Accidental Death. For the same Section 50 of the Insurance Act which provides that
amount, Buenaventura C. Ebrado designated Carponia T. Ebrado, "(t)he insurance shall be applied exclusively to the proper
as his wife as the revocable beneficiary in his policy. interest of the person in whose name it is made" cannot be
validly seized upon to hold that the policy includes the
On 1969, Buenaventura died when he was hit by a failing beneficiary. The word "interest" highly suggests that the
branch of a tree. The Insular Life Assurance Co., Ltd. liable to pay provision refers only to the "insured" and not to the beneficiary,
the coverage representing the face value of the policy. since a contract of insurance is personal in character. Otherwise,
the prohibitory laws against illicit relationships especially on
property and descent will be rendered nugatory, as the same
Carponia filed with the insurer a claim for the proceeds
could easily be circumvented by modes of insurance. Rather, the
of the Policy as the designated beneficiary therein, although she
general rules of civil law should be applied to resolve this void in
admits that she and the insured Buenaventura C. Ebrado were
the Insurance Law.
merely living as husband and wife without the benefit of
marriage.
When not otherwise specifically provided for by the
Insurance Law, the contract of life insurance is governed by the
Pascuala Vda. de Ebrado also filed her claim as the widow
general rules of the civil law regulating contracts. Under Article
of the deceased insured. She asserts that she is the one entitled
2012 of the Civil Code, "any person who is forbidden from
to the insurance proceeds, not the common-law wife, Carponia T.
receiving any donation under Article 739 cannot be named
Ebrado.
beneficiary of a life insurance policy by the person who cannot
make a donation to him. Common-law spouses are, definitely,
Trial court rendered judgment declaring that Carponia T.
barred from receiving donations from each other.
Ebrado is disqualified from becoming beneficiary of the insured
MERCANTILE LAW REVIEW || Insurance Law 17
Compilation of Digests (Except 37)

In essence, a life insurance policy is no different from a


civil donation insofar as the beneficiary is concerned. Both are
founded upon the same consideration: liberality. A beneficiary is
like a donee, because from the premiums of the policy which the
insured pays out of liberality, the beneficiary will receive the
proceeds or profits of said insurance.

There is every reason to hold that the bar in donations


between legitimate spouses and those between illegitimate ones
should be enforced in life insurance policies since the same are
based on similar consideration. If legitimate relationship is
circumscribed by legal disabilities, with more reason should an
illicit relationship be restricted by these disabilities.

The requisite proof of common-law relationship between


the insured and the beneficiary has been conveniently supplied
by the stipulations between the parties in the pre-trial
conference of the case. It was agreed upon and stipulated therein
that the deceased insured Buenaventura C. Ebrado was married
to Pascuala Ebrado with whom she has six legitimate children;
that during his lifetime, the deceased insured was living with his
common-law wife, Carponia Ebrado, with whom he has two
children. These stipulations are nothing less than judicial
admissions which, as a consequence, no longer require proof and
cannot be contradicted. 

Carponia T. Ebrado is hereby declared disqualified to be


the beneficiary of the late Buenaventura C. Ebrado in his life
insurance policy. As a consequence, the proceeds of the policy
are hereby held payable to the estate of the deceased insured.
MERCANTILE LAW REVIEW || Insurance Law 18
Compilation of Digests (Except 37)

INSURABLE INTEREST exist at the time the insurance takes effect and at the time the
loss occurs. The basis of such requirement of insurable interest
12. Sps. Nilo and Stella Cha, et. Al. vs Court of Appeals, et. al. in the property insured is based on sound public policy: to
GR NO.124520, August 18, 1997 prevent a person from taking out a insurance policy on property
upon which he has no insurable interest and collecting the
proceeds of said policy in case of loss of the property. In such a
FACTS: case, the contract of insurance is a mere wager which is void
Sps. Nilo Cha and Stella Uy Cha entered into a contract of lease under Sec. 25 of the Insurance Code.
with CKS Development Corporation. One of the stipulations of In the present case, it cannot be denied that CKS has no insurable
the contract of lease provides that the lessee cannot insure interest in the goods and merchandise inside the lease premises.
against fire the chattels, merchandise and effects stored in the Therefore, CKS cannot, under the Insurance Code, be validly a
lease premises without written consent and approval of the beneficiary of the fire insurance taken by the Spouses Cha over
lessor. If the lessee violates this stipulation then the policy is their merchandise.
deemed assigned and transferred to the lessor for its own
benefit. Despite such stipulation, the Sps. Cha insured against
loss by fire the merchandise inside the lease premises with the
United Insurance Co. without the written consent of CKS. On the
day the lease contract was to expire, fire broke out in the lease
premises. When CKS learned of the insurance, it wrote United a
demand letter asking that the proceeds of the insurance be paid
directly to them.
ISSUE:
Whether or not CKS is a valid beneficiary in the fire insurance
taken by Spouses Cha over their merchandise.
RULING:
NO.
Sec. 18. No contract or policy of insurance on property shall be
enforceable except for the benefit of some person having an
insurable interest in the property insured.
A non-life insurance policy such as the fire insurance policy
taken by Sps. Cha over their merchandise is primarily a contract
of indemnity. Insurable interest in the property insured must
MERCANTILE LAW REVIEW || Insurance Law 19
Compilation of Digests (Except 37)

13. GAISANO CAGAYAN, INC. vs. INSURANCE COMPANY OF Included in the items lost or destroyed in the fire were stocks of
NORTH AMERICA ready-made clothing materials sold and delivered by IMC and
G.R. No. 147839, June 8, 2006; AUSTRIA-MARTINEZ, J.: LSPI. 

Section 13 of our Insurance Code defines insurable interest as On February 4, 1992, respondent filed a complaint for damages
"every interest in property, whether real or personal, or any against petitioner. It alleges that IMC and LSPI filed with
relation thereto, or liability in respect thereof, of such nature that respondent their claims under their respective fire insurance
a contemplated peril might directly damnify the insured." policies with book debt endorsements; that as of February 25,
Parenthetically, under Section 14 of the same Code, an insurable 1991, the unpaid accounts of petitioner on the sale and delivery
interest in property may consist in: (a) an existing interest; (b) an of ready-made clothing materials with IMC was P2,119,205.00
inchoate interest founded on existing interest; or (c) an while with LSPI it was P535,613.00; that respondent paid the
expectancy, coupled with an existing interest in that out of which claims of IMC and LSPI and, by virtue thereof, respondent was
the expectancy arises. subrogated to their rights against petitioner; that respondent
Art. 2207. If the plaintiff's property has been insured, and made several demands for payment upon petitioner but these
he has received indemnity from the insurance company for the went unheeded.
injury or loss arising out of the wrong or breach of contract
complained of, the insurance company shall be subrogated to the In its Answer with Counter Claim dated July 4, 1995, petitioner
rights of the insured against the wrongdoer or the person who has contends that it could not be held liable because the property
violated the contract. covered by the insurance policies were destroyed due to
fortuities event or force majeure; that respondent's right of
FACTS: Intercapitol Marketing Corporation (IMC) is the maker of subrogation has no basis inasmuch as there was no breach of
Wrangler Blue Jeans. Levi Strauss (Phils.) Inc. (LSPI) is the local contract committed by it since the loss was due to fire which it
distributor of products bearing trademarks owned by Levi could not prevent or foresee; that IMC and LSPI never
Strauss & Co.. IMC and LSPI separately obtained from respondent communicated to it that they insured their properties; that it
fire insurance policies with book debt endorsements. The never consented to paying the claim of the insured.
insurance policies provide for coverage on "book debts in
connection with ready-made clothing materials which have been ISSUE: Whether the insurance in the instant case was one over
sold or delivered to various customers and dealers of the Insured credit.
anywhere in the Philippines." The policies defined book debts as
the "unpaid account still appearing in the Book of Account of the RULING: Yes. The insurable interest is the credit. The questioned
Insured 45 days after the time of the loss covered under this insurance policies provide coverage for "book debts in
Policy." connection with ready-made clothing materials which have been
sold or delivered to various customers and dealers of the Insured
Petitioner is a customer and dealer of the products of IMC and anywhere in the Philippines.” Thus, the insured against were the
LSPI. On February 25, 1991, the Gaisano Superstore Complex in accounts of IMC and LSPI with petitioner which remained unpaid
Cagayan de Oro City, owned by petitioner, was consumed by fire.
MERCANTILE LAW REVIEW || Insurance Law 20
Compilation of Digests (Except 37)

45 days after the loss through fire, and not the loss or the insurance company shall be subrogated to the rights of the
destruction of the goods delivered.  insured against the wrongdoer or the person who has violated
the contract.
IMC and LSPI did not lose complete interest over the goods. They
have an insurable interest until full payment of the value of the
delivered goods. In property insurance, one's interest is not
determined by concept of title, but whether insured has
substantial economic interest in the property.

Therefore, an insurable interest in property does not necessarily


imply a property interest in, or a lien upon, or possession of, the
subject matter of the insurance, and neither the title nor a
beneficial interest is requisite to the existence of such an interest,
it is sufficient that the insured is so situated with reference to the
property that he would be liable to loss should it be injured or
destroyed by the peril against which it is insured. Anyone has an
insurable interest in property who derives a benefit from its
existence or would suffer loss from its destruction.Indeed, a
vendor or seller retains an insurable interest in the property sold
so long as he has any interest therein, in other words, so long as
he would suffer by its destruction, as where he has a vendor's
lien. In this case, the insurable interest of IMC and LSPI pertain to
the unpaid accounts appearing in their Books of Account 45 days
after the time of the loss covered by the policies.

With respect to IMC, the respondent has adequately established


its claim. The subrogation receipt, by itself, is sufficient to
establish not only the relationship of respondent as insurer and
IMC as the insured, but also the amount paid to settle the
insurance claim. The right of subrogation accrues simply upon
payment by the insurance company of the insurance
claim. Respondent's action against petitioner is squarely
sanctioned by Article 2207 of the Civil Code which provides:
Art. 2207. If the plaintiff's property has been insured, and he has
received indemnity from the insurance company for the injury or
loss arising out of the wrong or breach of contract complained of,
MERCANTILE LAW REVIEW || Insurance Law 21
Compilation of Digests (Except 37)

14. GREAT PACIFIC LIFE ASSURANCE CORP. v. COURT OF was not physically healthy when he applied for an insurance
APPEALS and coverage.
MEDARDA V. LEUTERIO
G.R. No. 113899, October 13, 1999, J. QUISUMBING 1 The widow of the late Dr. Leuterio, respondent Medarda
V. Leuterio (Medarda), filed a complaint with RTC Misamis
FACTS Oriental against Grepalife for “Specific Performance with
A contract of group life insurance was executed between Damages.”
petitioner Great Pacific Life Assurance Corporation (Grepalife) During the trial, Dr. Hernando Mejia, who issued the
and Development Bank of the Philippines (DBP). Grepalife death certificate, was called to testify. Dr. Mejia's findings, based
agreed to insure the lives of eligible housing loan mortgagors of partly from the information given by the respondent widow,
DBP. stated that Dr. Leuterio complained of headaches presumably
Dr. Wilfredo Leuterio, a physician and a housing debtor due to high blood pressure. The inference was not conclusive
of DBP applied for membership in the group life insurance plan. because Dr. Leuterio was not autopsied, hence, other causes
In an application form, Dr. Leuterio answered questions were not ruled out. Aside from the death certiticate, the
concerning his health condition as follows: statement of Medarda was presented. Her testimony was the
medicines that Dr. Letuerio took which she was not even sure if it
7. Have you ever had, or consulted, a physician for a heart were for hypertension. Grepalife did not further produce any
condition, high blood pressure, cancer, diabetes, lung; witness who could attest to Dr. Leuterio’s medical history.
kidney or stomach disorder or any other physical The RTC rendered a decision in favor of Medarda. Upon
impairment? appeal, the CA sustained the RTC decision. Hence, this petition.

Answer: No. If so give details _____________.


ISSUES
8. Are you now, to the best of your knowledge, in good 1. Whether or not Dr. Leuterio’s widow, Medarda, has insurable
health? interest for her to claim benefits from the group life
insurance contract of the DBP against Grepalife.
Answer: [x] Yes [ ] NO. 2. Whether or not Dr. Leuterio concealed a material fact, i.e. his
hypertension, at the time of his application for insurance
Thereafter, Grepalife insured Dr. Leuterio to the extent of which would vitiate the insurance contract.
his DBP mortgage indebtedness amounting to Php 86,200.00. 3. Whether or not there is truth to Grepalife’s allegation that
Dr. Leuterio died. As certified in his death certificate, the there was no evidence as to the amount of Dr. Leuterio’s
cause of death is due to to "massive cerebral hemorrhage." outstanding indebtedness to DBP at the time of the
Consequently, DBP submitted a death claim to Grepalife. mortgagor’s death.
Grepalife denied the claim alleging concealment, i.e. Dr. Leuterio

1
Same with case numbers 17 and 53
MERCANTILE LAW REVIEW || Insurance Law 22
Compilation of Digests (Except 37)

RULING
1. YES. As held in Gonzales La O v. Yek Tong Lin Fire & Marine Sec. 8 of the Insurance Code provides:
Ins. Co.: Unless the policy provides, where a mortgagor of property
effects insurance in his own name providing that the loss
Insured, being the person with whom the contract was made, shall be payable to the mortgagee, or assigns a policy of
is primarily the proper person to bring suit thereon. * * insurance to a mortgagee, the insurance is deemed to be
* Subject to some exceptions, insured may thus sue, although upon the interest of the mortgagor, who does not cease to be
the policy is taken wholly or in part for the benefit of another a party to the original contract, and any act of his, prior to the
person named or unnamed, and although it is expressly made loss, which would otherwise avoid the insurance, will have
payable to another as his interest may appear or otherwise. * the same effect, although the property is in the hands of the
* * Although a policy issued to a mortgagor is taken out for mortgagee, but any act which, under the contract of
the benefit of the mortgagee and is made payable to him, yet insurance, is to be performed by the mortgagor, may be
the mortgagor may sue thereon in his own name, especially performed by the mortgagee therein named, with the same
where the mortgagee's interest is less than the full amount effect as if it had been performed by the mortgagor.
recoverable under the policy, * * *.
2. NO. Concealment exists where the assured had knowledge of
Insured may be regarded as the real party in interest, a fact material to the risk, and honesty, good faith, and fair
although he has assigned the policy for the purpose dealing requires that he should communicate it to the
of collection, or has assigned as collateral security assured, but he designedly and intentionally withholds the
any judgment he may obtain. (Emphasis supplied) same.
And since a policy of insurance upon life or Fraudulent intent on the part of the insured must be
health may pass by transfer, will or succession to any established to entitle the insurer to rescind the contract.
person, whether he has an insurable interest or not, Misrepresentation as a defense of the insurer to avoid
and such person may recover it whatever the insured liability is an affirmative defense and the duty to establish
might have recovered, the widow of the decedent Dr. such defense by satisfactory and convincing evidence rests
Leuterio may file the suit against the insurer, Grepalife. upon the insurer. 
(Emphasis supplied) Petitioner merely relied on the testimony of the
Moreover, the insured private respondent did not attending physician, Dr. Hernando Mejia, as supported by the
cede to the mortgagee all his rights or interests in the information given by the widow of the decedent. Grepalife
insurance, the policy stating that: "In the event of the asserts that Dr. Mejia's technical diagnosis of the cause of
debtor's death before his indebtedness with the Creditor death of Dr. Leuterio was a duly documented hospital record,
[DBP] shall have been fully paid, an amount to pay the and that the widow's declaration that her husband had
outstanding indebtedness shall first be paid to the "possible hypertension several years ago" should not be
creditor and the balance of sum assured, if there is any, considered as hearsay, but as part of res gestae.
shall then be paid to the beneficiary/ies designated by On the contrary the medical findings were not conclusive
the debtor." because Dr. Mejia did not conduct an autopsy on the body of
MERCANTILE LAW REVIEW || Insurance Law 23
Compilation of Digests (Except 37)

the decedent. As the attending physician, Dr. Mejia stated


that he had no knowledge of Dr. Leuterio's any previous The policy states that upon receipt of due proof of
hospital confinement. Dr. Leuterio's death certificate stated the Debtor's death during the terms of this insurance, a
that hypertension was only "the possible cause of death." The death benefit in the amount of P86,200.00 shall be paid.
private respondent's statement, as to the medical history of
her husband, was due to her unreliable recollection of events. In the event of the debtor's death before his
Hence, the statement of the physician was properly indebtedness with the creditor shall have been
considered by the trial court as hearsay. fully paid, an amount to pay the outstanding
indebtedness shall first be paid to the Creditor
The insured, Dr. Leuterio, had answered in his insurance and the balance of the Sum Assured, if there is
application that he was in good health and that he had not any shall then be paid to the beneficiary/ies
consulted a doctor or any of the enumerated ailments, including designated by the debtor." (Emphasis omitted)
hypertension; when he died the attending physician had certified
in the death certificate that the former died of cerebral However, the SC notes that the Court of Appeals' decision
hemorrhage, probably secondary to hypertension. From this was promulgated on May 17, 1993. In private respondent's
report, the appellant insurance company refused to pay the memorandum, she states that DBP foreclosed in 1995 their
insurance claim. Appellant alleged that the insured had residential lot, in satisfaction of mortgagor's outstanding loan.
concealed the fact that he had hypertension. Considering this supervening event, the insurance proceeds shall
inure to the benefit of the heirs of the deceased person or his
Contrary to appellant's allegations, there was no sufficient beneficiaries. Equity dictates that DBP should not unjustly enrich
proof that the insured had suffered from hypertension. Aside itself at the expense of another (Nemo cum alterius detrimenio
from the statement of the insured's widow who was not even protest). Hence, it cannot collect the insurance proceeds, after it
sure if the medicines taken by Dr. Leuterio were for already foreclosed on the mortgage. The proceeds now rightly
hypertension, the appellant had not proven nor produced any belong to Dr. Leuterio's heirs represented by his widow, herein
witness who could attest to Dr. Leuterio's medical history. private respondent Medarda Leuterio.

Hence, in the case at bar, the petitioner failed to clearly and


satisfactorily establish its defense, and is therefore liable to pay
the proceeds of the insurance.

3. NO. A life insurance policy is a valued policy. Unless the


interest of a person insured is susceptible of exact pecuniary
measurement, the measure of indemnity under a policy of
insurance upon life or health is the sum fixed in the policy.
The mortgagor paid the premium according to the coverage
of his insurance, which states that:
MERCANTILE LAW REVIEW || Insurance Law 24
Compilation of Digests (Except 37)

15. ONG LIM SING, JR. vs. FEB LEASING AND FINANCE On November 22, 2002, the trial court ruled in favor of JVL and
CORPORATION Lim and stressed the contradictory terms found in the lease
G.R. No. 168115 - June 8, 2007 agreement. The trial court stated, among others, that if JVL and
Lim (then defendants) were to be regarded as only a lessee,
Section 17 of the Insurance Code provides that the measure of an logically the lessor who asserts ownership will be the one
insurable interest in property is the extent to which the insured directly benefited or injured and therefore the lessee is not
might be damnified by loss or injury thereof. supposed to be the assured as he has no insurable interest.

FACTS: On December 27, 2002, FEB filed its Notice of Appeal.


Accordingly, on January 17, 2003, the court issued an Order
On March 9, 1995, FEB Leasing and Finance Corporation entered elevating the entire records of the case to the Court of Appeals.
into a lease of equipment and motor vehicles with JVL Food On March 15, 2005, the Court of Appeals issued its Decision
Products. On the same date, Vicente Ong Lim Sing, Jr. executed an declaring the transaction between the parties as a financial lease
Individual Guaranty Agreement with FEB to guarantee the agreement. The said decision reversed and set aside the trial
prompt and faithful performance of the terms and conditions of court’s decision dated November 22, 2002. Hence, Lim filed the
the aforesaid lease agreement. Corresponding Lease Schedules present Petition for Review on Certiorari.
with Delivery and Acceptance Certificates over the equipment
and motor vehicles formed part of the agreement. Under the ISSUE:
contract, JVL was obliged to pay FEB an aggregate gross monthly Whether or not petitioner has an insurable interest in the
rental of One Hundred Seventy Thousand Four Hundred Ninety- equipment and motor vehicles leased.
Four Pesos (P170,494.00).
RULING:
JVL defaulted in the payment of the monthly rentals. As of July
31, 2000, the amount in arrears, including the penalty charges Yes. The stipulation in Section 14 of the leased contract, that the
and insurance premiums, amounted to Three Million Four equipment shall be insured at the cost and expense of the lessee
Hundred Fourteen Thousand Four Hundred Sixty-Eight and against loss, damage, or destruction from fire, theft, accident, or
75/100 Pesos (P3,414,468.75). On August 23, 2000, FEB sent a other insurable risk for the full term of the lease, is a binding and
letter to JVL demanding payment of the said amount. However, valid stipulation. Petitioner, as a lessee, has an insurable interest
JVL failed to pay. in the equipment and motor vehicles leased. Section 17 of the
Insurance Code provides that the measure of an insurable
On December 6, 2000, FEB filed a Complaint with the Regional interest in property is the extent to which the insured might be
Trial Court of Manila for sum of money, damages, and replevin damnified by loss or injury thereof. It cannot be denied that JVL
against JVL, Lim, and John Doe. will be directly damnified in case of loss, damage, or destruction
In an Amended Answer, JVL and Lim admitted the existence of of any of the properties leased.
the lease agreement but asserted that it is in reality a sale of
equipment on instalment basis, with FEB acting as the financier.
MERCANTILE LAW REVIEW || Insurance Law 25
Compilation of Digests (Except 37)

CONCEALMENT - Has any life insurance company ever refused your application
for insurance or for reinstatement of a lapsed policy or offered
16. NG GAN ZEE vs. ASIAN CRUSADER LIFE ASSURANCE CORP. you a policy different from that applied for? If, so, name company
G.R. No. L-30685 May 30, 1983, ESCOLIN, J. and date.

"Concealment exists where the assured had knowledge of a fact The lower court ruled against the company on lack of evidence.
material to the risk, and honesty, good faith, and fair dealing Appellant further maintains that when the insured was examined
requires that he should communicate it to the assurer, but he in connection with his application for life insurance, he gave the
designedly and intentionally withholds the same." appellant's medical examiner false and misleading information
as to his ailment and previous operation. The company
FACTS: contended that he was operated on for peptic ulcer 2 years
Kwong Nam applied for a 20-year endowment insurance on his before the policy was applied for and that he never disclosed
life for the sum of P20,000.00, with his wife, appellee Ng Gan Zee such an operation.
as beneficiary. On the same date, Asian Crusader, upon receipt of
the required premium from the insured, approved the ISSUE:
application and issued the corresponding policy. Kwong Nam WON Asian Crusader was deceived into entering the contract or
died of cancer of the liver with metastasis. All premiums had in accepting the risk at the rate of premium agreed upon because
been paid at the time of his death. of insured's representation?

Ng Gan Zee presented a claim for payment of the face value of the HELD:
policy. On the same date, she submitted the required proof of NO. Petition dismissed. Section 27 of the Insurance Law: Sec. 27.
death of the insured. Appellant denied the claim on the ground Such party a contract of insurance must communicate to the
that the answers given by the insured to the questions in his other, in good faith, all facts within his knowledge which are
application for life insurance were untrue. material to the contract, and which the other has not the means
of ascertaining, and as to which he makes no warranty.
Appellee brought the matter to the attention of the Insurance "Concealment exists where the assured had knowledge of a fact
Commissioner. The latter, after conducting an investigation, material to the risk, and honesty, good faith, and fair dealing
wrote the appellant that he had found no material concealment requires that he should communicate it to the assurer, but he
on the part of the insured and that, therefore, appellee should be designedly and intentionally withholds the same." It has also
paid the full face value of the policy. The company refused to been held "that the concealment must, in the absence of
settle its obligation. inquiries, be not only material, but fraudulent, or the fact must
have been intentionally withheld." Fraudulent intent on the part
Appellant alleged that the insured was guilty of of the insured must be established to entitle the insurer to
misrepresentation when he answered "No" to the following rescind the contract. And as correctly observed by the lower
question appearing in the application for life insurance court, "misrepresentation as a defense of the insurer to avoid
liability is an 'affirmative' defense. The duty to establish such a
MERCANTILE LAW REVIEW || Insurance Law 26
Compilation of Digests (Except 37)

defense by satisfactory and convincing evidence rests upon the


defendant. The evidence before the Court does not clearly and
satisfactorily establish that defense." It bears emphasis that
Kwong Nam had informed the appellant's medical examiner of
the tumor. His statement that said tumor was "associated with
ulcer of the stomach" should be construed as an expression made
in good faith of his belief as to the nature of his ailment and
operation. While the information communicated was imperfect,
the same was sufficient to have induced appellant to make
further inquiries about the ailment and operation of the insured.

Section 32 of Insurance Law: Section 32. The right to information


of material facts maybe waived either by the terms of insurance
or by neglect to make inquiries as to such facts where they are
distinctly implied in other facts of which information is
communicated. Where a question appears to be not answered at
all or to be imperfectly answered, and the insurers issue a policy
without any further inquiry, they waive the imperfection of the
answer and render the omission to answer more fully
immaterial. The company or its medical examiner did not make
any further inquiries on such matters from the hospital before
acting on the application for insurance. The fact of the matter is
that the defendant was too eager to accept the application and
receive the insured's premium. It would be inequitable now to
allow the defendant to avoid liability under the circumstances.
MERCANTILE LAW REVIEW || Insurance Law 27
Compilation of Digests (Except 37)

17. GREAT PACIFIC LIFE ASS. CORP. vs. CA AND MEDARDA based from the findings of attending physician, Dr.
LEUTERIO Mejia. Allegedly, such non-disclosure constituted concealment
G.R. No. 113899, Oct. 13, 1999, Quisimbing, J.2 that justified the denial of the claim.

Concealment exists where the assured had knowledge of a fact ISSUE: Whether Dr. Leuterio concealed that he had hypertension,
material to the risk, and honesty, good faith, and fair dealing which would vitiate the insurance contract.
requires that he should communicate it to the assured, but he
designedly and intentionally withholds the same. RULING: NO. Petitioner contends that Dr. Leuterio failed to
disclose that he had hypertension, which might have caused his
FACTS: A contract of group life insurance was executed between death. Concealment exists where the assured had knowledge of a
Great Pacific Life Assurance Corporation (Grepalife) and fact material to the risk, and honesty, good faith, and fair dealing
Development Bank of the Philippines (DBP). Grepalife agreed to requires that he should communicate it to the assured, but he
insure the lives of eligible housing loan mortgagors of DBP. Dr. designedly and intentionally withholds the same. Petitioner
Wilfredo Leuterio, a physician and a housing debtor of DBP merely relied on the testimony of the attending physician, Dr.
applied for membership in the group life insurance plan. In an Hernando Mejia, as supported by the information given by the
application form, Dr. Leuterio answered questions concerning widow of the decedent. 
his health condition as follows:
Grepalife asserts that Dr. Mejia’s technical diagnosis of the cause
7. Have you ever had, or consulted, a physician for a heart of death of Dr. Leuterio was a duly documented hospital record,
condition, high blood pressure, cancer, diabetes, lung, and that the widow’s declaration that her husband had possible
kidney or stomach disorder or any other physical hypertension several years ago should not be considered as
impairment? hearsay, but as part of res gestae. On the contrary the medical
findings were not conclusive because Dr. Mejia did not conduct
Answer: No. If so give details ___________. an autopsy on the body of the decedent. As the attending
physician, Dr. Mejia stated that he had no knowledge of Dr.
8. Are you now, to the best of your knowledge, in good health? Leuterios any previous hospital confinement, Dr. Leuterio’s
Answer: [ x ] Yes [ ] No death certificate stated that hypertension was only the possible
cause of death. The private respondent’s statement, as to the
Dr. Leuterio died due to massive cerebral hemorrhage. medical history of her husband, was due to her unreliable
Consequently, DBP submitted a death claim to recollection of events. Hence, the statement of the physician was
Grepalife. Grepalife denied the claim alleging that Dr. Leuterio properly considered by the trial court as hearsay.
was not physically healthy when he applied for an
insurance. Grepalife insisted that Dr. Leuterio did not disclose he
had been suffering from hypertension, which caused his death

2
Same with case numbers 14 and 53
MERCANTILE LAW REVIEW || Insurance Law 28
Compilation of Digests (Except 37)

18. NEW LIFE ENTERPRISES and JULIAN SY vs. HON. COURT On February 8, 1982, Equitable Insurance
OF APPEALS, Corporation issued Fire Insurance Policy No. 39328 in the
G.R. No. 94071 March 31, 1992, REGALADO, J: amount of P200,000.00.

The terms of the contract are clear and unambiguous. Thus when the building occupied by the New Life Enterprises


The insured is specifically required to disclose to the insurer any was gutted by fire at about 2:00 o'clock in
other insurance and its particulars which he may have effected on the morning of October 19, 1982, the stocks in the
the same subject matter. The knowledge of such insurance trade inside said building were insured against fire in
by the insurer's agents, even assuming the acquisition thereof by the total amount of P1,550,000.00. 
the former, is not the "notice" that would estop the insurers from
denying the claim. Besides, the so-called theory of imputed After the fire, Julian Sy went to the agent of
knowledge, that is, knowledge of the agent is Reliance Insurance whom he asked to accompany him to the
knowledge of the principal, aside from being office of the company so that he can file his claim.  He further
of dubious applicability here has likewise been roundly testified that the three insurance companies are sister
refuted by respondent court whose factual findings we find companies, and as a matter of fact when he was following-
acceptable. up his claim with Equitable Insurance, the
Claims Manager told him to go first to Reliance
Facts: Insurance and if said company agrees to pay, they would
Julian Sy and Jose Sy Bang have formed a business partnership also pay. The same treatment was given him
in the City of Lucena. Under the business name of New Life by the other insurance companies. Ultimately, the
Enterprises, the partnership engaged in the sale of construction three insurance companies denied plaintiffs' claim for payment.
materials at its place of business, a two storey building situated
at Iyam, Lucena City. The facts show that Julian Sy insured the Issue:
stocks in trade of New Life Enterprises with Western Guaranty WON there was a violation of the insurance contract by obtaining
Corporation, Reliance Surety and Insurance. Co., Inc., and three insurance contracts, thus constituting concealment on the
Equitable Insurance Corporation. part of Sy?

On May 15, 1981, Western Guaranty Corporation Ruling:


issued Fire Insurance Policy No. 37201 in the amount The court finds that there was a violation of the insurance
of P350,000.00. This policy was renewed on May, 13, 1982. On contract by Sy.
July 30,1981, Reliance Surety and Insurance Co., Inc. issued Fire
Insurance Policy No. 69135 in the amount of P300,000.00 The insured shall give notice to the Company of any insurance or
(Renewed under Renewal Certificate No. 41997) An additional insurances already effected, or which may subsequently be
insurance was issued by the same company on effected, covering any of the property or properties consisting of
November 12, 1981 under Fire Insurance Policy No. 71547 in stocks in trade, goods in process and/or inventories only hereby
the amount of P700,000.00. insured, and unless such notice be given and the particulars of
MERCANTILE LAW REVIEW || Insurance Law 29
Compilation of Digests (Except 37)

such insurance or insurances be stated therein or endorsed on Since Sy, was a businessman, it was incumbent upon him to read
this policy pursuant to Section 50 of the Insurance Code, by or on the contracts.
behalf of the Company before the occurrence of any loss or
damage, all benefits under this policy shall be deemed forfeited,
provided however, that this condition shall not apply when the
total insurance or insurances in force at the time of loss or
damage not more than P200,000.00.

Sy never disclosed co-insurance in the contracts he entered into


with the three corporations. The insured is specifically required
to disclose the insurance that he had contracted with other
companies. Sy also contended that the insurance agents knew of
the co-insurance. However, the theory of imputed knowledge,
that the knowledge of the agent is presumed to be known by the
principal, is not enough.

When the words of the document are readily understandable by


an ordinary reader, there is no need for construction anymore.
The terms of the contract are clear and unambiguous.
The insured is specifically required to disclose to the insurer any
other insurance and its particulars which he may have effected
on the same subject matter. The knowledge of such insurance
by the insurer's agents, even assuming the acquisition thereof by
the former, is not the "notice" that would estop the insurers from
denying the claim. Besides, the so-called theory of imputed
knowledge, that is, knowledge of the agent is
knowledge of the principal, aside from being
of dubious applicability here has likewise been roundly
refuted by respondent court whose factual findings we find
acceptable.

The conformity of the insured to the terms of the policy is


implied with his failure to disagree with the terms of the
contract.
MERCANTILE LAW REVIEW || Insurance Law 30
Compilation of Digests (Except 37)

19. Ma. Lourdes S. Florendo vs. Philam Plans, G.R. No. diabetes, the assumption is that he has never been treated for the
186983, Feb. 22, 2012, Abad, J. said illnesses in the last five years preceding his application.
Since Philam Plans waived medical examination for Florendo, it
It may be true that insured persons may accept policies without had to rely largely on his stating the truth regarding his health in
reading them, and that this is not negligence per se. But, this is not his application. It is clear from these representations that there
without any exception. was concealment.
Facts:
The insurance plan contains an incontestability clause, which
Philam Plans, Inc. (Philam Plans) issued a comprehensive precludes the insurer from disowning liability under the policy it
pension plan with life insurance coverage, containing a one-year issued on the ground of concealment or misrepresentation
incontestability period to Manuel Florendo. Eleven months later regarding the health of the insured after a year of its issuance.
after the issuance Florendo died of blood poisoning. Lourdes Since Florendo died on the eleventh month following the
filed a claim for the payment of the benefits under her husband’s issuance of his plan, the one-year incontestability period has not
plan but Philam Plans declined her claim on the ground of yet set in. Consequently, Philam Plans was not barred from
concealment. Lourdes points out that, seeing the unfilled spaces questioning Lourdes entitlement to the benefits of her husband’s
in Manuel’s pension plan application relating to his medical pension plan.
history, Philam Plans should have returned it to him for
completion. Since Philam Plans chose to approve the application
just as it was, it cannot cry concealment on Manuel’s part. She
claims that any defect or insufficiency in the information
provided by his pension plan application should be deemed
waived after the same has been approved, the policy has been
issued, and the premiums have been collected.

Issue:

Whether Florendo is guilty of concealing his illness when he kept


blank and did not answer questions in his pension plan
application and whether Philam Plan’s approval of Florendo’s
pension plan application and acceptance of his premium
payments precluded it from denying Lourde’s claim.

Held:

Yes. In signing the application without filling in the details


regarding his continuing treatments for heart condition and
MERCANTILE LAW REVIEW || Insurance Law 31
Compilation of Digests (Except 37)

20. GREAT PACIFIC LIFE ASSURANCE COMPANY vs. The non-acceptance of the insurance plan by Pacific Life was
HONORABLE COURT OF APPEALS allegedly not communicated by petitioner Mondragon to private
G.R. No. L-31845 April 30, 1979, De Castro, J. respondent Ngo Hing. Instead, Mondragon wrote back Pacific
Life again strongly recommending the approval of the 20-year
LAPULAPU D. MONDRAGON vs. HON. COURT OF APPEALS endowment insurance plan to children.
and NGO HING
Helen Go died of influenza with complication of
The contract of insurance is one of perfect good faith uberrima bronchopneumonia. Thereupon, private respondent sought the
fides meaning good faith, absolute and perfect candor or openness payment of the proceeds of the insurance, but having failed in his
and honesty; the absence of any concealment or demotion, effort, he filed the action for the recovery of the same before the
however slight, not for the alone but equally so for the insurer. CFI of Cebu, which rendered the adverse decision.

FACTS: Private respondent Ngo Hing filed an application with ISSUE:


the Great Pacific Life Assurance Company (Pacific Life) for a 20- WON Ngo Hing concealed the state of health and physical
year endowment policy on the life of his one-year old daughter condition of Helen Go, which rendered void the policy.
Helen Go. Said respondent supplied the essential data which
petitioner Lapulapu D. Mondragon, Branch Manager of the HELD:
Pacific Life in Cebu City wrote on the corresponding form in his YES. This Court is of the firm belief that private respondent had
own handwriting. Mondragon finally type-wrote the data on the deliberately concealed the state of health and physical condition
application form which was signed by private respondent Ngo of his daughter Helen Go. Where private respondent supplied the
Hing. The latter paid the annual premium the sum of P1,077.75 required essential data for the insurance application form, he
going over to the Company, but he retained the amount of was fully aware that his one-year old daughter is typically a
P1,317.00 as his commission for being a duly authorized agent of mongoloid child. Such a congenital physical defect could never be
Pacific Life. Upon the payment of the insurance premium, the ensconced nor disguised. Nonetheless, private respondent, in
binding deposit receipt was issued to private respondent Ngo apparent bad faith, withheld the fact material to the risk to be
Hing. Likewise, petitioner Mondragon handwrote at the bottom assumed by the insurance company. As an insurance agent of
of the back page of the application form his strong Pacific Life, he ought to know, as he surely must have known. His
recommendation for the approval of the insurance application. duty and responsibility to such a material fact. Had he stated said
Mondragon received a letter from Pacific Life disapproving the significant fact in the insurance application form, Pacific Life
insurance application. The letter stated that the said life would have verified the same and would have had no choice but
insurance application for 20-year endowment plan is not to disapprove the application outright.
available for minors below seven years old, but Pacific Life can
consider the same under the Juvenile Triple Action Plan, and The contract of insurance is one of perfect good faith uberrima
advised that if the offer is acceptable, the Juvenile Non-Medical fides meaning good faith, absolute and perfect candor or
Declaration be sent to the company. openness and honesty; the absence of any concealment or
demotion, however slight [Black's Law Dictionary, 2nd Edition],
MERCANTILE LAW REVIEW || Insurance Law 32
Compilation of Digests (Except 37)

not for the alone but equally so for the insurer (Field man's
Insurance Co., Inc. vs. Vda de Songco, 25 SCRA 70). Concealment
is a neglect to communicate that which a party knows and Ought
to communicate (Section 25, Act No. 2427). Whether intentional
or unintentional the concealment entitles the insurer to rescind
the contract of insurance (Section 26, Id.: Yu Pang Cheng vs.
Court of Appeals, et al, 105 Phil 930; Satumino vs. Philippine
American Life Insurance Company, 7 SCRA 316). Private
respondent appears guilty thereof.

We are thus constrained to hold that no insurance contract was


perfected between the parties with the noncompliance of the
conditions provided in the binding receipt, and concealment, as
legally defined, having been committed by herein private
respondent.
MERCANTILE LAW REVIEW || Insurance Law 33
Compilation of Digests (Except 37)

21. SATURNINO VS. PHIL. AMERICAN LIFE show actual fraud on the part of the insured.
G.R. No. L-16163, February 28, 1963, Makalintal, J.
In this jurisdiction, concealment, whether intentional or
Concealment, whether intentional or unintentional entitled the unintentional entitled the insurer to rescind the contract of
insurer to rescind the contract of insurance, concealment being insurance, concealment being defined as “negligence to
defined as “negligence to communicate that which a party knows communicate that which a party knows and ought to
and ought to communicate.” The basis of the rule vitiating the communicate.” The basis of the rule vitiating the contract in
contract in cases of concealment is that it misleads or deceives the cases of concealment is that it misleads or deceives the insurer
insurer into accepting the risk, or accepting it at a rate of premium into accepting the risk, or accepting it at a rate of premium
agreed upon. agreed upon. The insurer, relying upon the belief that the
insured will disclose every material fact within his actual or
FACTS: Prior to the application for insurance policy, Saturnino presumed knowledge, is misled into a belief that the
was operated on for cancer involving complete removal of the circumstances withheld does not exist, and he is thereby induced
right breast, including the pectoral muscles and the glands. to estimate the risk upon a false basis that it does not exist.
Notwithstanding the fact of her operation, Saturnino did not
make a disclosure thereof in her application for insurance. She The information given by her in the application for insurance
stated therein that she did not have, nor had she ever had, among was false, namely, that she never had cancer or tumors or
others listed in the application, cancer or other tumors; that she consulted any physician or undergone any operation within the
had not consulted any physician, undergone any operation or preceding period of 5 years. The question to determine is: Are
suffered any injury within the preceding 5 years. She also stated the facts then falsely represented material? The Insurance Law
that she had never been treated for, nor did she ever have any provides that “materiality is to be determined not by the event,
illness or disease peculiar to her sex, particularly of the breast, but solely by the probable and reasonable influence of the facts
ovaries, uterus and menstrual disorders. The application also upon the party to whom the communication is due, in forming
recited that the declarations of Saturnino constituted a further his estimate of the proposed contract, or making his inquiries.
basis for the issuance of the policy.
PhilAm Life contends that the facts subject of the representation
ISSUE: Whether the insured fraudulently concealed material were not material in view of the non-medical nature of the
facts. insurance applied for, which does away with the usual
requirement of medical examination before the policy is issued.
RULING: YES. PhilAm Life contend that there was no fraudulent We hold that if anything, the waiver of medical examination
concealment of the truth as the insured herself did not know, renders even more material the information required of the
since her doctor never told her, that the disease for which she applicant concerning previous condition of health and diseases
had been operated on was cancer. Such contention should fail, suffered, for such information necessarily constitutes an
concealment of the fact of the operation itself was fraudulent, as important factor which the insurer takes into consideration in
there could not have been any mistake about it, no matter what deciding whether to issue the policy or not.
the ailment was. In order to avoid a policy, it is not necessary to
MERCANTILE LAW REVIEW || Insurance Law 34
Compilation of Digests (Except 37)

WARRANTIES nevertheless issued the policies in question subject to such


warranty, and received the corresponding premiums.
22. QUA CHEE GAN v. LAW UNION AND ROCK INSURANCE CO.,
LTD., represented by its agent, WARNER, BARNES AND CO., The insurance company was aware, even before the policies
LTD. were issued, that in the premises insured there were only two
G.R. No. L-4611, 17 December 1955, REYES, J. B. L., J fire hydrants installed by Qua Chee Gan and two others nearby,
owned by the municipality of Tabaco, contrary to the
Qua Chee Gan, a merchant, owned 4 warehouses in Albay requirements of the warranty in question. The inequitableness of
which were used for storage of copra and hemp. The the conduct observed by the insurance company in this case is
warehouses, together with its contents, were insured with Law heightened by the fact that after the insured had incurred the
Union and Rock Insurance Co., Ltd. The insurance policy states expense of installing the two hydrants, the company collected the
that Qua Chee Gan should install 11 hydrants in the warehouses’ premiums and issued him a policy so worded that it gave the
premises. Qua Chee Gan installed only two, but Law Union insured a discount much smaller than that he was normaly
nevertheless went on with the insurance policy and collected entitledto. According to the "Scale of Allowances," a policy
premiums from Qua Chee Gan. The insurance contract also subject to a warranty of the existence of one fire hydrant for
provides that “oil” should not be stored within the premises of every 150 feet of external wall entitled the insured to a discount
the warehouses. of 7 1/2 per cent of the premium; while the existence of
In 1940, three of the warehouses were destroyed by fire. "hydrants, in compund" (regardless of number) reduced the
Qua Chee Gan demanded insurance from Law Union but the allowance on the premium to a mere 2 1/2 per cent. This
latter refused as it alleged that after investigation from their part, schedule was logical, since a greater number of hydrants and fire
they found out that Qua Chee Gan caused the fire. Law Union in fighting appliances reduced the risk of loss. But the appellant
fact sued Qua Chee Gan for Arson but was acquitted. This time, company, in the particular case now before us, so worded the
Law Union averred that the insurance contract is void because policies that while exacting the greater number of fire hydrants
Qua Chee Gan failed to install 11 hydrants; and that gasoline was and appliances, it kept the premium discount at the minimum of
found in one of the warehouses. 2 1/2 per cent, thereby giving the insurance company a double
benefit. No reason is shown why appellant's premises, that had
ISSUE: Was there a violation of warranties of the insurance been insured with appellant for several years past, suddenly
policy? should be regarded in 1939 as so hazardous as to be accorded a
treatment beyond the limits of appellant's own scale of
RULING: NO. allowances. Such abnormal treatment of the insured strongly
points at an abuse of the insurance company's selection of the
The appellant is barred by waiver (or rather estoppel) to claim words and terms of the contract, over which it had absolute
violation of the so-called fire hydrants warranty, for the reason control.
that knowing fully all that the number of hydrants demanded
therein never existed from the very beginning, the appellant It is usually held that where the insurer, at the time of the
issuance of a policy of insurance, has knowledge of
MERCANTILE LAW REVIEW || Insurance Law 35
Compilation of Digests (Except 37)

existing facts which, if insisted on, would invalidate the Insurance is, in its nature, complex and difficult for the
contract from its very inception, such knowledge layman to understand. Policies are prepared by experts
constitutes a waiver of conditions in the contract who know and can anticipate the hearing and possible
inconsistent with the facts, and the insurer is stopped complications of every contingency. So long as insurance
thereafter from asserting the breach of such conditions. companies insist upon the use of ambiguous, intricate
The law is charitable enough to assume, in the absence of and technical provisions, which conceal rather than
any showing to the contrary, that an insurance company frankly disclose, their own intentions, the courts must, in
intends to executed a valid contract in return for the fairness to those who purchase insurance, construe every
premium received; and when the policy contains a ambiguity in favor of the insured. (Algoe vs. Pacific Mut.
condition which renders it voidable at its inception, and L. Ins. Co., 91 Wash. 324, LRA 1917A, 1237.)
this result is known to the insurer, it will be presumed to
have intended to waive the conditions and to execute a An insurer should not be allowed, by the use of obscure phrases
binding contract, rather than to have deceived the and exceptions, to defeat the very purpose for which the policy
insured into thinking he is insured when in fact he is not, was procured (Moore vs. Aetna Life Insurance Co., LRA 1915D,
and to have taken his money without consideration. (29 264).
Am. Jur., Insurance, section 807, at pp. 611-612.)
We see no reason why the prohibition of keeping gasoline in the
Also, appellant insurance company avers, that the insured premises could not be expressed clearly and unmistakably, in the
violated the "Hemp Warranty" provisions of the Policy against language and terms that the general public can readily
the storage of gasoline, since appellee admitted that there were understand, without resort to obscure esoteric expression (now
36 cans (latas) of gasoline in the building designed as "Bodega derisively termed "gobbledygook"). We reiterate the rule stated
No. 2" that was a separate structure not affected by the fire. It is in Bachrach vs. British American Assurance Co. (17 Phil. 555,
well to note that gasoline is not specifically mentioned among the 561):
prohibited articles listed in the so-called "hemp warranty." The
cause relied upon by the insurer speaks of "oils (animal and/or If the company intended to rely upon a condition of that
vegetable and/or mineral and/or their liquid products having a character, it ought to have been plainly expressed in the
flash point below 300 degrees Fahrenheit", and is decidedly policy.
ambiguous and uncertain; for in ordinary parlance, "Oils" mean
"lubricants" and not gasoline or kerosene. And how many
insured, it may well be wondered, are in a position to understand
or determine "flash point below 003o Fahrenheit. Here, again, by
reason of the exclusive control of the insurance company over
the terms and phraseology of the contract, the ambiguity must be
held strictly against the insurer and liberraly in favor of the
insured, specially to avoid a forfeiture (44 C. J. S., pp. 1166-1175;
29 Am. Jur. 180).
MERCANTILE LAW REVIEW || Insurance Law 36
Compilation of Digests (Except 37)

DOUBLE INSURANCE or damage, all benefits under this policy shall be deemed
forfeited, provided however, that this condition shall not
23. ARMANDO GEAGONIA, vs. COURT OF APPEALS and apply when the total insurance or insurances in force at
COUNTRY BANKERS INSURANCE CORPORATION the time of the loss or damage is not more than
G.R. No. 114427 February 6, 1995 Davide, Jr. J P200,000.00.

A double insurance exists where the same person is insured by On 27 May 1990, fire of accidental origin broke out and the
several insurers separately in respect of the same subject and insured stock-in-trade were completely destroyed prompting
interest. As earlier stated, the insurable interests of a mortgagor Geagonia to file with CBIC a claim under the policy. On 28
and a mortgagee on the mortgaged property are distinct and December 1990, CBIC denied the claim because it found that at
separate. the time of the loss, the petitioner's stocks-in-trade were
likewise covered by fire insurance policies No. GA-28146 and No.
FACTS GA-28144, for P100,000.00 each, issued by the Cebu Branch of
Armando Geagonia is the owner of Norman's Mart in San the Philippines First Insurance Co., Inc. (hereinafter PFIC). These
Francisco, Agusan del Sur. On 22 December 1989, he obtained policies indicate that the insured was "Messrs. Discount Mart
from Country Bankers Insurance Corporation {CBIC} fire (Mr. Armando Geagonia, Prop.)" with a mortgage clause reading:
insurance policy No. F-14622 for P100,000.00. The period of the MORTGAGE: Loss, if any shall be payable to Messrs. Cebu
policy was from 22 December 1989 to 22 December 1990 and Tesing Textiles, Cebu City as their interest may appear
covered the following: "Stock-in-trade consisting principally of subject to the terms of this policy. CO-INSURANCE
dry goods such as RTW's for men and women wear and other DECLARED: P100,000. — Phils. First CEB/F 24758.
usual to assured's business."
The petitioner declared in the policy under the subheading The basis of the private respondent's denial was the petitioner's
entitled CO-INSURANCE that Mercantile Insurance Co., Inc. was alleged violation of Condition 3 of the policy.
the co-insurer for P50,000.00. From 1989 to 1990, the petitioner
had in his inventory stocks amounting to P392,130.50, The petitioner then filed a complaint against the private
respondent with the Insurance Commission (Case No. 3340) for
The policy contained the following condition: the recovery of P100,000.00 under fire insurance policy No. F-
3. The insured shall give notice to the Company of any 14622.
insurance or insurances already affected, or which may
subsequently be effected, covering any of the property or The Insurance Commission found that the petitioner did not
properties consisting of stocks in trade, goods in process violate Condition 3. The Court of Appeals reversed the decision
and/or inventories only hereby insured, and unless such of the Insurance Commission
notice be given and the particulars of such insurance or
insurances be stated therein or endorsed in this policy ISSUE: Whether or not Double Insurance exist thereby
pursuant to Section 50 of the Insurance Code, by or on prohibiting petitioner Geagonia from recovering.
behalf of the Company before the occurrence of any loss
MERCANTILE LAW REVIEW || Insurance Law 37
Compilation of Digests (Except 37)

RULING: NO, double insurance does not exist. disclosure then of the former policies was not fatal to the
petitioner's right to recover on the private respondent's policy.
Provisions, conditions or exceptions in policies, which tend to
work a forfeiture of insurance policies, should be construed most Furthermore, by stating within Condition 3 itself that such
strictly against those for whose benefits they are inserted, and condition shall not apply if the total insurance in force at the time
most favorably toward those against whom they are intended to of loss does not exceed P200,000.00, the private respondent was
operate.  The reason for this is that, except for riders, which may amenable to assume a co-insurer's liability up to a loss not
later be inserted, the insured sees the contract already in its final exceeding P200,000.00. What it had in mind was to discourage
form and has had no voice in the selection or arrangement of the over-insurance. Indeed, the rationale behind the incorporation of
words employed therein. On the other hand, the language of the "other insurance" clause in fire policies is to prevent over-
contract was carefully chosen and deliberated upon by experts insurance and thus avert the perpetration of fraud. When a
and legal advisers who had acted exclusively in the interest of property owner obtains insurance policies from two or more
the insurers and the technical language employed therein is insurers in a total amount that exceeds the property's value, the
rarely understood by ordinary laymen. insured may have an inducement to destroy the property for the
purpose of collecting the insurance. The public as well as the
With these principles in mind, Condition 3 of the subject policy is insurer is interested in preventing a situation in which a fire
not totally free from ambiguity and must, perforce, be would be profitable to the insured.
meticulously analyzed. Such analysis leads us to conclude that
(a) the prohibition applies only to double insurance, and (b) the
nullity of the policy shall only be to the extent exceeding
P200,000.00 of the total policies obtained.

The first conclusion is supported by the portion of the condition


referring to other insurance "covering any of the property or
properties consisting of stocks in trade, goods in process and/or
inventories only hereby insured," and the portion regarding the
insured's declaration on the subheading CO-INSURANCE that the
co-insurer is Mercantile Insurance Co., Inc. in the sum of
P50,000.00. A double insurance exists where the same person is
insured by several insurers separately in respect of the same
subject and interest. As earlier stated, the insurable interests of a
mortgagor and a mortgagee on the mortgaged property are
distinct and separate. Since the two policies of the PFIC do not
cover the same interest as that covered by the policy of the
private respondent, no double insurance exists. The non-
MERCANTILE LAW REVIEW || Insurance Law 38
Compilation of Digests (Except 37)

24. MALAYAN INSURANCE CO., INC. vs. PHILIPPINES FIRST


INSURANCE CO., INC. and REPUTABLE FORWARDER Under the contract, Reputable undertook to answer for “all risks
SERVICES, INC. with respect to the goods and shall be liable to the COMPANY
G.R. No. 184300, July 11, 2012 J. REYES3 (Wyeth), for the loss, destruction, or damage of the
goods/products due to any and all causes whatsoever, including
By the express provision of Section 93 of the Insurance Code, theft, robbery, flood, storm, earthquakes, lightning, and other
double insurance exists where the same person is insured by force majeure while the goods/products are in transit and until
several insurers separately in respect to the same subject and actual delivery to the customers, salesmen, and dealers of the
interest. COMPANY”. The contract also required Reputable to secure an
insurance policy on Wyeth’s goods. Thus, on February 11, 1994,
FACTS:  Reputable signed a Special Risk Insurance Policy (SR Policy) with
Since 1989, Wyeth Philippines, Inc. (Wyeth) and respondent petitioner Malayan for the amount of P1,000,000.00. On October
Reputable Forwarder Services, Inc. (Reputable) had been 6, 1994, during the effectivity of the Marine Policy and SR Policy,
annually executing a contract of carriage, whereby the latter Reputable received from Wyeth 1,000 boxes of Promil infant
undertook to transport and deliver the former’s products to its formula worth P2,357,582.70 to be delivered by Reputable to
customers, dealers or salesmen. On November 18, 1993, Wyeth Mercury Drug Corporation in Libis, Quezon City.
procured Marine Policy No. MAR 13797 (Marine Policy) from
respondent Philippines First Insurance Co., Inc. (Philippines Unfortunately, on the same date, the truck carrying Wyeth’s
First) to secure its interest over its own products. Philippines products was hijacked by about 10 armed men. They threatened
First thereby insured Wyeth’s nutritional, pharmaceutical and to kill the truck driver and two of his helpers should they refuse
other products usual or incidental to the insured’s business to turn over the truck and its contents to the said highway
while the same were being transported or shipped in the robbers. The hijacked truck was recovered two weeks later
Philippines. without its cargo. Malayan questions its liability based on
sections 5 and 12 of the SR Policy.
The policy covers all risks of direct physical loss or damage from
any external cause, if by land, and provides a limit of ISSUE: 
P6,000,000.00 per any one land vehicle. On December 1, 1993, Whether or not there is double insurance in this case such that
Wyeth executed its annual contract of carriage with Reputable. It either Section 5 or Section 12 of the SR Policy may be applied.
turned out, however, that the contract was not signed by Wyeth’s
representative/s. Nevertheless, it was admittedly signed by HELD: 
Reputable’s representatives, the terms thereof faithfully NO. By the express provision of Section 93 of the Insurance Code,
observed by the parties and, as previously stated, the same double insurance exists where the same person is insured by
contract of carriage had been annually executed by the parties several insurers separately in respect to the same subject and
every year since 1989. interest. The requisites in order for double insurance to arise are
as follows:
3
Same with case number 57
MERCANTILE LAW REVIEW || Insurance Law 39
Compilation of Digests (Except 37)

1. The person insured is the same; 


2. Two or more insurers insuring separately; 
3. There is identity of subject matter; 
4. There is identity of interest insured; and 
5. There is identity of the risk or peril insured against.

In the present case, while it is true that the Marine Policy and the
SR Policy were both issued over the same subject matter, i.e.
goods belonging to Wyeth, and both covered the same peril
insured against, it is, however, beyond cavil that the said policies
were issued to two different persons or entities. It is undisputed
that Wyeth is the recognized insured of Philippines First under
its Marine Policy, while Reputable is the recognized insured of
Malayan under the SR Policy. The fact that Reputable procured
Malayan’s SR Policy over the goods of Wyeth pursuant merely to
the stipulated requirement under its contract of carriage with
the latter does not make Reputable a mere agent of Wyeth in
obtaining the said SR Policy.

The interest of Wyeth over the property subject matter of both


insurance contracts is also different and distinct from that of
Reputable’s. The policy issued by Philippines First was in
consideration of the legal and/or equitable interest of Wyeth
over its own goods. On the other hand, what was issued by
Malayan to Reputable was over the latter’s insurable interest
over the safety of the goods, which may become the basis of the
latter’s liability in case of loss or damage to the property and falls
within the contemplation of Section 15 of the Insurance Code.

Therefore, even though the two concerned insurance policies


were issued over the same goods and cover the same risk, there
arises no double insurance since they were issued to two
different persons/entities having distinct insurable interests.
Necessarily, over insurance by double insurance cannot likewise
exist. Hence, as correctly ruled by the RTC and CA, neither
Section 5 nor Section 12 of the SR Policy can be applied.
MERCANTILE LAW REVIEW || Insurance Law 40
Compilation of Digests (Except 37)

NOTICE OF LOSS prescriptive period was indicated therein.

25. PHILIPPINES AMERICAN GENERAL INSURANCE Co., INC. The trial court ruled in favor of PHILAMGEN and TAGUM
and TAGUM PLASTICS, INC. vs. SWEET LINES INC., DAVAO and ordered Sweet Lines to pay the sum of P34,902.00 with legal
VETERANS ARRASTRE and CA interest. On appeal, the CA reversed the decision of thhe trial
G.R. no. 87434, August 5, 1992, J. REGALADO court on the basis of prescription.

Before an action can properly be commenced all the ISSUE: Whether or not there was substantial compliance on the
essential elements of the cause of action must be in existence. part of petitioners with regard to their claims for loss/damages.

FACTS RULING

Petitioners Philippine American General Insurance Co., NO. The petitioners have not substantially complied with
(PHILAMGEN) and Tagum Plastics, Inc. (TAGUM) were insurers the conditions precedent to their right of action. All valid
and importers of Low Density Polyethylene (basic material for conditions precedent to the institution of the particular action
plastics). The said merchandise are to be shipped from the U.S. wheter prescribed by statute, fixed by agreement of the parties
through an Indian Ship “ SS Visva Yash” and are to be received in or implied by law must be performed or complied with before
Manila. After which the imported items will be shipped off to commencing the action, unless the conduct of the adverse party
Davao. has been such as to prevent or waive performance or excuse
non-performance of the condition.
The Indian vessel arrived in Manila and sought the
services of respondent Sweet Lines for the inter-island shipment Stipulations in bills of lading requiring notice of claim for
to Davao. However, when respondent’s MV Sweet Love arrived loss or damage is a condition precedent. The carrier is not liable
in Davao, TAGUM found out that some of the imported if notice is not given in accordance with the stipulation.
polyethylene were either missing or damaged beyond point of
being useful for the intended purpose. Delivered were only
5,820 bags only out of the ordered 7,000 bags.

As a consequence, PHILAMGEN and TAGUM filed a suit


against Sweet Lines and Davao Veterans Arrastre based on the
Bills of Lading. However, bills of lading and notice of loss were
not formally offered as evidence, hence it was not shown that
damage were incurred by the carrier and that a contractual
MERCANTILE LAW REVIEW || Insurance Law 41
Compilation of Digests (Except 37)

PREMIUMS
HELD: No.
26. Arce v. Capital Insurance & Surety Co., Inc.,
117 SCRA 63 (1982), ABAD SANTOS, J.: Sec. 72 of the Insurance Act, as amended by R.A. No. 3540 reads:

The amendment to Sec. 72 has radically changed the legal regime SEC. 72. An insurer is entitled to payment of
in that unless the premium is paid, there is no insurance. premium as soon as the thing insured is exposed
to the perils insured against, unless there is clear
FACTS: agreement to grant credit extension for the
premium due. No policy issued by an insurance
The INSURED (Pedro Arce) was the owner of a residential house company is valid and binding unless and until the
in Tondo, Manila, which had been insured with the COMPANY premium thereof has been paid " (Italics supplied.)
(CISCI) for Fire Insurance since 1961 When the COMPANY sent
to the INSURED a Renewal Certificate, the COMPANY also Moreover, the parties in this case had stipulated:
requested payment of the corresponding premium. However, the
INSURED could not yet able to pay the premium so he ask the IT IS HEREBY DECLARED AND AGREED that not.
COMPANY for an extension to pay the premium. The COMPANY withstanding anything to the contrary contained
acceded. Unfortunately, the INSURED still failed to pay the in the within policy, this insurance will be
premium on the agreed extension date. Four days after the lapse deemed valid and binding upon the Company
of the agreed extension date, the house of the INSURED was only when the premium and documentary stamps
totally destroyed by fire. therefor have actually been paid in full and duly
acknowledged in an official receipt signed by an
The INSURED then presented a claim for indemnity to the authorized official/representative of the
COMPANY. The COMPANY answered that no indemnity was due Company, " (pp. 45-46, Record on Appeal.)
because the premium on the policy was not paid. Nonetheless,
the COMPANY tendered a financial aid to the INSURED. After It is obvious from both the Insurance Act, as amended, and the
receiving the financial aid, the INSURED then sued the COMPANY stipulation of the parties that time is of the essence in respect of
on the policy for non-payment of indemnity. the payment of the insurance premium so that if it is not paid the
contract does not take effect unless there is still another
The trial court ruled in favor of the INSURED and so ordered the stipulation to the contrary. In the instant case, the INSURED was
COMAPANY to pay the INSURED for indemnity under the policy. given a grace period to pay the premium but the period having
The COMPANY appealed to the SC via a petition for review on expired with no payment made, he cannot insist that the
pure questions of law. COMPANY is nonetheless obligated to him.

ISSUE: Whether or not the INSURED is entitled to indemnity The amendment to Sec. 72 has radically changed the legal regime
under the fire insurance policy even if no premium was paid? in that unless the premium is paid there is no insurance.
MERCANTILE LAW REVIEW || Insurance Law 42
Compilation of Digests (Except 37)

27. MALAYAN INSURANCE CO., INC. (MICO) vs. GREGORIA On February 5, 1982, Pinca's payment was returned by MICO to
CRUZ ARNALDO Adora on the ground that her policy had been cancelled earlier.
G.R. No. L-67835 October 12, 1987, CRUZ, J But Adora refused to accept it.

SEC. 64. No policy of insurance other than life shall be cancelled by Pinca made the requisite demands for payment, which MICO
the insurer except upon prior notice thereof to the insured, and no rejected. She then went to the Insurance Commission and was
notice of cancellation shall be effective unless it is based on the ultimately sustained by the latter.
occurrence, after the effective date of the policy, of one or more of
the following: MICO's contends that there was no existing insurance at the time
(a) non-payment of premium; of the loss sustained by Pinca because her policy never became
effective for non-payment of premium. It also further argued that
SEC. 65. All notices of cancellation mentioned in the preceding since there was no payment of premium, the policy had been
section shall be in writing, mailed or delivered to the named cancelled before the occurrence of the loss.
insured at the address shown in the policy, and shall state
(a) which of the grounds set forth in section sixty-four is relied Pinca denied the contention of MICO that it cancelled the policy
upon and in question on October 15, 1981, for non-payment of premium
(b) that, insured, the upon written request of the named insurer and considering the strict language of Section 64 of the Insurance
will furnish the facts on which the cancellation is based. Code that no insurance policy shall be cancelled except upon
prior notice, it behooved MICO to make sure that the cancellation
FACTS: was actually sent to and received by the insured.
The petitioner (hereinafter called MICO) issued to the private
respondent, Coronacion Pinca, Fire Insurance Policy on her MICO contends that Pinca knew the policy had already been
property effective July 22, 1981, until July 22, 1982. cancelled and that when she paid the premium on December 24,
1981, her purpose was "to renew it." As this could NOT be done
On October 15,1981, MICO allegedly cancelled the policy for non- by the agent alone under the terms of the original policy, the
payment, of the premium and sent the corresponding notice to renewal thereof did not legally bind MICO. which had not ratified
Pinca. it.

On December 24, 1981, payment of the premium for Pinca was ISSUE: Whether MICO validly cancelled the policy due to non-
received by Domingo Adora, agent of MICO.  payment of premiums under Section 64 in accordance with
Section 65 of the Insurance Code.
On January 15, 1982, Adora remitted this payment to MICO,
together with other payments.  RULING: No. Citing the relevant provisions of the Insurance
Code
On January 18, 1982, Pinca's property was completely burned.  Section 64 reads as follows:
MERCANTILE LAW REVIEW || Insurance Law 43
Compilation of Digests (Except 37)

SEC. 64. No policy of insurance other than life shall be


cancelled by the insurer except upon prior notice thereof
to the insured, and no notice of cancellation shall be
effective unless it is based on the occurrence, after the
effective date of the policy, of one or more of the
following:
(b) non-payment of premium;
XXXX
(Other grounds omitted)

As for the method of cancellation, Section 65 provides as follows:


SEC. 65. All notices of cancellation mentioned in the
preceding section shall be in writing, mailed or delivered
to the named insured at the address shown in the policy,
and shall state (a) which of the grounds set forth in
section sixty-four is relied upon and (b) that, insured, the
upon written request of the named insurer will furnish
the facts on which the cancellation is based.

The court finds no proof that the notice, assuming it complied


with the other requisites mentioned above, was actually mailed
to and received by Pinca. All MICO's offers to show that the
cancellation was communicated to the insured is its employee's
testimony that the said cancellation was sent "by mail through
our mailing section." without more.

The court also look askance (with disapproval or distrust) at the


alleged cancellation, of which the insured and MICO's agent
himself had no knowledge, and the curious fact that although
Pinca's payment was remitted to MICO's by its agent on January
15, 1982, MICO sought to return it to Adora only on February 5,
1982, after it presumably had learned of the occurrence of the
loss insured against on January 18, 1982. These circumstances
make the motives of the petitioner highly suspect, to say the
least, and cast serious doubts upon its candor and bona fides.
MERCANTILE LAW REVIEW || Insurance Law 44
Compilation of Digests (Except 37)

28. PHILIPPINE PHOENIX SURETY & INSURANCE, INC. vs. therefor had been complied with, arose and became binding
WOODWORKS, INC. upon it, while the obligation of the insured to pay the remainder
G.R. No. L-22684, August 31, 1967, DIZON, J.: of the total amount of the premium due became demandable. 

As the contract had become perfected, the parties could demand The appellant's theory that non-payment by it of the premium
from each other the performance of whatever obligations they had due, produced the cancellation of the contract of insurance is
assumed. misplaced. Such theory would place exclusively in the hands of
one of the contracting parties the right to decide whether the
FACTS: contract should stand or not. Rather the correct view would
Appellee Philippine Phoenix Surety & Insurance Co., Inc. seem to be this: as the contract had become perfected, the parties
commenced this action in the Municipal Court of Manila to could demand from each other the performance of whatever
recover from appellant Woodworks, Inc. the sum of P3,522.09, obligations they had assumed. In the case of the insurer, it is
representing the unpaid balance of the premiums on a fire obvious that it had the right to demand from the insured the
insurance policy issued by appellee in favor of appellant for a completion of the payment of the premium due or sue for the
term of one year from April 1, 1960 to April 1, 1961. rescission of the contract. As it chose to demand specific
performance of the insured's obligation to pay the balance of the
On April 1, 1960, Philippine Phoenix Surety issued to premium, the latter's duty to pay is indeed indubitable.
Woodworks, Inc. Fire Policy No. 9652 for the amount of
P300,000.00. The premiums of said policy amounted to
P6,051.95. Woodworks paid P3,000.00 on September 22, 1960.
Philippine Phoenix Surety made several demands on defendant
to pay the amount of P3,522.09.1äwphï1.ñët

ISSUE:
Whether the risk in fire insurance policies attached upon the
partial payment of premium and subsequently the issuance and
delivery of the policy to the insured.

HELD:
YES. The risk already attached. There is, consequently, no doubt
at all that, as between the insurer and the insured, there was not
only a perfected contract of insurance but a partially performed
one as far as the payment of the agreed premium was concerned.
Thereafter the obligation of the insurer to pay the insured the
amount for which the policy was issued in case the conditions
MERCANTILE LAW REVIEW || Insurance Law 45
Compilation of Digests (Except 37)

29. SPS. ANTONIO A. TIBAY and VIOLETA R. TIBAY and provided herein. This policy shall be deemed effective, valid and
OFELIA M. RORALDO, VICTORINA M. RORALDO, VIRGILIO M. binding upon the Company only when the premiums therefor
RORALDO, MYRNA M. RORALDO and ROSABELLA M. have actually been paid in full and duly acknowledged in a
RORALDO vs. COURT OF APPEALS and FORTUNE LIFE AND receipt signed by any authorized official of the company
GENERAL INSURANCE CO., INC Where the premium has only been partially paid and the balance
G.R. No. 119655 May 24, 1996, BELLOSILLO, J paid only after the peril insured against has occurred, the
insurance contract did not take effect and the insured cannot
FACTS: collect at all on the policy. 

Violeta R. Tibay and/or Nicolas Roraldo insured their 2-story The Insurance Code which says that no policy or contract of
residential building with Fortune Life and General Insurance Co., insurance issued by an insurance company is valid and binding
Inc. (FORTUNE) for the amount Php 600,000.00 with total unless and until the premium has been paid.
premium of P2, 983.50. Tibay only paid P600.00 for the period
January 23, 1987 to January 23, 1988. On March 8, 1987, the What does “unless and until the premium thereof has been paid”
insured building was completely burned. Two days after, Tibay mean?
paid the balance premium and on the same day, demanded Escosura v. San Miguel- the legislative practice was to interpret
payment she filed with FORTUNE a claim on the fire insurance “with pay” in accordance to the intention of distinguish between
policy. FORTUNE denied the claim of Violeta for violation of full and partial payment, where the modifying term is used.
Policy Condition No. 2 and of Sec. 77 of the Insurance Code.
Violeta and other petitioners filed a case for damages. The trial Petitioners used Philippine Phoenix v. Woodworks, where partial
court adjudged FORTUNE liable for the insured value of the payment of the premium made the policy effective during the
building and the personal properties. On appeal, the CA reversed whole period of the policy.
the decision. Hence, a petition was raised to the SC.
The SC didn’t consider the 1967 Phoenix case as persuasive due
ISSUE: to the different factual scenario.

Whether or not a fire insurance policy is valid, binding and In Makati Tuscany v CA, the parties mutually agreed that the
enforceable upon mere partial payment of premium. premiums could be paid in installments, hence, this Court
refused to invalidate the insurance policy.
RULING:
Nothing in Article 77 of the Code suggested that the parties may
The pertinent provisions read: not agree to allow payment of the premiums in installment, or to
consider the contract as valid and binding upon payment of the
2. This policy including any renewal thereof and/or any first premium. Phoenix and Tuscany demonstrated the waiver of
endorsement thereon is not in force until the premium has been prepayment in full by the insurer. In this case however, there
fully paid to and duly receipted by the Company in the manner
MERCANTILE LAW REVIEW || Insurance Law 46
Compilation of Digests (Except 37)

was no waiver. There was a stipulation that the policy wasn’t in


force until the premium has been fully paid and receipted.

There was no juridical tie of indemnification from the fractional


payment of premium.  The insurance contract itself expressly
provided that the policy would be effective only when
the premium was paid in full. Verily, it is elemental law that the
payment of premium is requisite to keep the policy of insurance
in force.  If the premium is not paid in the manner prescribed in
the policy as intended by the parties the policy is ineffective. 
Partial payment even when accepted as a partial payment will
not keep the policy alive.

South Sea v CA stipulated 2 exceptions to the requirement of


payment of the entire premium as a prerequisite to the validity
of the insurance contract.  These are when in case the insurance
coverage relates to life or insurance when a grace period applies,
and when the insurer makes a written acknowledgment of the
receipt of premium to be conclusive evidence of payment.
Hence, in the absence of clear waiver of prepayment in full by the
insurer, the insured cannot collect on the proceeds of the policy.

“The terms of the insurance policy constitute the measure of the


insurer’s liability. In the absence of statutory prohibition to the
contrary, insurance companies have the same rights as
individuals to limit their liability and to impose whatever
conditions they deem best upon their obligations not
inconsistent with public policy.”
MERCANTILE LAW REVIEW || Insurance Law 47
Compilation of Digests (Except 37)

30. UCPB Gen. Insurance Co. Inc vs Masagana Telamart Inc On the same day, July 14, 1992, petitioner returned to
G.R. No. 137172 June 15, 19994 respondent the five (5) manager’s checks that it tendered, and at
the same time rejected respondent’s claim for the reasons (a)
An insurer is entitled to payment of the premium as soon as the that the policies had expired and were not renewed, and (b) that
thing insured is exposed to the peril insured against. the fire occurred on June 13, 1992, before respondent’s tender of
Notwithstanding any agreement to the contrary, no policy or premium payment.
contract of insurance issued by an insurance company is valid and
binding unless and until the premium thereof has been paid, except
in the case of a life or an industrial life policy whenever the grace Issue:
period provision applies.
Whether or not respondent is entitled to compensation
despite the renewal of the insurance policy after the occurrence
of the event insured.
Facts:
On April 15, 1991, petitioner issued five (5) insurance
policies covering respondent’s various property described Held:
therein against fire, for the period from May 22, 1991 to May 22,
1992. In March 1992, petitioner evaluated the policies and No. An insurance policy, other than life, issued originally
decided not to renew them upon expiration of their terms on or on renewal, is not valid and binding until actual payment of
May 22, 1992. Petitioner advised respondent’s broker, Zuellig the premium. Any agreement to the contrary is void. 11 The
Insurance Brokers, Inc. of its intention not to renew the policies. parties may not agree expressly or impliedly on the extension of
On April 6, 1992, petitioner gave written notice to respondent of creditor time to pay the premium and consider the policy
the non-renewal of the policies at the address stated in the binding before actual payment.
policies. On June 13, 1992, fire razed respondent’s property Here, the payment of the premium for renewal of the
covered by three of the insurance policies petitioner issued. On policies was tendered on July 13, 1992, a month after the fire
July 13, 1992, respondent presented to petitioner’s cashier at its occurred on June 13, 1992. The assured did not even give the
head office five (5) manager’s checks in the total amount of insurer a notice of loss within a reasonable time after occurrence
P225,753.95, representing premium for the renewal of the of the fire.
policies from May 22, 1992 to May 22, 1993. No notice of loss
was filed by respondent under the policies prior to July 14, 1992.
On July 14, 1992, respondent filed with petitioner its formal
claim for indemnification of the insured property razed by fire.

4
Same with case number 33
MERCANTILE LAW REVIEW || Insurance Law 48
Compilation of Digests (Except 37)

31. GULF RESORTS, INC. vs. PHILIPPINE CHARTER indemnify another against loss, damage or liability arising from
INSURANCE CORPORATION an unknown or contingent event. Thus, an insurance contract
G.R. No. 156167 May 16, 2005 J. Puno exists where the following elements concur:
A contract of adhesion is one wherein a party, usually a
corporation, prepares the stipulations in the contract, while the 1. The insured has an insurable interest;
other party merely affixes his signature or his "adhesion" thereto.
Through the years, the courts have held that in these type of 2. The insured is subject to a risk of loss by the happening of the
contracts, the parties do not bargain on equal footing, the weaker designated peril;
party's participation being reduced to the alternative to take it or
leave it. Thus, these contracts are viewed as traps for the weaker 3. The insurer assumes the risk;
party whom the courts of justice must protect.
4. Such assumption of risk is part of a general scheme to
FACTS: distribute actual losses among a large group of persons bearing a
Gulf was insured by American Home Assurance Company which similar risk; and
includes loss or damage to shock to any of the property insured
by the Policy occasioned by or through in consequence of 5. In consideration of the insurer's promise, the insured pays a
earthquake. Later on, an earthquake struck Central and Northern premium.26 (Emphasis ours)
Luzon causing damage to the property of Gulf. Gulf claimed from
the insurance company but was denied as the earthquake shock An insurance premium is the consideration paid an insurer for
coverage only covered the swimming pools of the resort. Gulf undertaking to indemnify the insured against a specified peril. In
contends that pursuant to this rider, no qualifications were fire, casualty, and marine insurance, the premium payable
placed on the scope of the earthquake’s shock coverage. Thus, becomes a debt as soon as the risk attaches. In the subject policy,
the policy extended to all of the insured properties. The RTC no premium payments were made with regard to earthquake
ruled in favor of American Home to which the CA affirmed. shock coverage, except on the two swimming pools. There is no
mention of any premium payable for the other resort properties
ISSUE: with regard to earthquake shock.
Whether or not Gulf is entitled to the earthquake shock coverage.

HELD:
NO. Gulf was entitled to claim.

A careful examination of the premium recapitulation will show


that it is the clear intent of the parties to extend earthquake
shock coverage only to the two swimming pools. Section 2(1) of
the Insurance Code defines a contract of insurance as an
agreement whereby one undertakes for a consideration to
MERCANTILE LAW REVIEW || Insurance Law 49
Compilation of Digests (Except 37)

32. American Home Association Co. vs Antonio Chua Regional Trial Court rule in favour of Antonio Chua for paying by
G.R. No. 130421, June 28, 1999, Davide, Jr. C.J. way of check a day before the fire occurred and the Court of
Appeals Affirmed. 
An insurer is entitled to payment of the premium as soon as the
thing insured is exposed to the peril insured against. 
Notwithstanding any agreement to the contrary, no policy or ISSUE/S:
contract of insurance issued by an insurance company is valid and 1. W/N there was a valid payment of premium considering that
binding unless and until the premium thereof has been paid, except the check was cashed after the occurrence of the fire since the
in the case of life or an industrial life policy whenever the grace renewal certificate issued containing the acknowledgement
period provision applies. receipt
2. W/N Chua violated the policy by his submission of fraudulent
Also, Section 306 of the Insurance Code provides that any documents and non-disclosure of the other existing insurance
insurance company which delivers a policy or contract of contracts or “other insurance clause"
insurance to an insurance agent or insurance broker shall be
deemed to have authorized such such agent or broker to receive on HELD:
its behalf payment of any premium which is due on such policy or
contract of insurance at the time of its issuance or delivery or 1. YES. 
which becomes due thereon Basis is Section 77 of the Insurance Code. An insurer is entitled to
payment of the premium as soon as the thing insured is exposed
FACTS: On April 5, 1990: Antonio Chua renewed the fire to the peril insured against.  Notwithstanding any agreement to
insurance for its stock-in-trade of his business, Moonlight the contrary, no policy or contract of insurance issued by an
Enterprises with American Home Assurance Company by issuing insurance company is valid and binding unless and until the
a check of P2,983.50 to its agent James Uy who delivered the premium thereof has been paid, except in the case of life or an
Renewal Certificate to him. On April 6, 1990: Moonlight industrial life policy whenever the grace period provision
Enterprises was completely razed by fire with an est. loss applies.
of P4,000,000 to P5,000,000. On April 10, 1990: An official
receipt was issued and subsequently, a policy was issued Section 66 of the Insurance Code is not applicable since
covering March 25 1990 to March 25 1991. not termination but renewal
renewal certificate issued contained the acknowledgment that
Antonio Chua filed an insurance claim with American Home and premium had been paid. 
4 other co-insurers (Pioneer Insurance and Surety Corporation, Section 306 of the Insurance Code provides that any insurance
Prudential Guarantee and Assurance, Inc. and Filipino Merchants company which delivers a policy or contract of insurance to an
Insurance Co), however American Home refused alleging the no insurance agent or insurance broker shall be deemed to have
premium was paid authorized such agent or broker to receive on its behalf payment
of any premium which is due on such policy or contract of
insurance at the time of its issuance or delivery or which
MERCANTILE LAW REVIEW || Insurance Law 50
Compilation of Digests (Except 37)

becomes due thereon. Best evidence of such authority is the fact


that petitioner accepted the check and issued the official receipt
for the payment.  It is, as well, bound by its agent’s
acknowledgment of receipt of payment.

Section 78 of the Insurance Code provides that an


acknowledgment in a policy or contract of insurance of the
receipt of premium is conclusive evidence of its payment, so far
as to make the policy binding, notwithstanding any stipulation
therein that it shall not be binding until the premium is actually
paid.

This Section establishes a legal fiction of payment and


should be interpreted as an exception to Section 77.
 
2. NO.

The purpose for the “other insurance clause”  is to prevent an


increase in the moral hazard
failure to disclose was not intentional and fraudulent. A policy
may declare that a violation of specified provisions thereof shall
avoid it, otherwise the breach of an immaterial provision does
not avoid the policy.
American Home is estopped because its loss adjusters
had previous knowledge of the co-insurers.
 
The loss adjuster, being an employee of petitioner, is deemed a
representative of the latter whose awareness of the other
insurance contracts binds petitioner. There is no legal and factual
basis for the award of P200,000 for loss of profit. Also, no such
fraud or bad faith therefore no moral damages.
MERCANTILE LAW REVIEW || Insurance Law 51
Compilation of Digests (Except 37)

33. UCPB General Ins. Co. Inc., vs. Masagana Telamart, Inc.             Masagana filed a civil complaint for recovery of the face
G.R. No. 137172, April5 value of the policies covering the insured property razed by fire.
RTC ruled in favor of Masagana, as it found it to have complied
An insurance policy, other than life, issued originally or on with the obligation to pay the premium; hence, the replacement-
renewal, is not valid and binding until actual payment of the renewal policy of these policies are effective and binding for
premium. Any agreement to the contrary is void.  another year [22 May 1992 – 22 May 1993].

FACTS             CA affirmed RTC, holding that following previous practice,


Masagana was allowed a 60-90 day credit term for the renewal
In 1991, UCPB issued 5 fire insurance policies covering of its policies, and that the acceptance of the late premium
Masagana Telamart’s various properties for the period from 22 payment suggested that payment could be made later.
May 1991 to 22 May 1992.
ISSUE
On March 1992 [2 months before policy expiration],
UCPB evaluated the policies and decided not to renew them upon Whether or not the fire insurance policies had expired on
expiration of their terms on 22 May 1992.  UCPB advised 22 May 1992, or had been extended or renewed by an implied
Masagana’s broker of its intention not to renew the policies. On credit arrangement though actual payment of premium was
April 1992 [1 month before policy expiration], UCPB gave tendered on a later date after the occurrence of the risk insured
written notice to Masagana of the non-renewal of the policies. On against fire.
June 1992 [policy already expired], Masagana’s property covered
by 3 UCPB-issued policies was razed by fire. HELD
YES. The fire insurance policies had expired.
On 13 July 1992, Masagana presented to UCPB’s cashier 5
manager’s checks, representing premium for the renewal of the An insurance policy, other than life is not valid and
policies for another year. binding until actual payment of the premium.  Any agreement to
the contrary is void. The parties may not agree expressly or
It was only on the following day, 14 July 1992, when impliedly on the extension of credit or time to pay the premium
Masagana filed with UCPB a formal claim for indemnification of and consider the policy binding before actual payment.
the insured property razed by fire. On the same day, UCPB
returned the 5 manager’s checks, and rejected Masagana’s claim The case of Malayan Insurance v. Cruz-Arnaldo cited by
since the policies had expired and were not renewed, and the fire the CA is not applicable. In that case, payment of the premium
occurred on 13 June 1992 (or before tender of premium was made on before the occurrence of the fire.  In the present
payment). case, the payment of the premium for renewal of the policies was
tendered a month after the fire occurred.  Masagana did not even
give UCPB a notice of loss within a reasonable time after
5
Same with case number 30 occurrence of the fire.
MERCANTILE LAW REVIEW || Insurance Law 52
Compilation of Digests (Except 37)

34. Makati Tuscany Condo. Corp. vs. Court of Appeals, Held:


G.R. No. 95546, Nov. 6, 1992, Bellosillo, J.
No. We hold that the subject policies are valid even if the
premiums were paid on installments. The records clearly show
Basic principles of equity and fairness would not allow the insurer that petitioner and private respondent intended subject
to continue collecting and accepting the premiums, although paid insurance policies to be binding and effective notwithstanding
on installments, and later deny liability on the lame excuse that the staggered payment of the premiums. The initial insurance
the premiums were not prepared in full. contract entered into in 1982 was renewed in 1983, then in
1984. In those three (3) years, the insurer accepted all the
Facts: installment payments. Such acceptance of payments speaks
American Home Assurance Co. (AHAC) issued in favor of Makati loudly of the insurer's intention to honor the policies it issued to
Tuscany Condominium Corporation (TUSCANY) Insurance Policy petitioner. Certainly, basic principles of equity and fairness
on the latter's building and premises for 2 consecutive years. The would not allow the insurer to continue collecting and accepting
premium was paid on installments, all of which were accepted by the premiums, although paid on installments, and later deny
AHAC. The policy was again renewed. On this renewed policy, liability on the lame excuse that the premiums were not
Tuscany made two installment payments, both accepted by prepared in full.
AHAC. Thereafter, Tuscany refused to pay the balance of the It appearing from the peculiar circumstances that the parties
premium. AHAC filed an action to recover the unpaid balance. actually intended to make three (3) insurance contracts valid,
Tuscany argues that there cannot be a perfected contract of effective and binding, petitioner may not be allowed to renege on
insurance upon mere partial payment of the premiums because its obligation to pay the balance of the premium after the
under Sec. 77 of the Insurance Code, no contract of insurance is expiration of the whole term of the third policy in March 1985.
valid and binding unless the premium thereof has been paid, Moreover, as correctly observed by the appellate court, where
notwithstanding any agreement to the contrary. As a the risk is entire and the contract is indivisible, the insured is not
consequence, Tuscany seeks a refund of all premium payments entitled to a refund of the premiums paid if the insurer was
made on the alleged invalid insurance policies. exposed to the risk insured for any period, however brief or
momentary.
Issue:
Whether payment by installment of the premiums due on an
insurance policy invalidates the contract of insurance, in view of
Sec. 77 of the Insurance Code and entitles the insured for the
refund of premium.
MERCANTILE LAW REVIEW || Insurance Law 53
Compilation of Digests (Except 37)

REINSTATEMENT Violeta filed a claim however the same was denied as the
contract was not reinstated.
35. VIOLETA R. LALICAN vs. THE INSULAR LIFE ASSURANCE
COMPANY LIMITED, AS REPRESENTED BY THE PRESIDENT ISSUE:
VICENTE R. AVILON Whether or not the Violeta could still recover from Insular Life.
GR No. 183526 August 25, 2009 J. Chico Nazario
HELD:
The stipulation in a life insurance policy giving the insured the NO. Violeta could no longer claim from Insular Life.
privilege to reinstate it upon written application does not give the
insured absolute right to such reinstatement by the mere filing of In the instant case, Eulogio’s death rendered impossible full
an application. The insurer has the right to deny the reinstatement compliance with the conditions for reinstatement of Policy No.
if it is not satisfied as to the insurability of the insured and if the 9011992. True, Eulogio, before his death, managed to file his
latter does not pay all overdue premium and all other Application for Reinstatement and deposit the amount for
indebtedness to the insurer. payment of his overdue premiums and interests thereon with
Malaluan; but Policy No. 9011992 could only be considered
FACTS: reinstated after the Application for Reinstatement had been
processed and approved by Insular Life during Eulogio’s lifetime
Eulogio Lalican applied for an insurance policy with Insular Life. and good health.
Malaluan, agent of Insular Life, issued in a policy in favor of
Eulogio which contained a 20-Year Endowment Variable Income Relevant herein is the following pronouncement of the Court in
Package. Violeta, Eulogio’s wife was named as primary Andres v. The Crown Life Insurance Company, citing McGuire v.
beneficiary. Under the terms of the policy, Eulogio was to pay The Manufacturer's Life Insurance Co.:
premiums on a quarterly basis. In the same, there was a 31-day
grace period and failure to make payments on the due dates "The stipulation in a life insurance policy giving the insured the
would render the policy in default and if unpaid within the grace privilege to reinstate it upon written application does not give
period the policy remains unpaid, it would make the policy void. the insured absolute right to such reinstatement by the mere
Eulogio faid to pay one of the premiums even within the grace filing of an application. The insurer has the right to deny the
period causing the policy to become void. Eulogio submitted to reinstatement if it is not satisfied as to the insurability of the
Insular Life, through Malaluan an Application for the insured and if the latter does not pay all overdue premium and
Reinstatement of the Policy but the same could not be reinstated all other indebtedness to the insurer. After the death of the
yet because of an interest he owes Insular Life. As Malaluan was insured the insurance Company cannot be compelled to
on a business errand, her husband received Eulogio’s second entertain an application for reinstatement of the policy because
Application for Reinstatement and issued a receipt for the the conditions precedent to reinstatement can no longer be
amount Eugolio deposited however, on the same day, Eulogio determined and satisfied."
died of cardio-respiratory arrest secondary to electrocution.
MERCANTILE LAW REVIEW || Insurance Law 54
Compilation of Digests (Except 37)

It does not matter that when he died, Eulogio’s Application for


Reinstatement and deposits for the overdue premiums and
interests were already with Malaluan. Insular Life, through the
Policy Contract, expressly limits the power or authority of its
insurance agents, thus:

Our agents have no authority to make or modify this contract, to


extend the time limit for payment of premiums, to waive any
lapsation, forfeiture or any of our rights or requirements, such
powers being limited to our president, vice-president or persons
authorized by the Board of Trustees and only in writing.
(Emphasis ours.)

Malaluan did not have the authority to approve Eulogio’s


Application for Reinstatement. Malaluan still had to turn over to
Insular Life Eulogio’s Application for Reinstatement and
accompanying deposits, for processing and approval by the
latter.
MERCANTILE LAW REVIEW || Insurance Law 55
Compilation of Digests (Except 37)

MARINE INSURANCE Held:


No. A perusal of the decisions of the courts below in all three
36. Aboitiz Shipping Corporation vs. Court of Appeals, et. al.,
petitions reveals that there is a categorical finding of negligence
G.R. No. 121833, Oct. 17, 2008, Tinga, J.
on the part of Aboitiz.
As a general rule, a ship owners liability is merely co-extensive
In this jurisdiction, the limited liability rule is embodied in
with his interest in the vessel, except where actual fault
Articles 587, 590 and 837 under Book III of the Code of
is attributable to the shipowner.
Commerce, thus:
Facts: Respondent Malayan Insurance Company, Inc. (Malayan)  
filed five separate actions against several defendants for the Art. 587. The ship agent shall also be civilly liable
collection of the amounts of the cargoes allegedly paid by for the indemnities in favor of third persons
Malayan under various marine cargo policies issued to the which may arise from the conduct of the captain
insurance claimants. Aboitiz raised the defenses of lack of in the care of the goods which he loaded on the
jurisdiction, lack of cause of action and prescription. It also vessel; but he may exempt himself therefrom by
claimed that M/V P. Aboitiz was seaworthy, that it exercised abandoning the vessel with all her equipment and
extraordinary diligence and that the loss was caused by a the freight it may have earned during the voyage.
fortuitous event.  
Art. 590. The co-owners of the vessel shall be
Respondents Asia Traders Insurance Corporation (Asia civilly liable in the proportion of their interests in
Traders) and Allied Guarantee Insurance Corporation (Allied) the common fund for the results of the acts of the
filed separate actions for damages against Aboitiz to recover by captain referred to in Art. 587.
way of subrogation the value of the cargoes insured by them and  
lost in the sinking of the vessel M/V P. Aboitiz. Aboitiz Each co-owner may exempt himself from this
reiterated the defense of force majeure. liability by the abandonment, before a notary, of
the part of the vessel belonging to him.
On 27 February 1981, Equitable Insurance Corporation  
(Equitable) filed an action for damages against Aboitiz to Art. 837. The civil liability incurred
recover by way of subrogation the value of the cargoes by shipowners in the case prescribed in this
insured by Equitable that were lost in the sinking of M/V P. section, shall be understood as limited to the
Aboitiz. Aboitiz invoked the doctrine of limited liability and value of the vessel with all its appurtenances and
claimed that the typhoon was the proximate cause of the loss. freightage served during the voyage.

Issue: Whether Aboitiz can avail limited liability on the basis of These articles precisely intend to limit the liability of
the real and hypothecary doctrine of maritime law. the shipowner or agent to the value of the vessel, its
appurtenances and freightage earned in the voyage, provided
that the owner or agent abandons the vessel. When the vessel is
MERCANTILE LAW REVIEW || Insurance Law 56
Compilation of Digests (Except 37)

totally lost in which case there is no vessel to abandon,


abandonment is not required. Because of such total loss the
liability of the shipowner or agent for damages is extinguished.
However, despite the total loss of the vessel, its insurance
answers for the damages for which a shipowner or agent may be
held liable. 

Nonetheless, there are exceptional circumstances wherein the


ship agent could still be held answerable despite the
abandonment of the vessel, as where the loss or injury was due
to the fault of the shipowner and the captain. The international
rule is to the effect that the right of abandonment of vessels, as a
legal limitation of a shipowners liability, does not apply to cases
where the injury or average was occasioned by
the shipowners own fault. Likewise, the shipowner may be held
liable for injuries to passengers notwithstanding the exclusively
real and hypothecary nature of maritime law if fault can be
attributed to the shipowner.
MERCANTILE LAW REVIEW || Insurance Law 57
Compilation of Digests (Except 37)

37. WHITE GOLD MARINE SERVICES, INC. vs. PIONEER the occurrence, contingency, or circumstances under which the
INSURANCE AND SURETY CORPORATION AND THE performance becomes requisite. It is not by what it is called.
STEAMSHIP MUTUAL UNDERWRITING ASSOCIATION Basically, an insurance contract is a contract of
(BERMUDA) LTD indemnity. In it, one undertakes for a consideration to indemnify
G.R. No. 154514 July 28, 2005 J. Quisumbing another against loss, damage or liability arising from an
unknown or contingent event.
A marine insurance undertakes to indemnify the assured against In particular, a marine insurance undertakes to
marine losses, such as the losses incident to a marine adventure. indemnify the assured against marine losses, such as the losses
Section 99 of the Insurance Code enumerates the coverage of incident to a marine adventure.
marine insurance. Relatedly, a mutual insurance company is a cooperative
enterprise where the members are both the insurer and insured.
FACTS: White Gold took a protection and indemnity coverage for In it, the members all contribute, by a system of premiums or
its vessels from Steamship Mutual through Pioneer. White Gold assessments, to the creation of a fund from which all losses and
was issued a Certificate of Entry and Acceptance. When White liabilities are paid, and where the profits are divided among
Gold failed to fully pay its obligation, Steamship Mutual refused themselves, in proportion to their interest.
to renew the coverage extended to them. Steamship Mutual filed A P & I Club is "a form of insurance against third party
a case against White Gold for the unpaid amount. White Gold, liability, where the third party is anyone other than the P & I Club
filed a complaint before the Insurance Commission claiming and the members." By definition then, Steamship Mutual as a P &
Steamship Mutual and Pioneer violated the Insurance Code but it I Club is a mutual insurance association engaged in the marine
was dismissed by the Commission on the ground that Steamship insurance business.
did not need to secure a license because it was not engaged in the The records reveal Steamship Mutual is doing business in
insurance business and that it was a P & I club. On the other the country albeit without the requisite certificate of authority
hand, Pioneer was not required to obtain another license as an mandated by Section 18720 of the Insurance Code. It maintains a
insurance agent because Steamship was not engaged in the resident agent in the Philippines to solicit insurance and to
business of insurance. The CA affirmed the decision of the collect payments in its behalf. We note that Steamship Mutual
insurance Commissioner. even renewed its P & I Club cover until it was cancelled due to
non-payment of the calls. Thus, to continue doing business here,
ISSUE: Whether or not Steamship Mutual is engaged in the Steamship Mutual or through its agent Pioneer, must secure a
business of insurance. license from the Insurance Commission.
Since a contract of insurance involves public interest,
HELD: YES. Steamship Manual is engaged in the business of regulation by the State is necessary. Thus, no insurer or
insurance. insurance company is allowed to engage in the insurance
business without a license or a certificate of authority from the
The test to determine if a contract is an insurance contract or Insurance Commission.
not, depends on the nature of the promise, the act required to be
performed, and the exact nature of the agreement in the light of
MERCANTILE LAW REVIEW || Insurance Law 58
Compilation of Digests (Except 37)

38. FGU INSURANCE CORPORATION vs. THE COURT OF Supervisor, Fernando Macabuag, requested ANCO’s
APPEALS, SAN MIGUEL CORPORATION, and ESTATE OF ANG representative to transfer the barge to a safer place but it
GUI, represented by LUCIO, JULIAN, and JAIME, all surnamed refused so around the midnight, the barge sunk along
ANG, and CO TO6 with 29,210 cases of Pale Pilsen and 500 cases of Cerveza Negra
G.R. No. 137775. March 31, 2005 Chico-Nazario J. totalling to P1,346,197.

It is a basic rule in insurance that the carelessness and negligence When SMC claimed against ANCO it stated that they agreed
of the insured or his agents constitute no defense on the part of the that it would not be liable for any losses or damages resulting to
insurer. This rule however presupposes that the loss has occurred the cargoes by reason of fortuitous event and it was agreed to be
due to causes which could not have been prevented by the insured, insured with FGU for 20,000 cases or P858,500.
despite the exercise of due diligence.
ANCO filed a third party complaint against FGU, alleging that
Facts: before the vessel of ANCO left for San Jose, Antique with the
cargoes owned by SMC, the cargoes, to the extent of Twenty
Anco Enterprises Company (ANCO), is a partnership between Thousand (20,000) cases, were insured with FGU for a total
Ang Gui and Co To and was engaged in the shipping business amount of PhpP858,500.00 under Marine Insurance Policy No.
operating two common carriers M/T ANCO tugboat and D/B 29591. According to ANCO, the loss of said cargoes occurred as a
Lucio barge. D/B Lucio barge has no engine of its own, it could result of risks insured against in the insurance policy and during
not maneuver by itself and had to be towed by a tugboat for it to the existence and lifetime of said insurance policy.
move from one place to another. On September 23 1979, San
Miguel Corporation (SMC) shipped from Mandaue City, Cebu, on FGU admitted the existence of the Insurance Policy under
board the D/B Lucio, for towage by M/T ANCO the following Marine Cover Note No. 29591 but maintained that the alleged
cargoes: loss of the cargoes covered by the said insurance policy cannot
be attributed directly or indirectly to any of the risks insured
 25,000 cases Pale Pilsen and 350 cases Cerveza against in the said insurance policy. According to FGU, it is only
Negra - consignee SMC’s Beer Marketing Division (BMD)- liable under the policy to Third-party Plaintiff ANCO and/or
Estancia Beer Sales Office, Estancia, Iloilo Plaintiff SMC in case of any of the following:
 15,000 cases Pale Pilsen and 200 cases Cerveza
Negra - consignee SMC’s BMD-San Jose Beer Sales Office, San a) total loss of the entire shipment;
Jose, Antique b) loss of any case as a result of the sinking of the vessel; or
c) loss as a result of the vessel being on fire.
On September 30, 1979, D/B Lucio was towed by the M/T
ANCO to San Jose, Antique and immediately left the barge. The Furthermore, FGU alleged that the Third-Party Plaintiff
clouds were dark and the waves were big so SMC’s District Sales ANCO and Plaintiff SMC failed to exercise ordinary diligence or
the diligence of a good father of the family in the care and
6
Same with case number 60
MERCANTILE LAW REVIEW || Insurance Law 59
Compilation of Digests (Except 37)

supervision of the cargoes insured to prevent its loss and/or to navigation and constitute a part of the perils that the insurer is
destruction. obliged to incur, such negligence or recklessness must not be of
such gross character as to amount to misconduct or wrongful
The trial court found that while the cargoes were indeed acts; otherwise, such negligence shall release the insurer from
lost due to fortuitous event, there was failure on ANCO’s part, liability under the insurance contract.
through their representatives, to observe the degree of diligence
required that would exonerate them from liability. The appellate In the instant case, there was blatant negligence on the
court affirmed the decision of the lower court. part of the employees of defendants-appellants when the patron
(operator) of the tug boat immediately left the barge at the San
Issues: Jose, Antique wharf despite the looming bad weather. Negligence
was likewise exhibited by the defendants-appellants’
Whether or not FGU can be held liable under the insurance policy representative who did not heed Macabuag’s request that the
to reimburse ANCO for the loss of the cargoes despite the barge be moved to a more secure place. The prudent thing to do,
findings of the respondent court that such loss was occasioned as was done by the other sea vessels at San Jose, Antique during
by the blatant negligence of the latter’s employees. the time in question, was to transfer the vessel to a safer
wharf. The negligence of the defendants-appellants is proved by
Ruling: the fact that on 01 October 1979, the only simple vessel left at the
wharf in San Jose was the D/B Lucio.
NO. One of the purposes for taking out insurance is to
protect the insured against the consequences of his own Such blatant negligence being the proximate cause of the
negligence and that of his agents. Thus, it is a basic rule in loss of the cargoes amounting to P1,346,197. This Court
insurance that the carelessness and negligence of the insured or concludes that the blatant negligence of ANCO’s employees is of
his agents constitute no defense on the part of the insurer. This such gross character that it amounts to a wrongful act which
rule however presupposes that the loss has occurred due to must exonerate FGU from liability under the insurance contract.
causes which could not have been prevented by the insured,
despite the exercise of due diligence. When evidence show that
the insured’s negligence or recklessness is so gross as to be
sufficient to constitute a willful act, the insurer must be
exonerated.

The United States Supreme Court has made a distinction


between ordinary negligence and gross negligence or negligence
amounting to misconduct and its effect on the insured’s right to
recover under the insurance contract. According to the Court,
while mistake and negligence of the master or crew are incident
MERCANTILE LAW REVIEW || Insurance Law 60
Compilation of Digests (Except 37)

39. The Philippine American General Insurance Company, about 7 a.m., the master of the vessel stopped the engine because
Inc. vs. Court of Appeals and Felman Shipping Lines the vessel was listing dangerously to portside. He ordered his
G.R. No. 116940 June 11, 1997 J. Bellosillo crew to shift the cargo back to starboard (right). The shifting of
cargo took about an hour after which he rang the engine room to
If the policy provides that the seaworthiness of the vessel as resume full speed.
between insured and insurer is admitted, the issue of
seaworthiness cannot be raised by the insurer without showing At around 8:45 a.m., the vessel suddenly listed to
concealment or misrepresentation by the insured. It may mean: portside and before the captain could decide on his next move,
1. That the warranty of seaworthiness is to be taken as some of the cargo on deck were thrown overboard and seawater
fulfilled; or  entered the engine room and cargo holds of the vessel. At that
2. That the risk of unseaworthiness is assumed by the insurer. instance, the master of the vessel ordered his crew to abandon
ship.
FACTS
Shortly thereafter, "MV Asilda" capsized and sank in the
On 6 July 1983 Coca-Cola Bottlers Philippines, Inc. (Coca- waters of Zamboanga del Norte bringing down her entire cargo
Cola Bottlers), loaded on board "MV Asilda" a vessel owned and with her including the subject 7,500 cases of 1-liter Coca-Cola
operated by respondent Felman Shipping Lines (FELMAN), 7,500 softdrink bottles. The Ship Captain ascribed the sinking to the
cases of 1-liter Coca-Cola softdrink bottles to be transported entry of seawater through a hole in the hull caused by the
from Zamboanga City to Cebu City for consignee Coca-Cola vessel's collision with a partially submerged log.
Bottlers Philippines, Inc., Cebu. The shipment was insured with
petitioner Philippine American General Insurance Co., Inc. On 15 July 1983 the consignee Coca-Cola Bottlers
(PHILAMGEN) Philippines, Inc., Cebu plant, filed a claim with respondent
FELMAN for recovery of damages. FELMAN denied the claim thus
In a joint statement, the Captain as well as the chief mate prompting the consignee to file an insurance claim with
of the vessel confirmed that the weather was fine when "MV PHILAMGEN which paid its claim of P755,250.00.
Asilda" left the port of Zamboanga at 8 p.m. on 6 July. The ship
captain stated that around 4 a.m. of 7 July 1983 he was Claiming its right of subrogation PHILAMGEN sought
awakened by the officer on duty to inform him that the vessel recourse against respondent FELMAN which disclaimed any
had hit a floating log. liability for the loss. Consequently, on 29 November 1983
PHILAMGEN sued the shipowner for sum of money and damages,
At that time, he noticed that the weather had alleging that the total loss of cargo was due to the vessel’s
deteriorated with strong southeast winds inducing big waves. unseaworthiness as she was put to sea in an unstable condition.
After 30 minutes, he observed that the vessel was listing slightly FELMAN, on the other hand, filed a motion to dismiss contending
to starboard and would not correct itself despite the heavy that there was no right of subrogation in favor of PHILAMGEN
rolling and pitching. He then ordered his crew to shift the cargo since it had abandoned all its rights, interests and ownership
from starboard to portside until the vessel was balanced. At over the vessel together with her freight and appurtenances for
MERCANTILE LAW REVIEW || Insurance Law 61
Compilation of Digests (Except 37)

the purpose of limiting and extinguishing its liability under Art. international rule is to the effect that the right of abandonment of
587 of the Code of Commerce. vessels, as a legal limitation of a ship owner's liability, does not
apply to cases where the injury or average was occasioned by the
The RTC dismissed PHILAMGEN’s complaint and ship owner's own fault. It must be stressed at this point that Art.
appealed to the CA which remanded the case and denied its 587 speaks only of situations where the fault or negligence is
motion for reconsideration. The RTC then ruled that the vessel committed solely by the captain. Where the ship owner is
was seaworthy and even if assumed unseaworthy, PHILAMGEN likewise to be blamed, Art. 587 will not apply, and such situation
still could not recover from FELMAN since Coca-Cola Bottlers will be covered by the provisions of the Civil Code on common
had breached its implied warranty on the vessel’s seaworthiness. carrier. Under Art 1733 of the Civil Code, "(c)ommon carriers,
from the nature of their business and for reasons of public policy,
On appeal, the CA ruled that the vessel was unseaworthy are bound to observe extraordinary diligence in the vigilance
for being topheavy as 2,500 cases of Coca-Cola softdrinks bottles over the goods and for the safety of the passengers transported
were improperly stowed on deck. Even though the vessel by them, according to all the circumstances of each case . . ." In
possessed the necessary Coast Guard certification indicating its the event of loss of goods, common carriers are presumed to
seaworthiness with respect to the structure of the ship itself, it have acted negligently. FELMAN, the ship owner, was not able to
was not seaworthy with respect to the cargo. However, it denied rebut this presumption.
the money claim of PHILAMGEN because of the implied breach of
warranty of seaworthiness by Coca-Cola Bottlers. Furthermore,
the filing of notice of abandonment had absolved FELMAN from
liability under the limited liability rule.

ISSUE

Whether or not the limited liability under Article 587 of


the Code of Commerce should apply

HELD

NO. The Supreme Court held that Article 587 of the Code
of Commerce is not applicable. The ship agent is liable for the
negligent acts of the captain in the care of the goods loaded on
the vessel. This liability, although can be limited through
abandonment of the vessel, its equipment and freightage, as
provided in Art. 587, there exceptional circumstances wherein
the ship agent could still be held answerable, as where the loss or
injury was due to the fault of the ship owner and the captain. The
MERCANTILE LAW REVIEW || Insurance Law 62
Compilation of Digests (Except 37)

40. CENTRAL SHIPPING COMPANY, INC. vs. INSURANCE ISSUES


COMPANY OF NORTH AMERICA
G.R. No. 150751, September 20, 2004, J. PANGANIBAN 1. Whether or not the carrier is liable for the loss of the cargo.
2. Whether or not the doctrine of limited liability is applicable.
Common carriers are presumed to have been at fault or to
have acted negligently, unless they prove that they observed RULING
extraordinary diligence.
1. YES. In the present case, petitioner has not given the Court
FACTS sufficient cogent reasons to disturb the conclusion of the CA
that the weather encountered by the vessel was not a “storm”
On July 25,1990 at Puerto Princesa, Palawan, the as contemplated by Article 1734(1). The evidence indicated
petitioner received on board its vessel, the M/V “Central Bohol” that strong southwest monsoons were common occurrence
376 pcs. of Apitong Round Logs and undertook to transport them during the month of July. Thus, the officers and crew of the
to Manila for delivery to Alaska Lumber Co., Inc. The cargo was vessel should have reasonably anticipated heavy rains,
insured by Insurance Company of North America for strong winds and rough seas. They should then have taken
P3,000,000.00 against total loss. extra precaution in stowing the logs in the hold, in
consonance with their duty of observing extraordinary
Upon completion of loading of the cargo, the vessel diligence in safeguarding the goods. But the carrier took a
commenced the voyage to Manila. On July 26,1990 at about 1:28 calculated risk in improperly securing the cargo. Having lost
am, while enroute to Manila, the vessel turned 15 degrees that risk, it cannot now escape responsibility for the loss.
sideward, the ship captain ordered his men to abandon the ship
and at exactly 1:30 am of the same day, the vessel completely 2. NO. The doctrine of limited liability under Art. 587 of the
sank and the cargo was totally lost. Code of Commerce is not applicable to the present case. This
rule does not apply to situations in which the loss or the
As a consequence, the consignee (Alaska Lumber) presented injury is due to the concurrent negligence of the shipowner
a claim for the value of the shipment to the petitioner but the and the captain. It has already been established that the
latter failed and refused to settle the claim. The insurer paid said sinking of the vessel had been caused by the fault or
claim and now seeks to be subrogated to all the rights and negligence of the ship captain and the crews as shown by the
actions of the consignee against the petitioner. improper stowage of the cargo logs. A closer supervision on
the part of the shipowner could have prevented this fatal
The Trial Court ruled in favor of the insurer and ordered the miscalculation. The shipowner cannot escape liability by
petitioner to pay P3,000,000.00 to the insurer. On appeal , the virtue of the limited liability rule.
appellate court affirmed the ruling of the trial court.
MERCANTILE LAW REVIEW || Insurance Law 63
Compilation of Digests (Except 37)

ON PRESENTATION OF POLICIES merely a complementary or supplementary document to the


contract of insurance that may have existed as between Malayan
41. MALAYAN INSURANCE CO., INC. vs. REGIS BROKERAGE and ABB Koppel. And while this observation may deviate from
CORP., the tenor of the assailed CA decisions, it does not presage any
GR NO.172156, November 23, 2007 ruling in favour of petitioner. Fundamentally, since Malayan
failed to introduced in evidence the Marine Insurance Policy
itself as the main insurance contract, or even advert to said
FACTS:
document in the complaint, ultimately then it failed to establish
A 120 pieces of motors on board China Airlines bound for Manila its cause of action for restitution as a subrogee for ABB Koppel.
from US was to be delivered to ABB Koppel. When the cargo
Malayan’s right of recovery as a subrogee of ABB Koppel cannot
arrive at NAIA, it was forwarded to Paircargo for temporary
be predicated alone on the liability of the respondent to ABB
storage until March 7, 1995. REGIS withdraw the cargo and
Koppel eventhough such liabity will necessarily have to be
delivered to ABB KOPPEL, however, the latter discovered that
established at the trial for Malayan to recover. Because Malayan’s
only 65 were actually delivered. The shipment was insured with
right to recovery derives from contractual subrogation as an
MALAYAN by ABB KOPPEL. Demand was made first upon REGIS
incident to an insurance relationship, and not from any
but the latter refused to pay, thus, MALAYAN paid by ABB
proximate injury to it inflicted by respondents, it is critical that
KOPPEL pursuant to its insurance agreement, and MALAYAN
Malayan establish the legal basis of such right to subrogation by
was on that basis subrogated to the rights of ABB KOPPEL
presenting the contract constitutive of the insurance relationship
against REGIS and Paircargo. On June 1986 MALAYAN filed a
between it and ABB Koppel. Without such legal basis, its casuse
complaint for damages against REGIS and Paircargo with the
of action cannot survive.
MeTC, and presented Marine Risk Note as proof that the cargo
was insured by MALAYAN. MeTC found REGIS alone liable to
MALAYAN and it was affirmed by the RTC. However, on appeal
by REGIS to the CA, the latter reversed the decision of the RTC
finding that the Marine Risk Note was invalid.
ISSUE: Whether or not the evidence presented by Malayan
specifically the Marine Risk Note is valid as a basis on their
subrogation rights.
RULING:
NO. What the Marine Risk Note bears, as a matter of evidence, is
that it is not apparently the contract of insurance by itself, but
MERCANTILE LAW REVIEW || Insurance Law 64
Compilation of Digests (Except 37)

42. INTERNATIONAL CONTAINER TERMINAL SERVICES, INC. ISSUE: Whether or not the case should be dismissed on the
vs. FGU INSURANCE CORPORATION ground that respondent’s failure to offer as evidence the
G.R. No. 161539, June 27, 2008, AUSTRIA-MARTINEZ, J. insurance policy.

The presentation of the insurance policy was not fatal because the RULING
loss of the cargo undoubtedly occurred while on board the petitioner's
vessel (Delsan Transport Lines, Inc. v. Court of Appeals) unlike when the NO. Jurisprudence has it that the marine insurance policy
cargo passed through several stages with different parties and it could not needs to be presented in evidence before the trial court or even
be determined when the damage to the cargo occurred, such that the belatedly before the appellate court. Jurisprudence dictates that
insurer should be liable for it. the presentation of the marine insurance policy was necessary,
as the issues raised therein arose from the very existence of an
FACTS: Petitioner's liability arose from a lost shipment of insurance contract between the insurance company and its
400kgs. Silver Nitrate shipped by Hapag-Lloyd AG through the consignee even prior to the loss of the shipment. (Malayan
vessel Hannover Express from Hamburg, Germany with Manila, Insurance Co., Inc. v. Regis Brokerage Corp)
Philippines as the port of discharge. Said shipment was insured
by FGU Insurance Corporation (FGU). Also the court already ruled that the insurance contract must be
presented in evidence in order to determine the extent of the
When RAGC's customs broker was claiming the shipment, coverage. (Wallem Philippines Shipping, Inc. v. Prudential
petitioner, could not find it in its storage area. After investigation Guarantee and Assurance, Inc. & Home Insurance Corporation v.
by the NBI and The AAREMA Marine and Cargo Surveyors, Inc. Court of Appeals)
Both found that the shipment was lost while in the custody and
responsibility of petitioner. However, as in every general rule, there are admitted exceptions.

As insurer, FGU paid RAGC . In turn, FGU sought reimbursement The Court stated that the presentation of the insurance policy
from petitioner, but the latter refused. This constrained FGU to was not fatal because the loss of the cargo undoubtedly occurred
file with the RTC of Manila Civil Case for the collection of sum of while on board the petitioner's vessel (Delsan Transport Lines,
money. Inc. v. Court of Appeals), unlike in Home Insurance Case in which
the cargo passed through several stages with different parties
The RTC rendered its Decision finding petitioner liable. and it could not be determined when the damage to the cargo
Petitioner appealed to the Court of Appeals which only affirmed occurred, such that the insurer should be liable for it.
the RTC Decision. Petitioner filed a motion for reconsideration
averring that the case should be dismissed on the ground that As in Delsan case, there is no doubt that the loss of the cargo in
respondent failed to offer the insurance policy in evidence. the present case occurred while in petitioner's custody.
MERCANTILE LAW REVIEW || Insurance Law 65
Compilation of Digests (Except 37)

SUBROGATION extinguishers and buckets of water. Moreover, life jackets and


construction materials of the Deck B ceiling were combustible
43. KEPPEL CEBU SHIPYARD, INC. v. PIONEER INSURANCE that permitted the fire to spread within the ceiling void. There
AND SURETY CORPORATION were also cans of paint and thinner, in addition to plywood
G.R. No. 180880-81 & 180896-97, September 25, 2009, J. partitions and foam mattresses on Deck B.
NACHURA
WG&A declared the vessel’s damage as a “total constructive loss”
Subrogation is the substitution of one person by another with
and, hence, filed an insurance claim with Pioneer. The latter paid
reference to a lawful claim or right, so that he who is substituted
the Vessel $8,472,581.78 equivalent to P360M. WG&A executed a
succeeds to the rights of the other in relation to a debt or claim,
Loss and Subrogation Receipt in favor of Pioneer. Pioneer tried
including its remedies or securities.
to collect from KCSI by virtue of such receipt but the latter
The right of subrogation is not dependent upon, nor does it grow denied any responsibility despite repeated demands.
out of, any privity of contract. It accrues simply upon payment by
Pioneer filed a Request for Arbitration before the Construction
the insurance company of the insurance claim. The doctrine of
Industry Arbitration Commission (CIAC). It declared both WG&A
subrogation has its roots in equity. It is designed to promote and to
and KCSI negligent thereby ordering KCSI to pay Pioneer P25M
accomplish justice; and is the mode that equity adopts to compel
with interest and arbitration costs. Upon appeal, the CA
the ultimate payment of a debt by one who, in justice, equity, and
dismissed it. When Pioneer and KCSI sought reconsideration and
good conscience, ought to pay.
partial reconsideration, the Former Fifth Division of the CA
The Loss and Subrogation Receipt issued by the insured to the partially granted the petition ordering KCSI to pay Pioneer for
insurer is the best evidence of payment of the insurance proceeds P25M without legal interest.
to the insured.
Pioneer’s argument before the CIAC was that KCSI was solely
FACTS: Keppel Cebu Shipyard, Inc. (KCSI) and WG&A Jebsens responsible for the loss because its employee, Sevillejo, at the
Shipmanagement, Inc. (WG&A) executed a Ship Repair time the fire broke out, was doing his assigned task as to the hot
Agreement wherein KCSI (a.k.a . the Yard) would renovate and works done on board the vessel. KCSI claims otherwise, stating
reconstruct WG&A’s (a.k.a. the Vessel) M/V “Superferry 3” using that the hot work done was beyond the scope of Sevillejo’s
its dry docking facilities in Lapu-Lapu City, Cebu. Further, KCSI’s assigned tasks, i.e. not having being authorized under the Work
liability thereunder is P50M. Prior to the agreement, the ferry Order or under the Ship Repair Agreement. KCSI further posits
was already insured with Pioneer Insurance and Surety that WG&A was also negligent through its crew, Dr. Raymundo
Corportion (Pioneer) for $8,472,581.72. Joniga (Dr. Joniga), for failing to remove the life jackets from the
ceiling void, causing the immediate spread of the fire to the other
In the course of the repair, the ferry was gutted by fire allegedly areas of the ship.
caused by the welder, Severino Sevillejo (Sevillejo), who is KCSI’s
employee, who did hot works (i.e. welding) on Deck A. The fire Pioneer argues that there is total constructive loss so that it had
was inside the ceiling void of Deck B therefore it was extremely to pay WG&A the full amount of the insurance coverage and, by
difficult to contain or extinguish it despite the use of fire operation of law, it was entitled to be subrogated to the rights of
MERCANTILE LAW REVIEW || Insurance Law 66
Compilation of Digests (Except 37)

WG&A to claim the amount of the loss. It further contends that wrongdoer or the person who has violated the contract.
the limitation of liability clause in the Ship Repair Agreement is If the amount paid by the insurance company does not
null and void for being iniquitous and against public policy. fully cover the injury or loss, the aggrieved party shall be
entitled to recover the deficiency from the person
KCSI counters that a total constructive loss was not adequately causing the loss or injury.
proven by Pioneer, and that there is no proof of payment of the
insurance proceeds. KCSI insists on the validity of the limited- Subrogation is the substitution of one person by another with
liability clause up to P50M, because WG&A assented to the reference to a lawful claim or right, so that he who is substituted
provision when it executed the Ship Repair Agreement. KCSI also succeeds to the rights of the other in relation to a debt or claim,
claims that the salvage value of the vessel should be deducted including its remedies or securities. The principle covers a
from whatever amount it will be made to pay Pioneer. situation wherein an insurer has paid a loss under an insurance
ISSUE: Is subrogation proper? If proper, to what extend can policy is entitled to all the rights and remedies belonging to the
subrogation be made? insured against a third party with respect to any loss covered by
the policy. It contemplates full substitution such that it places the
RULING: YES. Subrogation is proper. Considering the extent of party subrogated in the shoes of the creditor, and he may use all
the damage, WG&A opted to abandon the ship and claimed the means that the creditor could employ to enforce payment.
value of its policies. Pioneer, finding the claim compensable, paid
the claim, with WG&A issuing a Loss and Subrogation Receipt We have held that payment by the insurer to the insured
evidencing receipt of the payment of the insurance proceeds operates as an equitable assignment to the insurer of all the
from Pioneer. On this note, we find as unacceptable the claim of remedies that the insured may have against the third party
KCSI that there was no ample proof of payment simply because whose negligence or wrongful act caused the loss. The right of
the person who signed the Receipt appeared to be an employee subrogation is not dependent upon, nor does it grow out of, any
of Aboitiz Shipping Corporation. The Loss and Subrogation privity of contract. It accrues simply upon payment by the
Receipt issued by WG&A to Pioneer is the best evidence of insurance company of the insurance claim. The doctrine of
payment of the insurance proceeds to the former, and no subrogation has its roots in equity. It is designed to promote and
controverting evidence was presented by KCSI to rebut the to accomplish justice; and is the mode that equity adopts to
presumed authority of the signatory to receive such payment. compel the ultimate payment of a debt by one who, in justice,
equity, and good conscience, ought to pay.
On the matter of subrogation, Article 2207 of the Civil Code
provides—
Art. 2207. If the plaintiff’s property has been insured and
he has received indemnity from the insurance company
for the injury or loss arising out of the wrong or breach of
contract complained of, the insurance company shall be
subrogated to the rights of the insured against the
MERCANTILE LAW REVIEW || Insurance Law 67
Compilation of Digests (Except 37)

44. Malayan Insurance Co vs Alberto all the remedies that the insured may have against the third
GR No. 194320, February 1, 2012, J. Velasco party whose negligence or wrongful act caused the loss. The
right of subrogation is not dependent upon, nor does it grow out
FACTS of, any privity ofcontract. It accrues simply upon payment by the
insurance company of the insurance claim. The doctrine of
A Mitsubishi Galant was insured by Malayan Insurance subrogation has its roots in equity. It is designed to promote
against third party liability, own damage and theft. The and to accomplish justice; and is the mode that equity adopts to
Mitsubishi Galant got into an accident with three other vehicles compel the ultimate payment of a debt by one who, in justice,
(Nissan Bus, Isuzu Tanker and Fuzo Cargo Truck). Malayan equity, and good conscience, ought to pay.
Insurance paid the owner of the Mitsubishi Galant for the
damages its vehicle suffered. Maintaining that it has been
subrogated to the rights and interests of the assured by
operation of law upon its payment to the latter,
Malayan Insurance sent several demand letters to the owner and
driver of the Fuzo Cargo Truck to Malayan Insurance nut they
refused. Malayan Insurance filed a complaint for damages.

ISSUE

Whether or Not Malayan Insurance can be subrogated to


the rights of the owner of the Mitsubishi Galant

HELD

YES. Subrogation is the substitution of one person by


another with reference to a lawful claim or right, so that he who
is substituted succeeds to the rights of the other in relation to a
debt or claim, including its remedies or securities. The principle
covers a situation wherein an insurer has paid a loss under an
insurance policy is entitled to all the rights and remedies
belonging to the insured against a third party with respect to any
loss covered by the policy. It contemplates full substitution such
that it places the party subrogated in the shoes of the creditor,
and he may use all means that the creditor could employ to
enforce payment. We have held that payment by the insurer to
the insured operates as an equitable assignment to the insurer of
MERCANTILE LAW REVIEW || Insurance Law 68
Compilation of Digests (Except 37)

FIRE INSURANCE ISSUE:

45. UY HO & CO. VS PRUDENTIAL ASSURANCE CO., LTD., G.R. Won evidence for claiming would bar the recovery of issurance
No. 27778, December 16, 1927 proceeds?

HELD:
Where a fire insurance policy provides that "If the claim be in any
respect fraudulent, or if any false declaration be made or used in Yes. Where a fire insurance policy provides that "If the claim be
support thereof, or if any fraudulent means or devices are used by in any respect fraudulent, or if any false declaration be made or
the Insured or anyone acting on his behalf to obtain any benefit used in support thereof, or if any fraudulent means or devices
under this Policy," and the evidence is conclusive that the proof of are used by the Insured or anyone acting on his behalf to obtain
claim which the insured submitted was false and fraudulent both any benefit under this Policy," and the evidence is conclusive that
as to the kind, quality and amount of the goods and their value the proof of claim which the insured submitted was false and
destroyed by the fire, such a proof of claim is a bar against the fraudulent both as to the kind, quality and amount of the goods
insured to recover on the policy even for the amount of his actual and their value destroyed by the fire, such a proof of claim is a
loss. bar against the insured to recover on the policy even for the
amount of his actual loss.
FACTS:

Uy Ho & Co entered into an insurance contract with the


Prudential Assurance against loss or destruction of its
merchadise by fire. A fire broke out engulfing its store and
destroying their alleged merchardise. For this, Uy Ho claimed to
be indemnified by virtue of the issurance policy from the
Prudential Assurance with list of merchandise desteoyed by fire
as evidence. However, the latyer refuse to indemnify the former
on the contwntion that the evidence sumbitted or the claim is
fraudulent invoking one of the provisions in the policy that
states, any false or fraudulent declaration, or claims, or use of
fraudulent means and devices to claim payment for the loss shall
declare the policy null and void. Dismayed, the Uy Ho filed a
cased against the Prudential. The trial court held infavor of the
petitioner, however, it reduced the amount from 30k to 16k. MR
was filed bybthe respondent but was seasonably denied. Hence,
this petition.
MERCANTILE LAW REVIEW || Insurance Law 69
Compilation of Digests (Except 37)

46. MALAYAN INSURANCE COMPANY, INC. v. PAP CO., LTD.


G.R. No. 200784, J. Mendoza ISSUE:

FACTS Is Malayan liable under the insurance contract?

On May 13, 1996, Malayan Insurance Company HELD


(Malayan) issued Fire Insurance Policy to PAP Co., Ltd. (PAP Co.)
for the latter’s machineries and equipment located at Sanyo NO. Under the policy and when it was renewed, it
Precision Philsin Rosario, Cavite (Sanyo Building). The insurance, forbade the removal of the insured properties unless sanctioned
which was for Fifteen Million Pesos and effective for a period of or consented by Malayan. PAP failed to notify and to obtain
one (1) year, was procured by PAP Co. for Rizal Commercial consent of Malayan regarding the removal. The transfer also
Banking Corporation (RCBC), the mortgagee of the insured increased the risk. With the transfer of location of the subject
machineries and equipment. properties, without notice to and consent of Malayan, PAP
committed concealment, misrepresentation and breach of material
After the passage of almost a year but prior to the warranty. Under Insurance Code, Malayan can rescind the
expiration of the insurance coverage, PAP Co. renewed the policy insurance contract.
on an "as is" basis. Pursuant thereto, a renewal policy, Fire
Insurance Policy was issued by Malayan to PAP Co. for the period
May 13, 1997 to May 13, 1998.

On October 12, 1997 and during the subsistence of the


renewal policy, the insured machineries and equipment were
totally lost by fire. Hence, PAP Co. filed a fire insurance claim
with Malayan in the amount insured.

In a letter, dated December 15, 1997, Malayan denied the


claim upon the ground that, at the time of the loss, the insured
machineries and equipment were transferred by PAP Co. to a
location different from that indicated in the policy. Specifically,
that the insured machineries were transferred in September
1996 from the Sanyo Building to the Pace Pacific Bldg in Rosario,
Cavite (Pace Pacific). Contesting the denial, PAP Co. argued that
Malayan cannot avoid liability as it was informed of the transfer
by RCBC, the party duty-bound to relay such information.
However, Malayan reiterated its denial of PAP Co.’s claim.
Distraught, PAP Co. filed the complaint below against Malayan.
MERCANTILE LAW REVIEW || Insurance Law 70
Compilation of Digests (Except 37)

47. UNITED MERCHANTS CORPORATION vs COUNTRY On 3 July 1996, a fire gutted the warehouse rented by
BANKERS INSURANCE CORPORATION UMC. CBIC designated CRM Adjustment Corporation (CRM) to
G.R. No. 198588, July 11, 2012, CARPIO, J. investigate and evaluate UMCs loss by reason of the fire. CBICs
reinsurer, Central Surety, likewise requested the National
Burden of proof is the duty of any party to present Bureau of Investigation (NBI) to conduct a parallel investigation.
evidence to establish his claim or defense by the amount of On 6 July 1996, UMC, through CRM, submitted to CBIC its Sworn
evidence required by law, which is preponderance of evidence in Statement of Formal Claim, with proofs of its loss. On 20
civil cases. The party, whether plaintiff or defendant, who asserts November 1996, UMC demanded for at least fifty percent (50%)
the affirmative of the issue has the burden of proof to obtain a payment of its claim from CBIC. On 25 February 1997, UMC
favorable judgment. Particularly, in insurance cases, once an received CBICs letter, dated 10 January 1997, rejecting UMCs
insured makes out a prima facie case in its favor, the burden of claim due to breach of Condition No. 15 of the Insurance Policy.
evidence shifts to the insurer to controvert the insureds prima Condition No. 15 states:
facie case.; The most liberal human judgment cannot attribute  
such difference to mere innocent error in estimating or counting If the claim be in any respect fraudulent, or if any
but to a deliberate intent to demand from insurance companies false declaration be made or used in support
payment for indemnity of goods not existing at the time of the thereof, or if any fraudulent means or devices are
fire. This constitutes the so-called fraudulent claim which, by used by the Insured or anyone acting in his behalf
express agreement between the insurers and the insured, is a to obtain any benefit under this Policy; or if the
ground for the exemption of insurers from civil liability. loss or damage be occasioned by the willful act,
or with the connivance of the Insured, all the
FACTS benefits under this Policy shall be forfeited.

Petitioner United Merchants Corporation (UMC) is ISSUE


engaged in the business of buying, selling, and manufacturing
Christmas lights. UMC leased a warehouse at 19-B Dagot Street, Wheter UMC is entitled to claim from CBIC the full
San Jose Subdivision, Barrio Manresa, Quezon City, where UMC coverage of its fire insurance policy.
assembled and stored its products.
  RULING
On 6 September 1995, UMCs General Manager Alfredo
Tan insured UMCs stocks in trade of Christmas lights against fire The Court DENIES the claim of the insurer based on
with defendant Country Bankers Insurance Corporation (CBIC) fraud.
for P15,000,000.00. The Fire Insurance Policy No. F-HO/95-576
(Insurance Policy) and Fire Invoice No. 12959A, valid until 6 Particularly, in insurance cases, once an insured makes
September 1996 out a prima facie case in its favor, the burden of evidence shifts to
the insurer to controvert the insureds prima facie case. In the
present case, UMC established a prima facie case against CBIC.
MERCANTILE LAW REVIEW || Insurance Law 71
Compilation of Digests (Except 37)

CBIC does not dispute that UMCs stocks in trade were insured designated CRM to evaluate UMCs loss. Third, the Certification by
against fire under the Insurance Policy and that the warehouse, the Bureau of Fire Protection states that the fire was accidental
where UMCs stocks in trade were stored, was gutted by fire on 3 in origin. This Certification enjoys the presumption of regularity,
July 1996, within the duration of the fire insurance. However, which CBIC failed to rebut.
since CBIC alleged an excepted risk, then the burden of evidence
shifted to CBIC to prove such exception. However, in the present case, arson and fraud are two
  separate grounds based on two different sets of evidence, either
An insurer who seeks to defeat a claim because of an of which can void the insurance claim of UMC. The absence of
exception or limitation in the policy has the burden of one does not necessarily result in the absence of the other.
establishing that the loss comes within the purview of the
exception or limitation.[23] If loss is proved apparently within a Thus, on the allegation of fraud, we affirm the findings of
contract of insurance, the burden is upon the insurer to establish the Court of Appeals. The invoices, however, cannot be taken as
that the loss arose from a cause of loss which is excepted or for genuine. The invoices reveal that the stocks in trade purchased
which it is not liable, or from a cause which limits its liability. In for 1996 amounts to P20,000,000.00 which were purchased in
the present case, CBIC failed to discharge its primordial burden one month. Thus, UMC needs to prove purchases amounting
of establishing that the damage or loss was caused by arson, a to P30,000,000.00 worth of stocks in trade for 1995 and prior
limitation in the policy. years.
 
In prosecutions for arson, proof of the crime charged is While it is a cardinal principle of insurance law that a
complete where the evidence establishes: (1) the corpus delicti, contract of insurance is to be construed liberally in favor of the
that is, a fire caused by a criminal act; and (2) the identity of the insured and strictly against the insurer company, contracts of
defendants as the one responsible for the crime. insurance, like other contracts, are to be construed according to
  the sense and meaning of the terms which the parties themselves
In the present case, CBICs evidence did not prove that the have used. If such terms are clear and unambiguous, they must
fire was intentionally caused by the insured. First, the findings of be taken and understood in their plain, ordinary and popular
CBICs witnesses, Cabrera and Lazaro, were based on an sense. Courts are not permitted to make contracts for the parties;
investigation conducted more than four months after the fire. the function and duty of the courts is simply to enforce and carry
The testimonies of Cabrera and Lazaro, as to the boxes doused out the contracts actually made.
with kerosene as told to them by barangay officials, are hearsay
because the barangay officials were not presented in court.
Cabrera and Lazaro even admitted that they did not conduct a
forensic investigation of the warehouse nor did they file a case
for arson.[28] Second, the Sworn Statement of Formal Claim
submitted by UMC, through CRM, states that the cause of the fire
was faulty electrical wiring/accidental in nature. CBIC is bound
by this evidence because in its Answer, it admitted that it
MERCANTILE LAW REVIEW || Insurance Law 72
Compilation of Digests (Except 37)

SURETY The Court of Appeals was not convinced with petitioner's


contentions; it affirmed the decision of the trial court.
48. Paramount Insurance Corporation v. Court of Appeals
310 SCRA 377 In the instant petition, PARAMOUNT seeks to reverse and
set aside the decision of the Court of Appeals.
It may not be amiss to point out that by the contract of
suretyship, it is not for the obligee to see to it that the principal ISSUE
pays the debt or fulfills the contract, but for the surety to see to it
that the principal pay or perform. Whether or not there was a sufficient evidence to
establish the liability of the petitioner on its injunction bond.
FACTS
RULING
McAdore Finance and Investment, Inc. (McADORE) was
the owner and operator of the McAdore International Palace NO. The petition is devoid of merit.
Hotel. McADORE and DECORP entered into a contract whereby
DECORP shall provide electric power to McADORE's Hotel. Injunction is an extraordinary remedy calculated to
DECORP noticed discrepancies between the actual monthly preserve the status quo of things  and to prevent actual or
billings and the estimated monthly billings of McADORE. It was threatened acts violative of the rules of equity and good
discovered that the terminal in the transformers connected to conscience as would consequently afford an injured party a
the meter had been interchanged resulting in the slow rotation of cause of action resulting from the failure of the law to provide for
the meter. DECORP issued a corrected bill but McADORE refused an adequate or complete relief. Its sole purpose is not to correct
to pay. As a result, DECORP disconnected power supply to the a wrong of the past, in the sense of redress for injury already
hotel. sustained, but to prevent further injury.

McADORE commenced a suit against DECORP for A preliminary injunction or temporary restraining order
damages with prayer for a writ of preliminary injunction. may be granted only when the applicant files with the court
McADORE posted injunction bonds from several sureties, one of where the action or proceeding is pending a bond executed to
which was herein petitioner PARAMOUNT. the party or person enjoined to the effect that the applicant will
pay such party or person all damages which he may sustain by
The Regional Trial Court rendered judgment in favor of reason of the injunction or temporary restraining order if the
DECORP. McADORE did not appeal. PARAMOUNT, however, court should finally decide that the applicant was not entitled
appealed to the Court of Appeals. PARAMOUNT contended that it thereto.
was not given its day in court because it was not notified by
DECORP of its intention to present evidence of damages against In order for the injunction bond to become answerable
its injunction bond. for the above-described damages, the following requisites must
concur:
MERCANTILE LAW REVIEW || Insurance Law 73
Compilation of Digests (Except 37)

1. The application for damages must be filed in the question of its solidary liability for damages arising from a
same case where the bond was issued; wrongful injunction order. Withal, the fact that the matter of
2. Such application for damages must be filed before damages was among the issues tried during the hearings on the
the entry of judgment; and merits will not render unnecessary or superfluous a summary
3. After hearing with notice to the surety. hearing to determine the extent of a surety's liability unless: (1)
the surety had been impleaded as a party, or (2) otherwise earlier
DECORP filed its Answer raising compulsory notified and given opportunity to be present and ventilate its side
counterclaims for rescission of contract, moral damages, on the matter during the trial.
exemplary damages, attorney's fees and litigation expenses.
During the trial, Atty. Nonito Cordero appearedas counsel for What is necessary only is for the attaching party and his
petitioner. PARAMOUNT as well as the other sureties were surety or sureties to be duly notified and given the opportunity
properly notified of the hearing and given their day in court. to be heard. In the case at bench, this Court accords due respect
Specifically, notice was sent to Atty. Cordero of the hearing on to the factual finding of the Court of Appeals that PARAMOUNT
April 27, 1985, which was set for the purpose of determining the was present and represented by its counsel.
liability of the sureties. The counterclaims for damages of
DECORP were proven at the trial and yet PARAMOUNT did not PARAMOUNT also argues that assuming it is liable on its
exert any effort to controvert the evidence presented. injunction bond, its liability should be limited only to the amount
PARAMOUNT cannot hide under the cloak of non-liability on its of damages accruing from the time the injunction bond was
injunction bond on the mere expediency that it was deprived of issued until the termination of the case, and not from the time
due process. the suit was commenced. In short, it claims that the injunction
bond is prospective and not retroactive in application.
PARAMOUNT argues that contrary to the ruling of the PARAMOUNT further maintains that it is liable to pay actual
Court of Appeals, there is a need for a separate hearing for the damages only.
purpose of presenting evidence on the alleged damages claimed
by DECORP on petitioner's injunction bond. This Court does not agree.

Contrary to petitioner's thesis, it is neither mandatory Rule 58, Section 4(b), provides that a bond is executed in
nor fatal that there should be a separate hearing in order that favor of the party enjoined to answer for all damages which he
damages upon the bond can be claimed, ascertained and may sustain by reason of the injunction .No distinction was made
awarded, as can be gleaned from a cursory reading of the as to when the damages should have been incurred.
provisions of Rule 57, Section 20. PARAMOUNT is liable, jointly and severally, for actual damages,
moral damages, exemplary damages, attorney's fees and costs of
A final adjudication that the applicant is not entitled to the suit, to the extent of the amount of the bond.
the injunction does not suffice to make the surety liable. It is
necessary, in addition, that the surety be accorded due process,
that is, that it be given an opportunity to be heard on the
MERCANTILE LAW REVIEW || Insurance Law 74
Compilation of Digests (Except 37)

49. SPOUSES NOE and CLARITA QUIAMCO (Petitioners) vs. A writ of execution was served by the sheriff of the NLRC on
CAPITAL INSURANCE & SURETY CO., INC. (Respondent) respondent to collect on the supersedeas bond. This was to fully
G.R. No. 170852, September 12, 2008, Corona, J. satisfy the judgment amount in the labor case. Respondent paid
to the NLRC the amount guaranteed by the bond. It notified
Contracts are perfected by mere consent. This is manifested by the petitioners and forthwith deposited the undated check. It was,
meeting of the offer and the acceptance upon the object and cause however, dishonored because the account was already closed.
which are to constitute the contract. Here, the object of the  
contract was the issuance of the bond. The cause or consideration Respondent filed in the RTC of Cebu City, a complaint for sum of
consisted of the premiums paid. The bond was issued after money and damages with prayer for a writ of preliminary
petitioners complied with the requirements. At this point, the attachment against petitioners. The RTC ruled in favor of
contract of suretyship was perfected. respondent. It ordered petitioners to pay to respondent.
 
FACTS: On appeal, the CA affirmed the RTCs decision. Reconsideration
Petitioners applied for a supersedeas bond with respondent was denied in its resolution. The CA agreed with the RTC that the
Capital Insurance & Surety Co., Inc., a surety and non-life surety agreement between petitioners and respondent had been
insurance company. This bond was required in order to perfect perfected. Its perfection was not dependent on the acceptance by
their appeal to the National Labor Relations Commission the NLRC of the appeal of petitioners in the labor case. Thus,
(NLRC). Respondent required petitioners to do the following: (1) respondent correctly paid the indebtedness of petitioners.
to issue and deliver to it an undated check in the amount
equivalent to that of the supersedeas bond which it would issue; ISSUE:
(2) to execute a supplementary counter-guaranty with chattel WON the Surety Agreement was perfected.
mortgage over the sea vessel M/L Gretchen 2 owned by
petitioners and to surrender their original copy of certificate of HELD:
ownership over the vessel; (3) to execute an indemnity YES. There is no dispute that the parties entered into a contract
agreement wherein petitioners would agree to indemnify of suretyship wherein respondent as surety bound itself
respondent all damages it might sustain in its capacity as surety solidarily with petitioners (the principal debtors) to fulfill an
and (4) to pay the premiums. Except for the original copy of the obligation. The obligation was to pay the monetary award in the
certificate of ownership of M/L Gretchen 2, these requirements labor case should the decision become final and executory
were complied with. against petitioners.
 
Accordingly, the bond was issued and delivered to petitioners Contracts are perfected by mere consent. This is manifested by
who filed it in the NLRC. The NLRC dismissed the appeal for the meeting of the offer and the acceptance upon the object and
petitioners’ failure to post the bond within 10 days from receipt cause which are to constitute the contract. Here, the object of the
of the decision. This made the decision in the labor case final contract was the issuance of the bond. The cause or
against them. consideration consisted of the premiums paid. The bond was
 
MERCANTILE LAW REVIEW || Insurance Law 75
Compilation of Digests (Except 37)

issued after petitioners complied with the requirements. At this indemnity for attorneys fees shall be twenty (20%) percent of
point, the contract of suretyship was perfected. the amount claimed by the SURETY, but in no case less than TWO
  THOUSAND PESOS (P2,000.00), whether the SURETYS claim is
Petitioners cannot insist that the contract was subject to a settled judicially or extra-judicially.
suspensive condition, that is, the stay of the judgment of the  
labor arbiter. This was not a condition for the perfection of the INCONSTESTABILITY OF PAYMENT MADE BY THE SURETY:
contract but merely a statement of the purpose of the bond in its - Any payment or disbursement made by the SURETY on
whereas clauses. Aside from this, there was no mention of the account of the above-mentioned bond, either in the belief that
condition that before the contract could become valid and the SURETY was obligated to made such payment or in the belief
binding, perfection of the appeal was necessary. If the intention that said payment was necessary in order to avoid a greater loss
was to make it a suspensive condition, then the parties should or obligation for which the SURETY might be liable by virtue of
have made it clear in certain and unambiguous terms. the terms of the above-mentioned bond shall be final, and will
  not be contested by the undersigned, who jointly and
From the moment the contract is perfected, the parties are severally bind themselves to indemnity the SURETY for any
bound to comply with what is expressly stipulated as well as such payment or disbursement. (Emphasis supplied)
with what is required by the nature of the obligation in keeping  
with good faith, usage and the law. A surety is considered in law Undoubtedly, under these provisions, they are obligated to
to be on the same footing as the principal debtor in relation to reimburse respondent.
whatever is adjudged against the latter. Accordingly, as surety of
petitioners, respondent was obliged to pay on the bond when a
writ of execution was served on it. Consequently, it now has the
right to seek full reimbursement from petitioners for the amount
paid.
 
Moreover, petitioners signed an indemnity agreement which
contained the following stipulations:
 
INDEMNIFICATION: - To indemnify the SURETY for all damages,
payments, advances, losses, costs, taxes, penalties, charges,
attorney’s fees and expenses of whatever kind and nature that
the SURETY may at any time sustain or incur as a consequence of
having become surety upon the above-mentioned bond, and to
pay, reimburse and make good to the SURETY, its successors
and assigns, all sums or all money which it shall pay or
become liable to pay by virtue to said bond even if said
payment/s or liability exceeds the amount of the bond. The
MERCANTILE LAW REVIEW || Insurance Law 76
Compilation of Digests (Except 37)

50. STRONGHOLD INSURANCE COMPANY, INCORPORATED accepts the surety’s solidary undertaking to pay if the debtor
vs. TOKYU CONSTRUCTION COMPANY, LTD. does not pay. Such acceptance, however, does not change in any
GR Nos. 158820-21 June 5, 2009 J. Nachura material way the creditor’s relationship with the principal debtor
nor does it make the surety an active party to the principal
A surety is released from its obligation when there is a material creditor-debtor relationship. In other words, the acceptance does
alteration of the principal contract in connection with which the not give the surety the right to intervene in the principal
bond is given, such as a change which imposes a new obligation on contract. The surety’s role arises only upon the debtor’s default,
the promising party, or which takes away some obligation already at which time, it can be directly held liable by the creditor for
imposed, or one which changes the legal effect of the original payment as a solidary obligor.
contract and not merely its form. However, a surety is not released
by a change in the contract, which does not have the effect of The surety is considered in law as possessed of the identity of the
making its obligation more onerous. debtor in relation to whatever is adjudged touching upon the
obligation of the latter. Their liabilities are so interwoven as to
FACTS: MIAA awarded Tokyu a contract to construct NAIA be inseparable. Although the contract of a surety is, in essence,
Terminal 2. Tokyu entered into a sub-contract agreement with secondary only to a valid principal obligation, the surety’s
Gabriel Enterprise for the construction of a storm drainage and a liability to the creditor is direct, primary, and absolute; he
sewage treatment plant in the area. In relation to this, Gabriel becomes liable for the debt and duty of another although he
Enterprise obtained a surety bond with Stronghold. Gabriel possesses no direct or personal interest over the obligations nor
failed to satisfy its obligation on the maturity date however, does he receive any benefit therefrom.
Tokyu modified the sub-contract agreement and extended
Gabriel’s completion date however it still failed to satisfy its Indeed, a surety is released from its obligation when there is a
obligation prompting Tokyu to file for claims against Stronghold material alteration of the principal contract in connection with
and a second insurance company, Tico, which Tokyu contracted which the bond is given, such as a change which imposes a new
with. obligation on the promising party, or which takes away some
obligation already imposed, or one which changes the legal effect
ISSUE: Whether or not Stronghold is exonerated from its liability of the original contract and not merely its form. However, a
due to the revision of the subcontract. surety is not released by a change in the contract, which does not
have the effect of making its obligation more onerous.
HELD:
NO. Stronghold is not exonerated from liability. In the instant case, the revision of the subcontract agreement did
not in any way make the obligations of both the principal and the
We wish to stress herein the nature of suretyship, which actually surety more onerous. To be sure, petitioner never assumed
involves two types of relationship --- the underlying principal added obligations, nor were there any additional obligations
relationship between the creditor (respondent) and the debtor imposed, due to the modification of the terms of the contract.
(Gabriel), and the accessory surety relationship between the Failure to receive any notice of such change did not, therefore,
principal (Gabriel) and the surety (petitioner).The creditor exonerate petitioner from its liabilities as surety.
MERCANTILE LAW REVIEW || Insurance Law 77
Compilation of Digests (Except 37)

51. FIRST LEPANTO-TAISHO INSURANCE CORP. (now known ISSUE:


as FLT PRIME INSURANCE CORP.) vs. CHEVRON PHILIPPINES,
INC. (formerly known as CALTEX [PHILIPPINES], INC.) Is First Lepanto liable notwithstanding the fact that there was
G.R. No. 177839, 18 January 2012, VILLARAMA, JR., J. non-compliance of the requirement under the bond agreement to
submit the principal contract?
A surety contract is merely a collateral one, its basis is the
principal contract or undertaking which it secures. Necessarily, the RULING:
stipulations in such principal agreement must at least be
communicated or made known to the surety. It is basic that if the No.
terms of a contract are clear and leave no doubt upon the
intention of the contracting parties, the literal meaning of its The law is clear that a surety contract should be read and
stipulations shall control. Hence, non-compliance by the creditor interpreted together with the contract entered into between the
impacts not on the validity or legality of the surety contract but on creditor and the principal. Section 176 of the Insurance
the creditor’s right to demand performance from the surety. Code states:

FACTS: Sec. 176. The liability of the surety or sureties shall be


joint and several with the obligor and shall be limited to
Chevron Philippines, Inc. (Chevron) sued First Lepanto-Taisho the amount of the bond. It is determined strictly by the
Insurance Corp. (First Lepanto) as surety of Fumitechniks for the terms of the contract of suretyship in relation to the
latter’s non-compliance of its obligation to pay its oil and principal contract between the obligor and the obligee.
petroleum purchases as a distributor of Chevron. However, First
Lepanto refused to pay Chevron on the ground that the latter A surety contract is merely a collateral one, its basis is the
failed to submit to the former a copy of the distributor principal contract or undertaking which it secures. Necessarily,
agreement in which the surety agreement depends. First Lepanto the stipulations in such principal agreement must at least be
argues that the bond agreement specifically requires Chevron to communicated or made known to the surety particularly in this
submit a copy of the principal agreement such that the non- case where the bond expressly guarantees the payment of
compliance of which will make the surety agreement ineffective. Chevron’s fuel products withdrawn by Fumitechniks in
accordance with the terms and conditions of their agreement.
Chevron maintains that the delivery of the bond and acceptance The bond specifically makes reference to a written agreement. It
of premium payment by First Lepanto binds the latter as surety, is basic that if the terms of a contract are clear and leave no
notwithstanding the non-submission of the oral distributorship doubt upon the intention of the contracting parties, the literal
and credit agreement which understandably cannot be attached meaning of its stipulations shall control.  Moreover, being an
to the bond. onerous undertaking, a surety agreement is strictly construed
against the creditor, and every doubt is resolved in favor of the
solidary debtor. Having accepted the bond, Chevron as creditor
must be held bound by the recital in the surety bond that the
MERCANTILE LAW REVIEW || Insurance Law 78
Compilation of Digests (Except 37)

terms and conditions of its distributorship contract be reduced


in writing or at the very least communicated in writing to the
surety. Such non-compliance by the creditor (Chevron) impacts
not on the validity or legality of the surety contract but on the
creditor’s right to demand performance.

It bears stressing that the contract of suretyship imports entire


good faith and confidence between the parties in regard to the
whole transaction, although it has been said that the creditor
does not stand as a fiduciary in his relation to the surety. The
creditor is generally held bound to a faithful observance of the
rights of the surety and to the performance of every duty
necessary for the protection of those rights.  Moreover, in this
jurisdiction, obligations arising from contracts have the force of
law between the parties and should be complied with in good
faith. Chevron is charged with notice of the specified form of the
agreement or at least the disclosure of basic terms and
conditions of its distributorship and credit agreements with its
client Fumitechniks after its acceptance of the bond delivered by
the latter. However, it never made any effort to relay those terms
and conditions of its contract with Fumitechniks upon the
commencement of its transactions with said client, which
obligations are covered by the surety bond issued by First
Lepanto. Contrary to Chevron’s assertion, there is no indication
in the records that First Lepanto had actual knowledge of its
alleged business practice of not having written contracts with
distributors; and even assuming First Lepanto was aware of such
practice, the bond issued to Fumitechniks and accepted by
Chevron specifically referred to a "written agreement."
MERCANTILE LAW REVIEW || Insurance Law 79
Compilation of Digests (Except 37)

52. INTRA-STRATA ASSURANCE CORPORATION and Customs demanded payment of the amounts due from Grand
PHILIPPINE HOME ASSURANCE CORPORATION vs. REPUBLIC Textile as importer, and from Intra-Strata and PhilHome as
OF THE PHILIPPINES, represented by the BUREAU OF sureties. All three failed to pay. The government responded on
CUSTOMS January 14, 1983 by filing a collection suit against the parties
with the RTC of Manila.
G.R. No. 156571, July 9, 2008, BRION, J.:
After hearing, the RTC rendered its January 4, 1995 decision
Section 175 of the Insurance Code defines a contract of suretyship
finding Grand Textile (as importer) and the petitioners (as
as an agreement whereby a party called the surety guarantees the
sureties) liable for the taxes, duties, and charges due on the
performance by another party called the principal or obligor of an
imported articles.
obligation or undertaking in favor of another party called the
obligee.
The CA fully affirmed the RTC decision in its decision dated
November 26, 2002.
FACTS: Grand Textile is a local manufacturing corporation. In
1974, it imported from different countries various articles such
ISSUE: Whether the withdrawal of the stored goods, wares, and
as dyestuffs, spare parts for textile machinery, polyester filament
merchandise – without notice to them as sureties – released
yarn, textile auxiliary chemicals, trans open type reciprocating
them from any liability for the duties, taxes, and charges they
compressor, and trevira filament. Subsequent to the importation,
committed to pay under the bonds they issued.
these articles were transferred to Customs Bonded Warehouse
No. 462. As computed by the Bureau of Customs, the customs
HELD:
duties, internal revenue taxes, and other charges due on the
importations amounted to ₱2,363,147.00. To secure the payment
No. The surety is still liable in the absence of the express
of these obligations pursuant to Section 1904 of the Tariff and
provision requiring prior notice.
Customs Code (Code),4 Intra-Strata and PhilHome each issued
general warehousing bonds in favor of the Bureau of Customs.
Section 175 of the Insurance Code defines a contract of
These bonds, the terms of which are fully quoted below,
suretyship as an agreement whereby a party called the surety
commonly provide that the goods shall be withdrawn from the
guarantees the performance by another party called the
bonded warehouse "on payment of the legal customs duties,
principal or obligor of an obligation or undertaking in favor of
internal revenue, and other charges to which they shall then be
another party called the obligee, and includes among its various
subject."
species bonds such as those issued pursuant to Section 1904 of
the Code. Significantly, "pertinent provisions of the Civil Code of
Without payment of the taxes, customs duties, and charges due
the Philippines shall be applied in a suppletory character
and for purposes of domestic consumption, Grand Textile
whenever necessary in interpreting the provisions of a contract
withdrew the imported goods from storage. 6 The Bureau of
MERCANTILE LAW REVIEW || Insurance Law 80
Compilation of Digests (Except 37)

of suretyship." By its very nature under the terms of the laws


regulating suretyship, the liability of the surety is joint and
several but limited to the amount of the bond, and its terms are
determined strictly by the terms of the contract of suretyship in
relation to the principal contract between the obligor and the
obligee.

There are effectively two (2) contracts involved when a surety


agreement comes into play – a principal contract and an
accessory contract of suretyship. Under the accessory contract,
the surety becomes directly, primarily, and equally bound with
the principal as the original promissor although he possesses no
direct or personal interest over the latter’s obligations and does
not receive any benefit therefrom.

The obligation to pay, principally by the importer, is shared by


the latter with a willing third party under a suretyship
agreement under Section 1904 of the Code.

There is, firstly, a principal obligation belonging to the importer-


obligor as provided under Section 101; secondly, there is an
accessory obligation, assumed by the sureties pursuant to
Section 1904 which, by the nature of a surety agreement,
directly, primarily, and equally bind them to the obligee to pay
the obligor’s obligation.

Significantly, nowhere in the petitioners’ bonds does it state that


prior notice is required to fix the sureties’ liabilities. Without
such express requirement, the creditor’s right to enforce
payment cannot be denied as the petitioners became bound as
soon as Grand Textile, the principal debtor, defaulted. Thus, the
filing of the collection suit was sufficient notice to the sureties of
their principal’s default.
MERCANTILE LAW REVIEW || Insurance Law 81
Compilation of Digests (Except 37)

LIFE INSURANCE that justified the denial of the claim. The widow of the late Dr.
Leuterio, respondent Medarda V. Leuterio, filed a complaint
53. GREAT PACIFIC LIFE ASSURANCE CORP. vs. COURT OF against Grepalife for Specific Performance with Damages. During
APPEALS AND MEDARDA V. LEUTERIO the trial, Dr. Hernando Mejia, who issued the death certificate,
G.R. No. 113899 October 13, 1999 J. QUISUMBING7 was called to testify. Dr. Mejia’s findings, based partly from the
information given by the respondent widow, stated that Dr.
A life insurance policy is a valued policy. Unless the interest of a Leuterio complained of headaches presumably due to high blood
person insured is susceptible of exact pecuniary measurement, the pressure. The inference was not conclusive because Dr. Leuterio
measure of indemnity under a policy of insurance upon life or was not autopsied, hence, other causes were not ruled out. The
health is the sum fixed in the policy. trial court rendered a decision in favor of respondent widow and
against Grepalife. The Court of Appeals sustained the trial courts
FACTS: decision. Hence, the present petition.
A contract of group life insurance was executed between
petitioner Great Pacific Life Assurance Corporation (Grepalife) ISSUE:
and Development Bank of the Philippines (DBP). Grepalife Whether the Court of Appeals erred in holding Grepalife liable in
agreed to insure the lives of eligible housing loan mortgagors of the amount of eighty six thousand, two hundred (P86,200.00)
DBP. Dr. Wilfredo Leuterio, a physician and a housing debtor of pesos without proof of the actual outstanding mortgage payable
DBP applied for membership in the group life insurance plan. In by the mortgagor to DBP? (Note: There are many issues in this
an application form, Dr. Leuterio answered questions concerning case, but this is the one which is related to the topic of life
his health condition. Among those questions were: Have you ever isurance. This case is repeated four times in the insurance case
had, or consulted, a physician for a heart condition, high blood list)
pressure, cancer, diabetes, lung, kidney or stomach disorder or
any other physical impairment? Dr. Leuterio answered “no.” and HELD:
answered “yes” to the question: Are you now, to the best of your NO. Petitioner claims that there was no evidence as to the
knowledge, in good health? Grepalife issued Certificate No. B- amount of Dr. Leuterio's outstanding indebtedness to DBP at the
18558, as insurance coverage of Dr. Leuterio, to the extent of his time of the mortgagor's death. Hence, for private respondent's
DBP mortgage indebtedness amounting P86,200.00. On August 6, failure to establish the same, the action for specific performance
1984, Dr. Leuterio died due to massive cerebral hemorrhage. should be dismissed. Petitioner's claim is without merit. A life
Consequently, DBP submitted a death claim to Grepalife. insurance policy is a valued policy. Unless the interest of a
Grepalife denied the claim alleging that Dr. Leuterio was not person insured is susceptible of exact pecuniary measurement,
physically healthy when he applied for an insurance coverage on the measure of indemnity under a policy of insurance upon life
November 15, 1983. Grepalife insisted that Dr. Leuterio did not or health is the sum fixed in the policy. The mortgagor paid the
disclose he had been suffering from hypertension, which caused premium according to the coverage of his insurance, which
his death. Allegedly, such nondisclosure constituted concealment states that:

7
Same with case numbers 14 and 17
MERCANTILE LAW REVIEW || Insurance Law 82
Compilation of Digests (Except 37)

The policy states that upon receipt of due proof of


the Debtor's death during the terms of this
insurance, a death benefit in the amount of
P86,200.00 shall be paid.

In the event of the debtor's death before his


indebtedness with the creditor shall have been
fully paid, an amount to pay the outstanding
indebtedness shall first be paid to the Creditor
and the balance of the Sum Assured, if there is
any shall then be paid to the beneficiary/ies
designated by the debtor." 

However, we noted that the Court of Appeals' decision was


promulgated on May 17, 1993. In private respondent's
memorandum, she states that DBP foreclosed in 1995 their
residential lot, in satisfaction of mortgagor's outstanding loan.
Considering this supervening event, the insurance proceeds shall
inure to the benefit of the heirs of the deceased person or his
beneficiaries. Equity dictates that DBP should not unjustly enrich
itself at the expense of another (Nemo cum alterius detrimenio
protest). Hence, it cannot collect the insurance proceeds, after it
already foreclosed on the mortgage. The proceeds now rightly
belong to Dr. Leuterio's heirs represented by his widow, herein
private respondent Medarda Leuterio
MERCANTILE LAW REVIEW || Insurance Law 83
Compilation of Digests (Except 37)

ON MOTOR VEHICLE INSURANCE installments according to the schedule of payment indicated in


said note and secured by a chattel mortgage over a brand new
54. Perla Compania De Seguros, Inc. vs. The Court of Appeals, red Ford Laser 1300 5DR Hatchback 1981 model which is
Herminio Lim and Evelyn Lim registered under the name of private respondent Herminio Lim
G.R. No. 96452 May 7, 1992 J. Nocon and insured with the petitioner Perla Compania de Seguros, Inc.
(Perla) for comprehensive coverage under Policy No. PC/41PP-
There is no causal connection between the possession of a QCB-43383.
valid driver's license and the loss of a vehicle. To rule otherwise
would render car insurance practically a sham since an insurance Supercars, Inc., with notice to private respondent
company can easily escape liability by citing restrictions which are spouses, assigned to petitioner FCP Credit Corporation (FCP) its
not applicable or germane to the claim, thereby reducing rights, title and interest on said promissory note and chattel
indemnity to a shadow. mortgage as shown by the Deed of Assignment.

The insurance policy was therefore meant to be an At around 2:30 P.M. of November 9, 1982, said vehicle
additional security to the principal contract, that is, to insure that was carnapped while parked at the back of Broadway Centrum
the promissory note will be paid in case the automobile is lost along N. Domingo Street, Quezon City.
through accident or theft.
On November 10, 1982, private respondent Evelyn Lim
Theft is an entirely different legal concept from that of reported said incident to the Land Transportation Commission in
accident. Theft is committed by a person with the intent to gain or, Quezon City, as shown by the letter of her counsel to said office,
to put it in another way, with the concurrence of the doer's will. On [8] in compliance with the insurance... requirement. She also
the other hand, accident, although it may proceed or result... from filed a complaint with the Headquarters, Constabulary Highway
negligence, is the happening of an event without the concurrence Patrol Group.
of the will of the person by whose agency it was caused.
Private respondent filed a claim for loss with the
Clearly, the risk against accident is distinct from the risk petitioner Perla but said claim was denied on November 18,
against theft. The 'authorized driver clause' in a typical insurance 1982 on the ground that Evelyn Lim, who was using the vehicle
policy is in contemplation or anticipation of accident in the legal before it was carnapped, was... in possession of an expired
sense in which it should be understood, and not in... contemplation driver's license at the time of the loss of said vehicle which is in
or anticipation of an event such as theft. violation of the authorized driver clause of the insurance policy...
private respondents requested from petitioner FCP for a
FACTS suspension of payment.

On December 24, 1981, private respondents spouses Perla, however, denied private respondents' claim.
Herminio and Evelyn Lim executed a promissory note in favor of
Supercars, Inc. in the sum of P77,940.00, payable in monthly
MERCANTILE LAW REVIEW || Insurance Law 84
Compilation of Digests (Except 37)

Consequently, petitioner FCP demanded that private applicable or germane to the claim, thereby reducing indemnity
respondents pay the whole balance of the promissory note or to to a shadow.
return the vehicle but the latter refused.
Private respondents are not relieved of their obligation
Private respondents appealed the same to the Court of to pay the former the installments due on the promissory note
Appeals, which reversed said decision. on account of the loss of the automobile.

ISSUE The insurance policy was therefore meant to be an


additional security to the principal contract, that is, to insure that
Whether or not there is a grave abuse of discretion on the the promissory note will still be paid in case the automobile is
part of the appellate court in holding that private respondents lost through accident or theft.
did not violate the insurance contract because the authorized
driver clause is not applicable to the "Theft" clause of said It is clear from the abovementioned provision that upon
Contract? the loss of the insured vehicle, the insurance company Perla
undertakes to pay directly to the mortgagor or to their assignee,
HELD FCP, the outstanding balance of the mortgage at the time of said
loss under the mortgage... contract.
NO. The car was insured against a malicious act such as
theft. Therefore the “Theft” clause in the contract should apply
and not the authorized driver clause. The risk against accident is
different from the risk against theft. 

The appellate court stated: The "authorized driver


clause" in a typical insurance policy is in contemplation or
anticipation of accident in the legal sense in which it should be
understood, and not in contemplation or anticipation of an event
such as theft. The distinction — often seized upon by insurance
companies in resisting claims from their assureds — between
death occurring as a result of accident and death occurring as a
result of intent may, by analogy, apply to the case at bar.

There was no connection between valid possession of a


license and the loss of a vehicle. To rule otherwise would render
car insurance practically a sham since an insurance company can
easily escape liability by citing restrictions... which are not
MERCANTILE LAW REVIEW || Insurance Law 85
Compilation of Digests (Except 37)

THEFT CLAUSE
ISSUE/S:
55. PARAMOUNT INSURANCE CORPORATION vs. SPOUSES
YVES and MARIA TERESA REMONDEULAZ Whether or not the loss of the car of the spouses falls
G.R. No. 173773, November 28, 2012, Justice Peralta within the “theft clause” of the insurance contract, making it
compensable
The taking of a vehicle by another person without the permission
or authority from the owner thereof is sufficient to place it within
the ambit of the word theft as contemplated in the policy, and is HELD:
therefore, compensable
Yes. To interpret the "theft clause" of an insurance policy,
FACTS: the Court explained that when one takes the motor vehicle of
another without the latter’s consent even if the motor vehicle is
On May 26, 1994, respondent spouses Yves and Maria later returned, there is theft – there being intent to gain as the
Teresa (Remondeulaz) insured their 1994 Toyota Corolla sedan use of the thing unlawfully taken constitutes gain. The taking of a
under a comprehensive car insurance policy with petitioner vehicle by another person without the permission or authority
Paramount Insurance Corporation. While the contract was in from the owner thereof is sufficient to place it within the ambit
effect, the spouses’ car was taken by one Ricardo Sales, to whom of the word theft as contemplated in the policy, and is therefore,
they entrusted the car to add accessories and improvement, but compensable.
did not return the same within the agreed three-day period. 
They filed a complaint sheet and immediately reported the theft In the instant case, Sales did not have juridical possession
to the Traffic Management Command of the PNP.  They notified over the vehicle.  Here, it is apparent that the taking of
the petitioner insurance company to claim for reimbursement, respondents’ vehicle by Sales is without any consent or authority
but the latter refused to pay, hence they filed a case for sum of from the former. Records would show that respondents
money against the company before the RTC. RTC dismissed the entrusted possession of their vehicle only to the extent that Sales
complaint. Aggrieved, respondents filed an appeal to the Court of will introduce repairs and improvements thereon, and not to
Appeals. The appellate court reversed and set aside the Order permanently deprive them of possession thereof.  Since, theft can
issued by the trial court stating that the company is liable to the also be committed through misappropriation, the fact that Sales
spouses under the “theft clause” of their insurance contract, failed to return the subject vehicle to respondents constitutes
hence, the present case. Petitioner argued that they are not liable Qualified Theft.  Hence, since respondents’ car is undeniably
because the car was not stolen but was entrusted to another covered by a Comprehensive Motor Vehicle Insurance Policy that
person. allows for recovery in cases of theft, petitioner is liable under the
policy for the loss of respondents’ vehicle under the “theft
clause.”
MERCANTILE LAW REVIEW || Insurance Law 86
Compilation of Digests (Except 37)

56. ALPHA INSURANCE AND SURETY CO. vs. ARSENIA SONIA the insurance policy, since Section III thereof did not qualify as to
CASTOR who would commit the theft. Thus:
G.R. No. 198174, September 2, 2013
Theft perpetrated by a driver of the insured is not an exception
FACTS: to the coverage from the insurance policy subject of this case.
This is evident from the very provision of Section III – “Loss or
On February 21, 2007, respondent entered into a contract of Damage.” The insurance company, subject to the limits of
insurance, Motor Car Policy No. MAND/CV-00186, with liability, is obligated to indemnify the insured against theft. Said
petitioner, involving her motor vehicle, a Toyota Revo DLX DSL. provision does not qualify as to who would commit the theft.
The contract of insurance obligates the petitioner to pay the Thus, even if the same is committed by the driver of the insured,
respondent the amount of P630,000.00 in case of loss or damage there being no categorical declaration of exception, the same
to said vehicle during the period covered, shiich is from February must be covered. As correctly pointed out by the plaintiff, “An
26, 2007 to February 26, 2008. On April 16, 2007, at about 9:00 insurance contract should be interpreted as to carry out the
am., respondent instructed her driver, Jose Joel Salazr Lanuza, to purpose for which the parties entered into the contract which is
bring the above-described vehicle to a nearby auto-shop for a to insure against risks of loss or damage to the goods. Such
tune-up. However, Lanuza no longer returned the motor vehicle interpretation should result from the natural and reasonable
to respondet and despite diligent efforts to locate the same, said meaning of language in the policy. Where restrictive provisions
efforts proved futile. Resultantly, respondent promptly reorted are open to two interpretations, that which is most favourable to
the incident to the police and notified the petitioner of the said the insured is adopted.”
loss and demanded payment of the insurance proceeds. In a
letter dated July 5, 20017, petitioner denied the insurance claim Moreover, contracts of insurance, like other contracts, are to be
of respondent, stating among others, thus: Upon verification of construed according to the sense and meaning of the terms
the documents submitted, particularly the police report and your which the parties themselves have used. If such terms which the
affidavit, which states that the culprit, who stole the Insured unit, parties themselves have used. If such terms are clear and
is employed with you. We would like it invite you on the unambiguous, they must be taken and understood in their plain,
provision of the Policy under Exceptions to Section-III. ordinary and popular sense. Accordingly, in interpreting the
exclusions in an insurance contract, the terms used specifying
ISSUE: the excluded classes therein are to be given their meaning as
understood in common speech.
Whether or not the loss of respondent’s vehicle is excluded
under the insurance policy.

RULING:

NO. The RTC scrupulously elaborated that theft perpetrated by


the driver of the insured is not an exception to the coverage from
MERCANTILE LAW REVIEW || Insurance Law 87
Compilation of Digests (Except 37)

OTHER INSURANCE CLAUSE Subsequently, Reputable impleaded Malayan as third-party


defendant.
57. MALAYAN INSURANCE CO., INC. vs. PHILIPPINES FIRST
INSURANCE CO., INC. and REPUTABLE FORWARDER Malayan argued, among others, that under Section 5 of the SR
SERVICES, INC. 8 Policy, the insurance does not cover any loss or damage to
property which at the time of the happening of such loss or
G.R. No. 184300, July 11, 2012, REYES, J. damage is insured by any marine policy and that the SR Policy
expressly excluded third-party liability.
FACTS: On November 18, 1993, Wyeth procured Marine Policy
No. MAR 13797 (Marine Policy) from respondent Philippines Section 5. INSURANCE WITH OTHER COMPANIES. The
First Insurance Co., Inc. (Philippines First) to secure its interest insurance does not cover any loss or damage to property
over its own products. The policy covers all risks of direct which at the time of the happening of such loss or
physical loss or damage from any external cause. On December 1, damage is insured by or would but for the existence of
1993, Wyeth executed its annual contract of carriage with this policy, be insured by any Fire or Marine policy or
Reputable. policies except in respect of any excess beyond the
amount which would have been payable under the Fire
Under the contract, Reputable undertook to answer for all risks or Marine policy or policies had this insurance not been
with respect to the goods while the goods/products are in transit effected.
and until actual delivery to the customers, salesmen, and dealers
of the COMPANY. On February 11, 1994, Reputable signed a Malayan sought the dismissal of the third-party complaint
Special Risk Insurance Policy (SR Policy) with petitioner Malayan against it. In the alternative, it prayed that it be held liable for no
for the amount of P1,000,000.00. more than P468,766.70, its alleged pro-rata share of the loss
based on the amount covered by the policy, subject to the
On October 6, 1994, during the effectivity of the Marine Policy provision of Section 12 of the SR Policy, which states:
and SR Policy, Reputable received from Wyeth 1,000 boxes of
Promil infant formula worth P2,357,582.70 to be delivered by 12. OTHER INSURANCE CLAUSE. If at the time of any loss
Reputable to Mercury Drug Corporation in Libis, Quezon City. or damage happening to any property hereby insured,
Unfortunately, on the same date, the truck carrying Wyeth’s there be any other subsisting insurance or insurances,
products was hijacked by about 10 armed men. whether effected by the insured or by any other person
or persons, covering the same property, the company
Philippines First paid Wyeth P2,133,257.00 as indemnity. shall not be liable to pay or contribute more than its
Philippines First then demanded reimbursement from ratable proportion of such loss or damage.
Reputable. The latter, however, ignored the demand.
RTC rendered its Decision finding Reputable liable to Philippines
First for the amount of indemnity it paid to Wyeth, among others.
8
Same with case number 24
MERCANTILE LAW REVIEW || Insurance Law 88
Compilation of Digests (Except 37)

In turn, Malayan was found by the RTC to be liable to Reputable Even though the two concerned insurance policies were issued
to the extent of the policy coverage. over the same goods and cover the same risk, there arises no
double insurance since they were issued to two different
CA rendered the assailed decision sustaining the ruling of the persons/entities having distinct insurable interests. Necessarily,
RTC. over insurance by double insurance cannot likewise exist. Hence,
as correctly ruled by the RTC and CA, neither Section 5 nor
ISSUE: Whether the “other insurance” clause and “over Section 12 of the SR Policy can be applied.
insurance” clause are applicable.
Suffice it to say that Malayan's and Reputable's respective
HELD: No. Neither Sections (5 and 12) were applicable. liabilities arose from different obligations- Malayan's is based on
the SR Policy while Reputable's is based on the contract of
Section 5 is actually the other insurance clause (also called carriage.
"additional insurance" and "double insurance"). Section 5 does
not provide for the nullity of the SR Policy but simply limits the
liability of Malayan only up to the excess of the amount that was
not covered by the other insurance policy. In interpreting the
"other insurance clause" in Geagonia, the Court ruled that the
prohibition applies only in case of double insurance.

Section 12 of the SR Policy, on the other hand, is the over


insurance clause. More particularly, it covers the situation where
there is over insurance due to double insurance. In such case,
Section 15 provides that Malayan shall "not be liable to pay or
contribute more than its ratable proportion of such loss or
damage." This is in accord with the principle of contribution
provided under Section 94(e) of the Insurance Code,  which states
that "where the insured is over insured by double insurance,
each insurer is bound, as between himself and the other insurers,
to contribute ratably to the loss in proportion to the amount for
which he is liable under his contract."

Clearly, both Sections 5 and 12 presuppose the existence of a


double insurance. The pivotal question that now arises is
whether there is double insurance in this case such that either
Section 5 or Section 12 of the SR Policy may be applied.
MERCANTILE LAW REVIEW || Insurance Law 89
Compilation of Digests (Except 37)

ON MORTGAGE REDEMPTION INSURANCE We have held that provisions, conditions or exceptions


tending to work a forfeiture of insurance policies should be
58. NORA CANSING SERRANO v. COURT OF APPEALS and construed most strongly against those for whose benefit they are
SOCIAL SECURITY COMMISSION inserted, and most favorably toward those against whome they are
G.R. No. L-35529, July 16, 1984, J. MAKASIAR intended to operate. (Trinidad v. Orient Protective Ass., 67 Phil.
181)
Section 2 of Article II of the Group Mortgage Redemption
Insurance Policy provides tha the insurance coverage shall be The Mortgage Redemption Insurance device is not only for
“automatic” and limited only by the amount of insurance and age the protection of the SYSTEM but also for the benefit of the
requirement. While the same section has for its title the mode of mortgagor. On the part of the SYSTEM, it has to enter into such
acceptance, what is controlling is the meaning of the provision form of contract so that in the event of the unexpected demise of
itself. The said section can only convey the idea that the mortgagor the mortgagor during the subsistence of the mortgage contract,
who is eligible for coverage on or after the date of issue shall be the proceeds from such insurance will be applied to the payment of
automatically insured. The only condition is that the age the mortgage debt, thereby relieving the heirs of the mortgagor
requirement should be satisfied, which had been complied with by from paying the obligation. The SYSTEM insures the payment to
the deceased mortgagor in the instant case. Under said Section 2, itself of the loan with the insurance proceeds. It also negates any
mortgage redemption insurance is not just automatic; it is future problem that can crop up should the heirs be not in a
compulsory for all qualified borrowers. This is the same automatic position to pay the mortgage loan. In short, the process of
redemption insurance applied to all qualified borrowers by the amortization is hastened and possible litigation in the future is
GSIS (Government Service Insurance System) and the DBP avoided. In a similar vein, ample protection is given to the
(Development Bank of the Philippines). mortgagor under such a concept so that in the event of his death;
the mortgage obligation will be extinguished by the application of
However, Section 3 of Article II of the Group Mortgage the insurance proceeds to the mortgage indebtedness.
Redemption Insurance Policy presents and ambiguity. The effective
date of coverage can be interpreted to mean that the insurance FACTS
contract takes effect “from the beginning of the amortization
perios of such Mortgage Loan” or “partial release of Mortgage Petitioner Nora Cansing Serrano (Nora) is the widow of
Loan.” The ambiguity in Section 3 of Article II should be resolved in the late Bernardo G. Serrano (Bernardo). The latter was an
favor of the petitioner. The Civil Code provides: airline pilot of Air Manila, Inc. and a member of the Social
Article 1377. The interpretation of obscure words or Security System (SSS). Bernardo applied for a Group Mortgage
stipulations in a contract shall not favor the party who Redemption Policy and was issued by Private Life Insurance
caused the obscurity. Companies, operating in the Philippines, for Php 37,4000.00 for
the construction of Bernardo’s house. Nora was a co-mortgagor.
MERCANTILE LAW REVIEW || Insurance Law 90
Compilation of Digests (Except 37)

and who would not be over 75 nearest birthday on the


It was group life insurance policy on the lives of housing date on which the original term of the Mortgage Loan
loan mortgagors of the SYSTEM. Under this Group Mortgage expires shall be eligible for insurance coverage under this
Redemption scheme, a grantee of a housing loan of the SYSTEM Policy, provided that if the total indebtedness to the
is required to mortgage the house constructed out of the loan Creditor under the new Mortgage Loan and the
and the lot on which it stands. The SYSTEM takes a life insurance outstanding balance of any prior Mortgage Loan or Loans
insured hereunder, exceeds P70,000.00, he will be
on the eligible mortgagor to the extent of the mortgage
eligible for insurance coverage up to this maximum limit
indebtedness such that if the mortgagor dies, the proceeds of his
only. 
life insurance under the Group Redemption Policy will be used to
pay his indebtedness to the SYSTEM and the deceased's heirs Co-makers or co-signers of mortgage contract are not
will thereby be relieved of the burden of paying for the eligible for coverage under this Policy. 
amortization of the deceased's still unpaid loan to the SYSTEM
Section 2.  Mode of Acceptance. — Any Mortgagor who is
A partial release in the amount of Php 35,400.00 was eligible for coverage on or after the Date of Issue shall
effected. In less than a year therefrom, Captain Serrano died in a be automatically insured, subject to the amount of
plane crash. The SYSTEM closed his housing loan account to the insurance limit in Section 1 hereof, without proof of
released amount of Php 35,400.00. insurability provided that he is not more than age 60
nearest birthday at the time the Mortgage Loan is
Nora sent a request letter to the SSS Chairman for the granted. Such a mortgagor who is over age 60 nearest
birthday at the time the Mortgage Loan is granted may be
exension of the benefits of the Group Mortgage Redemption
accepted for insurance only subject to the submission of
Insurance to her. The Administrator of the SYTEM recommended
evidence of insurability satisfactory to the Subscribing
its disapproval because Cpt. Serrano was allegedly not covered Companies. 
by the Group Mortgage Redemption Insurance policy at the time
of his death. The SSS sustained the SYSTEM’s decision. Upon Any eligible Mortgagor who was already a Mortgagor
appeal, the CA affirmed the SSS. before the Date of Issue shall be automatically insured,
subject to the amount of insurance limit in Section 1
The pertitent provisions of the insurance policy under hereof, without proof of insurability provided that he is
Article II are as follows: not more than age 60 nearest birthday on the Date of
Issue and that he makes written application to the
Section 1. Eligibility.— Every mortgagor who is not over Creditor for coverage within ninety (90) days from the
age 65 nearest birthday at the time the Mortgage Loan is Date of Issue. If such a Mortgagor applies for coverage
granted (or, in the case of a Mortgagor applying for after ninety (90) days from the Date of Issue. he may be
insurance coverage on a Mortgage Loan granted before accepted for insurance upon written application therefor,
the Date of Issue, at the time he makes such application)
MERCANTILE LAW REVIEW || Insurance Law 91
Compilation of Digests (Except 37)

subject to the submission of evidence of insurability to


the Subscribing Companies.  ISSUE

Section 3. Effective Date of Insurance. — The insurance on When is the effective date of coverage for the Mortgage
the life of each eligible Mortgagor Loan or partial release Redemption Insurance?
of Mortgage Loan accepted for coverage who becomes a
Mortgagor on or after the Date of Issue shall take effect
RULING
from the beginning of the amortization period of such
Mortgage Loan or partial release of Mortgage Loan. 
From the partial release of the loan. Noteworthy is
The beginning of the amortization period as used herein Section 16 of the Mortgage Contract of Social Security System in
shall mean the first day of the month preceding the connection with the applications for housing loans:
month in which the first monthly amortization payment
falls due.  Section 16. – (a) The loan shall be secured against the
death of the borrower through the Mortgage Redemption
It is hereby understood that before any release on any Insurance Plan; (b) Coverage shall take effect on the date
approved Mortgage Loan is made by the Creditor, the of the first release voucher of the loan and shall continue
requisites binding the Mortgagor and the Creditor as until the real estate mortgage loan is fully paid;…”
regards to said Mortgage Loan shall have been (Emphasis supplied)
completed 
Further, considering Section 3 of Article II, the ambiguity
xxx xxx xxx
should be resolved in favor of Nora. The interpretation of
obscure words or stipulations in a contract shall not favor the
Sections 2 and 3 of the policy are conflicting. Section 2
party who caused the obscurity. (Article 1377, Civil Code)
provides that “any mortgagor who is eligible for coverage on or
after the Date of Issue shall be automatically insured,…” (Italics
While the issuance of the Group Mortgage Redemption
supplied) Further, the limit of the coverage is the age
Insurance is a contract between SSS and Private Life Insurance
requirement laid down in the provision.
Companies, the fact is that the SYSTEM entered into such a
contract to afford protection not only to itself should the
Section 3 provides that the insurance “shall take effect
mortgagor die before fully paying the loan but also to afford
from the beginning of the amortization period of such Mortgage
protection to the mortgagor. The rationale of the insurance
Loan or partial release of Mortgage loan.” (Italics supplied) Said
scheme by the SSS is in such a way that upon the death of the
provision is ambiguous wherein the effectivity date of coverage
mortgagor before the debt has been paid, the proceeds from such
can either be “from the beginning of the amortization period of
insurance will be applied to the payment of the mortgage debt
such Mortgage Loan” or “partial release of Mortgage Loan.”
MERCANTILE LAW REVIEW || Insurance Law 92
Compilation of Digests (Except 37)

thereby relieving the mortgagor’s heirs from paying the


obligation. It negates future problems that may arise should the
heirs be not in a position to pay the mortgage loan. It also
hastens amortization and avoids possible litigation. On the part
of the mortgagor, he is protected in the event of his death by the
extinguishment of the obligation upon the application of the
insurance proceeds to the mortgage indebtedness.

Further, SSS cannot be allowed to collect insurance


benefits from private life insurance companies and at the same
time avoid giving benefits of the Mortgage Redemption
Insurance plan to the mortgagor. It cannot unjustly enrich itself
at the expense of another (Nemo cum alterius detriment protest).
Every person must, in the exercise of his rights and in the
performance of his duties, act with justice, give everyone his due,
and observe honesty and good faith. (Article 19, Civil Code)
Furthermore, the raison d’être of the SSS is to extend social
benefits “to establish, develop to establish, develop, promote and
perfect a sound and viable tax-exempt social security service
suitable to the needs of the people throughout the Philippines,
which shall provide to covered employees and their families
protection against the hazards of disability, sickness, old age, and
death with a view to promote their well-being in the spirit of
social justice" (The Social Security Law, R.A. No. 1161, as
amended)

By the principle of social justice, as enshrined in the


Constitution (Section 6, Article II, 1976 Constitution), the SC
rules in favor of Nora.
MERCANTILE LAW REVIEW || Insurance Law 93
Compilation of Digests (Except 37)

59. GREAT PACIFIC LIFE ASSURANCE CORP. vs. COURT OF


APPEALS AND MEDARDA V. LEUTERIO9 ISSUE:
G.R. No. 113899. October 13, 1999 QUISUMBING, J. Whether the CA erred in holding Grepalife liable to DBP as
beneficiary in a group life insurance contract from a complaint
The rationale of a group insurance policy of mortgagors, filed by the widow of the decedent/mortgagor.
otherwise known as the mortgage redemption insurance, is a
device for the protection of both the mortgagee and the HELD:
mortgagor. On the part of the mortgagee, it has to enter into such NO. The CA did not erred in holding Grepalife liable to
form of contract so that in the event of the unexpected demise of DBP.
the mortgagor during the subsistence of the mortgage contract, The rationale of a group of insurance policy of
the proceeds from such insurance will be applied to the payment of mortgagors, otherwise known as the “mortgage redemption
the mortgage debt, thereby relieving the heirs of the mortgagor insurance,” is a device for the protection of both the mortgagee
from paying the obligation. and the mortgagor. On the part of the mortgagee, it has to enter
into such form of contract so that in the event of the unexpected
FACTS: demise of the mortgagor during the subsistence of the mortgage
Great Pacific Life Assurance Corporation (Grepalife) executed a contract, the proceeds from such insurance will be applied to the
contract of group life insurance with Development Bank of the payment of the mortgage debt, thereby relieving the heirs of the
Philippines (DBP) wherein Grepalife agreed to insure the lives of mortgagor from paying the obligation. In a similar vein, ample
eligible housing loan mortgagors of DBP. protection is given to the mortgagor under such a concept so that
One such loan mortgagor is Dr. Wilfredo Leuterio. In an in the event of death, the mortgage obligation will be
application form, Dr. Leuterio answered questions concerning extinguished by the application of the insurance proceeds to the
his test, attesting among others that he does not have any heart mortgage indebtedness. In this type of policy insurance, the
conditions and that he is in good health to the best of his mortgagee is simply an appointee of the insurance fund. Such
knowledge. loss-payable clause does not make the mortgagee a party to the
However, after about a year, Dr. Leuterio died due to “massive contract.
cerebral hemorrhage.” When DBP submitted a death claim to The insured, being the person with whom the contract was
Grepalife, the latter denied the claim, alleging that Dr. Leuterio made, is primarily the proper person to bring suit thereon.
did not disclose he had been suffering from hypertension, which Subject to some exceptions, insured may thus sue, although the
caused his death. Allegedly, such non-disclosure constituted policy is taken wholly or in part for the benefit of another person,
concealment that justified the denial of the claim. such as a mortgagee.
Hence, the widow of the late Dr. Leuterio filed a complaint And since a policy of insurance upon life or health may pass by
against Grepalife for “Specific Performance with Damages.” Both transfer, will or succession to any person, whether he has an
the trial court and the Court of Appeals found in favor of the insurable interest or not, and such person may recover it
widow and ordered Grepalife to pay DBP. whatever the insured might have recovered, the widow of the
decedent Dr. Leuterio may file the suit against the insurer,
9
Same with case number 14, 17, and 53 Grepalife.
MERCANTILE LAW REVIEW || Insurance Law 94
Compilation of Digests (Except 37)

ON THE LIABILITY OF THE INSURER DUE TO negligence, if there is no pre-existing contractual relation
NEGLIGENCE between the parties, is called a quasi-delict . . . . ". To sustain a
claim based thereon, the following requisites must concur: (a)
60. FGU Insurance Corp. v. Court of Appeals10 damage suffered by the plaintiff; (b) fault or negligence of the
defendant; and, (c) connection of cause and effect between the
G.R. No. 137775, March 31, 2005
fault or negligence of the defendant and the damage incurred by
the plaintiff. We agree with respondent court that petitioner
FACTS:
failed to prove the existence of the second requisite, i.e., fault or
negligence of defendant FILCAR, because only the fault or
On April 21, 1987, a car owned by private respondent FILCAR
negligence of Dahl-Jensen was sufficiently established, not that of
Transport Inc., rented to and driven by Dahl-Jensen, a Danish
FILCAR. It should be noted that the damage caused on the vehicle
tourist, swerved into the right and hit the car owned by Lydia
of Soriano was brought about by the circumstance that Dahl-
Soriano and driven by Benjamin Jacildone. Dahl-Jensen did not
Jensen swerved to the right while the vehicle that he was driving
possess a Philippine driver’s license. Petitioner, as the insurer of
was at the center lane. It is plain that the negligence was solely
Soriano’s car, paid the latter P25,382.20 and, by way of
attributable to Dahl-Jensen thus making the damage suffered by
subrogation, sued FILCAR, Dahl-Jensen, and Fortune Insurance
the other vehicle his personal liability. Respondent FILCAR did
Corporation, FILCAR’s insurer, for quasi-delict. The trial court
not have any participation therein. Respondent FILCAR being
dismissed the petition for failure to substantiate the claim for
engaged in a rent-a-car business was only the owner of the car
subrogation. The Court of Appeals affirmed the decision, but on
leased to Dahl-Jensen. As such, there was no vinculum juris
the ground that only Dahl-Jensen’s negligence was proven, not
between them as employer and employee. Respondent FILCAR
that of FILCAR. Hence, this instant petition.
cannot in any way be responsible for the negligent act of Dahl-
Jensen, the former not being an employer of the latter. In that
ISSUE:
case, the negligent and reckless operation of the truck owned by
petitioner corporation caused injuries to several persons and
Whether an action based on quasi-delict will prosper against a
damage to property. Intending to exculpate itself from liability,
rent-a-car company and, consequently, its insurer for fault or
the corporation raised the defense that at the time of the
negligence of the car lessee in driving the rented vehicle.
collision it had no more control over the vehicle as it was leased
to another; and, that the driver was not its employee but of the
HELD:
lessee. The trial court was not persuaded as it found that the true
nature of the alleged lease contract was nothing more than a
We find no reversible error committed by respondent court in
disguise effected by the corporation to relieve itself of the
upholding the dismissal of petitioner's complaint. The pertinent
burdens and responsibilities of an employer. We upheld this
provision is Art. 2176 of the Civil Code which states: "Whoever
finding and affirmed the declaration of joint and several liability
by act or omission causes damage to another, there being fault or
of the corporation with its driver.
negligence, is obliged to pay for the damage done. Such fault or
10
Same with case number 38

You might also like